Feat: sujet dS8 pour les TST_sti2d
continuous-integration/drone/push Build is passing Details

This commit is contained in:
Bertrand Benjamin 2021-04-08 12:12:13 +02:00
parent 5b0de2d51d
commit 04b6545631
43 changed files with 5605 additions and 0 deletions

View File

@ -0,0 +1,136 @@
\documentclass[a4paper,10pt]{article}
\usepackage{myXsim}
% Title Page
\title{DS8 \hfill BAHBAH Zakaria}
\tribe{TST sti2d}
\date{\hfillÀ render pour le vendredi 9 avril à 10h au plus tard}
\xsimsetup{
solution/print = false
}
\begin{document}
\maketitle
\begin{exercise}[subtitle={Étude de fonction}]
On considère la fonction $f$ définie sur $\intOF{0}{+\infty}$ par $ f(x) = 4x^2 + 72x + 160\ln(x)$
\begin{enumerate}
\item Démontrer que la dérivée de $f$ est $f'(x) = \frac{8x^2 + 72x + 160}{x}$.
\item Étude du numérateur de $f'(x)$: $N(x) = 8x^2 + 72x + 160$
\begin{enumerate}
\item Démontrer que $x=- 4$ et $x=- 5$ sont deux racines de $N(x)$..
\item Proposer une forme factorisée de $N(x)$.
\item Proposer une forme factorisée de $f'(x)$.
\end{enumerate}
\item Étudier le signe de $f'$ et en déduire les variations de $f$.
\end{enumerate}
\end{exercise}
\begin{solution}
\begin{enumerate}
\item pas de correction disponible
\item
\begin{enumerate}
\item \[N(- 4) = 0\]
\[N(- 5) = 0\]
\item \[
N(x) = 8(x - - 4)(x - - 5)
\]
\item
\[
f'(x) = \frac{8(x - - 4)(x - - 5)}{x}
\]
\end{enumerate}
\item Pas de correction disponible
\end{enumerate}
\end{solution}
\begin{exercise}[subtitle={Complexes}]
\begin{enumerate}
\item Mettre le nombre complexe suivant sous forme algébrique $z_1 = \dfrac{2 + 8 i}{-9 + 3 i} $
\item Mettre le complexe suivante sous forme exponentielle $z_2 = 10 \sqrt{3} - 10 i$
\item Mettre le complexe suivante sous forme exponentielle $z_3 = 7 + 7 \sqrt{3} i$
\item Calculer le produit $z_4=z_2\times z_3$ donner le résultat sous forme exponentielle puis algébrique.
\item Calculer le quotient $z_5=\frac{z_2}{z_3}$ donner le résultat sous forme exponentielle puis algébrique.
\end{enumerate}
\end{exercise}
\begin{solution}
\begin{enumerate}
\item $z_1 = \frac{1}{15} - \frac{13 i}{15}$
\item $z_2 = 20 e^{- \frac{i \pi}{6}}$
\item $z_3 = 14 e^{\frac{i \pi}{3}}$
\item $z_4 = 280 e^{\frac{i \pi}{6}} = 140 \sqrt{3} + 140 i = 243.0 + 140.0 i$
\item $z_5 = \frac{10}{7} e^{- \frac{i \pi}{2}} = - \frac{10 i}{7} = - 1.43 i$
\end{enumerate}
\end{solution}
\begin{exercise}[subtitle={Sortie du congélateur}]
Marie a invité quelques amis pour le thé. Elle souhaite leur proposer ses macarons maison.
Elle les sort de son congélateur à $-17$~\degres C et les place dans une pièce à $16$~\degres C.
Au bout de 15 minutes, la température des macarons est de $-3$~\degres C.
\bigskip
\textbf{Premier modèle}
\medskip
On suppose que la vitesse de décongélation est constante : chaque minute la hausse de
température des macarons est la même.
Estimer dans ce cadre la température au bout de $30$~minutes, puis au bout de $45$~minutes.
Cette modélisation est-elle pertinente?
\bigskip
\textbf{Deuxième modèle}
\medskip
On suppose maintenant que la vitesse de décongélation est proportionnelle à la différence
de température entre les macarons et l'air ambiant (il s'agit de la loi de Newton).
On désigne par $\theta$ la température des macarons à l'instant $t$, et par $\theta'$ la vitesse de décongélation.
L'unité de temps est la minute et l'unité de température le degré Celsius.
\smallskip
On négligera la diminution de température de la pièce et on admettra donc qu'il existe un
nombre réel $a$ tel que, pour $t$ positif :
\[\theta'(t) = a [\theta(t) - 16]\quad (E)\]
\medskip
\begin{enumerate}
\item Vérifier que l'équation $(E)$ a pour solutions $\theta(t) = K e^{at} + 16$$K$ est un nombre réel.
Donner alors, en fonction de $a$, l'ensemble des solutions de $(E)$.
\end{enumerate}
On rappelle que la température des macarons à l'instant $t = 0$ est égale à $-17$~\degres C et que, au bout de $15$~min, elle est de $-3$~\degres C.
\begin{enumerate}
\setcounter{enumi}{1}
\item En utilisant la condition à $t=0$ démontrer que $K = -33$.
\item En utilisant la condition à $t=15$ démontrer que $a \approx -0.04$.
\item En déduire l'expression de la solution de l'équation différentielle puis étudier ses variations.
\item La température idéale de dégustation des macarons étant de $13$~\degres C, Marie estime que
celle-ci sera atteinte au bout de $30$~min. A-t-elle raison ? Justifier la réponse.
Sinon, combien de temps faudra-t-il attendre ?
\end{enumerate}
\end{exercise}
\end{document}
%%% Local Variables:
%%% mode: latex
%%% TeX-master: "master"
%%% End:

View File

@ -0,0 +1,136 @@
\documentclass[a4paper,10pt]{article}
\usepackage{myXsim}
% Title Page
\title{DS8 \hfill BENALI Ilyas}
\tribe{TST sti2d}
\date{\hfillÀ render pour le vendredi 9 avril à 10h au plus tard}
\xsimsetup{
solution/print = false
}
\begin{document}
\maketitle
\begin{exercise}[subtitle={Étude de fonction}]
On considère la fonction $f$ définie sur $\intOF{0}{+\infty}$ par $ f(x) = 2.5x^2 + - 95x + 450\ln(x)$
\begin{enumerate}
\item Démontrer que la dérivée de $f$ est $f'(x) = \frac{5x^2 + - 95x + 450}{x}$.
\item Étude du numérateur de $f'(x)$: $N(x) = 5x^2 - 95x + 450$
\begin{enumerate}
\item Démontrer que $x=9$ et $x=10$ sont deux racines de $N(x)$..
\item Proposer une forme factorisée de $N(x)$.
\item Proposer une forme factorisée de $f'(x)$.
\end{enumerate}
\item Étudier le signe de $f'$ et en déduire les variations de $f$.
\end{enumerate}
\end{exercise}
\begin{solution}
\begin{enumerate}
\item pas de correction disponible
\item
\begin{enumerate}
\item \[N(9) = 0\]
\[N(10) = 0\]
\item \[
N(x) = 5(x - 9)(x - 10)
\]
\item
\[
f'(x) = \frac{5(x - 9)(x - 10)}{x}
\]
\end{enumerate}
\item Pas de correction disponible
\end{enumerate}
\end{solution}
\begin{exercise}[subtitle={Complexes}]
\begin{enumerate}
\item Mettre le nombre complexe suivant sous forme algébrique $z_1 = \dfrac{10 + 5 i}{-3 + 10 i} $
\item Mettre le complexe suivante sous forme exponentielle $z_2 = - 5 \sqrt{3} + 5 i$
\item Mettre le complexe suivante sous forme exponentielle $z_3 = - 8 \sqrt{2} - 8 \sqrt{2} i$
\item Calculer le produit $z_4=z_2\times z_3$ donner le résultat sous forme exponentielle puis algébrique.
\item Calculer le quotient $z_5=\frac{z_2}{z_3}$ donner le résultat sous forme exponentielle puis algébrique.
\end{enumerate}
\end{exercise}
\begin{solution}
\begin{enumerate}
\item $z_1 = \frac{20}{109} - \frac{115 i}{109}$
\item $z_2 = 10 e^{\frac{5 i \pi}{6}}$
\item $z_3 = 16 e^{- \frac{3 i \pi}{4}}$
\item $z_4 = 160 e^{\frac{i \pi}{12}} = 40 \sqrt{2} + 40 \sqrt{6} + i \left(- 40 \sqrt{2} + 40 \sqrt{6}\right) = 155.0 + 41.4 i$
\item $z_5 = \frac{5}{8} e^{\frac{19 i \pi}{12}} = - \frac{5 \sqrt{2}}{32} + \frac{5 \sqrt{6}}{32} + i \left(- \frac{5 \sqrt{6}}{32} - \frac{5 \sqrt{2}}{32}\right) = 0.162 - 0.604 i$
\end{enumerate}
\end{solution}
\begin{exercise}[subtitle={Sortie du congélateur}]
Marie a invité quelques amis pour le thé. Elle souhaite leur proposer ses macarons maison.
Elle les sort de son congélateur à $-17$~\degres C et les place dans une pièce à $17$~\degres C.
Au bout de 15 minutes, la température des macarons est de $-1$~\degres C.
\bigskip
\textbf{Premier modèle}
\medskip
On suppose que la vitesse de décongélation est constante : chaque minute la hausse de
température des macarons est la même.
Estimer dans ce cadre la température au bout de $30$~minutes, puis au bout de $45$~minutes.
Cette modélisation est-elle pertinente?
\bigskip
\textbf{Deuxième modèle}
\medskip
On suppose maintenant que la vitesse de décongélation est proportionnelle à la différence
de température entre les macarons et l'air ambiant (il s'agit de la loi de Newton).
On désigne par $\theta$ la température des macarons à l'instant $t$, et par $\theta'$ la vitesse de décongélation.
L'unité de temps est la minute et l'unité de température le degré Celsius.
\smallskip
On négligera la diminution de température de la pièce et on admettra donc qu'il existe un
nombre réel $a$ tel que, pour $t$ positif :
\[\theta'(t) = a [\theta(t) - 17]\quad (E)\]
\medskip
\begin{enumerate}
\item Vérifier que l'équation $(E)$ a pour solutions $\theta(t) = K e^{at} + 17$$K$ est un nombre réel.
Donner alors, en fonction de $a$, l'ensemble des solutions de $(E)$.
\end{enumerate}
On rappelle que la température des macarons à l'instant $t = 0$ est égale à $-17$~\degres C et que, au bout de $15$~min, elle est de $-1$~\degres C.
\begin{enumerate}
\setcounter{enumi}{1}
\item En utilisant la condition à $t=0$ démontrer que $K = -34$.
\item En utilisant la condition à $t=15$ démontrer que $a \approx -0.04$.
\item En déduire l'expression de la solution de l'équation différentielle puis étudier ses variations.
\item La température idéale de dégustation des macarons étant de $14$~\degres C, Marie estime que
celle-ci sera atteinte au bout de $30$~min. A-t-elle raison ? Justifier la réponse.
Sinon, combien de temps faudra-t-il attendre ?
\end{enumerate}
\end{exercise}
\end{document}
%%% Local Variables:
%%% mode: latex
%%% TeX-master: "master"
%%% End:

View File

@ -0,0 +1,136 @@
\documentclass[a4paper,10pt]{article}
\usepackage{myXsim}
% Title Page
\title{DS8 \hfill BERNADAT Noah}
\tribe{TST sti2d}
\date{\hfillÀ render pour le vendredi 9 avril à 10h au plus tard}
\xsimsetup{
solution/print = false
}
\begin{document}
\maketitle
\begin{exercise}[subtitle={Étude de fonction}]
On considère la fonction $f$ définie sur $\intOF{0}{+\infty}$ par $ f(x) = 3.5x^2 + - 21x + - 196\ln(x)$
\begin{enumerate}
\item Démontrer que la dérivée de $f$ est $f'(x) = \frac{7x^2 + - 21x + - 196}{x}$.
\item Étude du numérateur de $f'(x)$: $N(x) = 7x^2 - 21x - 196$
\begin{enumerate}
\item Démontrer que $x=- 4$ et $x=7$ sont deux racines de $N(x)$..
\item Proposer une forme factorisée de $N(x)$.
\item Proposer une forme factorisée de $f'(x)$.
\end{enumerate}
\item Étudier le signe de $f'$ et en déduire les variations de $f$.
\end{enumerate}
\end{exercise}
\begin{solution}
\begin{enumerate}
\item pas de correction disponible
\item
\begin{enumerate}
\item \[N(- 4) = 0\]
\[N(7) = 0\]
\item \[
N(x) = 7(x - - 4)(x - 7)
\]
\item
\[
f'(x) = \frac{7(x - - 4)(x - 7)}{x}
\]
\end{enumerate}
\item Pas de correction disponible
\end{enumerate}
\end{solution}
\begin{exercise}[subtitle={Complexes}]
\begin{enumerate}
\item Mettre le nombre complexe suivant sous forme algébrique $z_1 = \dfrac{8 + 9 i}{-9 + 5 i} $
\item Mettre le complexe suivante sous forme exponentielle $z_2 = - 8 \sqrt{3} + 8 i$
\item Mettre le complexe suivante sous forme exponentielle $z_3 = -5 - 5 \sqrt{3} i$
\item Calculer le produit $z_4=z_2\times z_3$ donner le résultat sous forme exponentielle puis algébrique.
\item Calculer le quotient $z_5=\frac{z_2}{z_3}$ donner le résultat sous forme exponentielle puis algébrique.
\end{enumerate}
\end{exercise}
\begin{solution}
\begin{enumerate}
\item $z_1 = - \frac{27}{106} - \frac{121 i}{106}$
\item $z_2 = 16 e^{\frac{5 i \pi}{6}}$
\item $z_3 = 10 e^{- \frac{2 i \pi}{3}}$
\item $z_4 = 160 e^{\frac{i \pi}{6}} = 80 \sqrt{3} + 80 i = 139.0 + 80.0 i$
\item $z_5 = \frac{8}{5} e^{\frac{3 i \pi}{2}} = - \frac{8 i}{5} = - 1.6 i$
\end{enumerate}
\end{solution}
\begin{exercise}[subtitle={Sortie du congélateur}]
Marie a invité quelques amis pour le thé. Elle souhaite leur proposer ses macarons maison.
Elle les sort de son congélateur à $-15$~\degres C et les place dans une pièce à $22$~\degres C.
Au bout de 15 minutes, la température des macarons est de $-1$~\degres C.
\bigskip
\textbf{Premier modèle}
\medskip
On suppose que la vitesse de décongélation est constante : chaque minute la hausse de
température des macarons est la même.
Estimer dans ce cadre la température au bout de $30$~minutes, puis au bout de $45$~minutes.
Cette modélisation est-elle pertinente?
\bigskip
\textbf{Deuxième modèle}
\medskip
On suppose maintenant que la vitesse de décongélation est proportionnelle à la différence
de température entre les macarons et l'air ambiant (il s'agit de la loi de Newton).
On désigne par $\theta$ la température des macarons à l'instant $t$, et par $\theta'$ la vitesse de décongélation.
L'unité de temps est la minute et l'unité de température le degré Celsius.
\smallskip
On négligera la diminution de température de la pièce et on admettra donc qu'il existe un
nombre réel $a$ tel que, pour $t$ positif :
\[\theta'(t) = a [\theta(t) - 22]\quad (E)\]
\medskip
\begin{enumerate}
\item Vérifier que l'équation $(E)$ a pour solutions $\theta(t) = K e^{at} + 22$$K$ est un nombre réel.
Donner alors, en fonction de $a$, l'ensemble des solutions de $(E)$.
\end{enumerate}
On rappelle que la température des macarons à l'instant $t = 0$ est égale à $-15$~\degres C et que, au bout de $15$~min, elle est de $-1$~\degres C.
\begin{enumerate}
\setcounter{enumi}{1}
\item En utilisant la condition à $t=0$ démontrer que $K = -37$.
\item En utilisant la condition à $t=15$ démontrer que $a \approx -0.03$.
\item En déduire l'expression de la solution de l'équation différentielle puis étudier ses variations.
\item La température idéale de dégustation des macarons étant de $19$~\degres C, Marie estime que
celle-ci sera atteinte au bout de $30$~min. A-t-elle raison ? Justifier la réponse.
Sinon, combien de temps faudra-t-il attendre ?
\end{enumerate}
\end{exercise}
\end{document}
%%% Local Variables:
%%% mode: latex
%%% TeX-master: "master"
%%% End:

View File

@ -0,0 +1,136 @@
\documentclass[a4paper,10pt]{article}
\usepackage{myXsim}
% Title Page
\title{DS8 \hfill BUDIN Nathan}
\tribe{TST sti2d}
\date{\hfillÀ render pour le vendredi 9 avril à 10h au plus tard}
\xsimsetup{
solution/print = false
}
\begin{document}
\maketitle
\begin{exercise}[subtitle={Étude de fonction}]
On considère la fonction $f$ définie sur $\intOF{0}{+\infty}$ par $ f(x) = 3.5x^2 + - 84x + 140\ln(x)$
\begin{enumerate}
\item Démontrer que la dérivée de $f$ est $f'(x) = \frac{7x^2 + - 84x + 140}{x}$.
\item Étude du numérateur de $f'(x)$: $N(x) = 7x^2 - 84x + 140$
\begin{enumerate}
\item Démontrer que $x=10$ et $x=2$ sont deux racines de $N(x)$..
\item Proposer une forme factorisée de $N(x)$.
\item Proposer une forme factorisée de $f'(x)$.
\end{enumerate}
\item Étudier le signe de $f'$ et en déduire les variations de $f$.
\end{enumerate}
\end{exercise}
\begin{solution}
\begin{enumerate}
\item pas de correction disponible
\item
\begin{enumerate}
\item \[N(10) = 0\]
\[N(2) = 0\]
\item \[
N(x) = 7(x - 10)(x - 2)
\]
\item
\[
f'(x) = \frac{7(x - 10)(x - 2)}{x}
\]
\end{enumerate}
\item Pas de correction disponible
\end{enumerate}
\end{solution}
\begin{exercise}[subtitle={Complexes}]
\begin{enumerate}
\item Mettre le nombre complexe suivant sous forme algébrique $z_1 = \dfrac{8 + 8 i}{-7 + 4 i} $
\item Mettre le complexe suivante sous forme exponentielle $z_2 = 7 \sqrt{3} - 7 i$
\item Mettre le complexe suivante sous forme exponentielle $z_3 = - 2 \sqrt{3} + 2 i$
\item Calculer le produit $z_4=z_2\times z_3$ donner le résultat sous forme exponentielle puis algébrique.
\item Calculer le quotient $z_5=\frac{z_2}{z_3}$ donner le résultat sous forme exponentielle puis algébrique.
\end{enumerate}
\end{exercise}
\begin{solution}
\begin{enumerate}
\item $z_1 = - \frac{24}{65} - \frac{88 i}{65}$
\item $z_2 = 14 e^{- \frac{i \pi}{6}}$
\item $z_3 = 4 e^{\frac{5 i \pi}{6}}$
\item $z_4 = 56 e^{\frac{2 i \pi}{3}} = -28 + 28 \sqrt{3} i = -28.0 + 48.5 i$
\item $z_5 = \frac{7}{2} e^{- i \pi} = - \frac{7}{2} = -3.5$
\end{enumerate}
\end{solution}
\begin{exercise}[subtitle={Sortie du congélateur}]
Marie a invité quelques amis pour le thé. Elle souhaite leur proposer ses macarons maison.
Elle les sort de son congélateur à $-19$~\degres C et les place dans une pièce à $23$~\degres C.
Au bout de 15 minutes, la température des macarons est de $3$~\degres C.
\bigskip
\textbf{Premier modèle}
\medskip
On suppose que la vitesse de décongélation est constante : chaque minute la hausse de
température des macarons est la même.
Estimer dans ce cadre la température au bout de $30$~minutes, puis au bout de $45$~minutes.
Cette modélisation est-elle pertinente?
\bigskip
\textbf{Deuxième modèle}
\medskip
On suppose maintenant que la vitesse de décongélation est proportionnelle à la différence
de température entre les macarons et l'air ambiant (il s'agit de la loi de Newton).
On désigne par $\theta$ la température des macarons à l'instant $t$, et par $\theta'$ la vitesse de décongélation.
L'unité de temps est la minute et l'unité de température le degré Celsius.
\smallskip
On négligera la diminution de température de la pièce et on admettra donc qu'il existe un
nombre réel $a$ tel que, pour $t$ positif :
\[\theta'(t) = a [\theta(t) - 23]\quad (E)\]
\medskip
\begin{enumerate}
\item Vérifier que l'équation $(E)$ a pour solutions $\theta(t) = K e^{at} + 23$$K$ est un nombre réel.
Donner alors, en fonction de $a$, l'ensemble des solutions de $(E)$.
\end{enumerate}
On rappelle que la température des macarons à l'instant $t = 0$ est égale à $-19$~\degres C et que, au bout de $15$~min, elle est de $3$~\degres C.
\begin{enumerate}
\setcounter{enumi}{1}
\item En utilisant la condition à $t=0$ démontrer que $K = -42$.
\item En utilisant la condition à $t=15$ démontrer que $a \approx -0.05$.
\item En déduire l'expression de la solution de l'équation différentielle puis étudier ses variations.
\item La température idéale de dégustation des macarons étant de $20$~\degres C, Marie estime que
celle-ci sera atteinte au bout de $30$~min. A-t-elle raison ? Justifier la réponse.
Sinon, combien de temps faudra-t-il attendre ?
\end{enumerate}
\end{exercise}
\end{document}
%%% Local Variables:
%%% mode: latex
%%% TeX-master: "master"
%%% End:

View File

@ -0,0 +1,136 @@
\documentclass[a4paper,10pt]{article}
\usepackage{myXsim}
% Title Page
\title{DS8 \hfill CHION Léa}
\tribe{TST sti2d}
\date{\hfillÀ render pour le vendredi 9 avril à 10h au plus tard}
\xsimsetup{
solution/print = false
}
\begin{document}
\maketitle
\begin{exercise}[subtitle={Étude de fonction}]
On considère la fonction $f$ définie sur $\intOF{0}{+\infty}$ par $ f(x) = 5x^2 + 130x + 300\ln(x)$
\begin{enumerate}
\item Démontrer que la dérivée de $f$ est $f'(x) = \frac{10x^2 + 130x + 300}{x}$.
\item Étude du numérateur de $f'(x)$: $N(x) = 10x^2 + 130x + 300$
\begin{enumerate}
\item Démontrer que $x=- 10$ et $x=- 3$ sont deux racines de $N(x)$..
\item Proposer une forme factorisée de $N(x)$.
\item Proposer une forme factorisée de $f'(x)$.
\end{enumerate}
\item Étudier le signe de $f'$ et en déduire les variations de $f$.
\end{enumerate}
\end{exercise}
\begin{solution}
\begin{enumerate}
\item pas de correction disponible
\item
\begin{enumerate}
\item \[N(- 10) = 0\]
\[N(- 3) = 0\]
\item \[
N(x) = 10(x - - 10)(x - - 3)
\]
\item
\[
f'(x) = \frac{10(x - - 10)(x - - 3)}{x}
\]
\end{enumerate}
\item Pas de correction disponible
\end{enumerate}
\end{solution}
\begin{exercise}[subtitle={Complexes}]
\begin{enumerate}
\item Mettre le nombre complexe suivant sous forme algébrique $z_1 = \dfrac{8 + 10 i}{-3 + 2 i} $
\item Mettre le complexe suivante sous forme exponentielle $z_2 = - \sqrt{2} - \sqrt{2} i$
\item Mettre le complexe suivante sous forme exponentielle $z_3 = - 10 \sqrt{2} + 10 \sqrt{2} i$
\item Calculer le produit $z_4=z_2\times z_3$ donner le résultat sous forme exponentielle puis algébrique.
\item Calculer le quotient $z_5=\frac{z_2}{z_3}$ donner le résultat sous forme exponentielle puis algébrique.
\end{enumerate}
\end{exercise}
\begin{solution}
\begin{enumerate}
\item $z_1 = - \frac{4}{13} - \frac{46 i}{13}$
\item $z_2 = 2 e^{- \frac{3 i \pi}{4}}$
\item $z_3 = 20 e^{\frac{3 i \pi}{4}}$
\item $z_4 = 40 e^{0} = 40 = 40.0$
\item $z_5 = \frac{1}{10} e^{- \frac{3 i \pi}{2}} = \frac{i}{10} = 0.1 i$
\end{enumerate}
\end{solution}
\begin{exercise}[subtitle={Sortie du congélateur}]
Marie a invité quelques amis pour le thé. Elle souhaite leur proposer ses macarons maison.
Elle les sort de son congélateur à $-20$~\degres C et les place dans une pièce à $24$~\degres C.
Au bout de 15 minutes, la température des macarons est de $-3$~\degres C.
\bigskip
\textbf{Premier modèle}
\medskip
On suppose que la vitesse de décongélation est constante : chaque minute la hausse de
température des macarons est la même.
Estimer dans ce cadre la température au bout de $30$~minutes, puis au bout de $45$~minutes.
Cette modélisation est-elle pertinente?
\bigskip
\textbf{Deuxième modèle}
\medskip
On suppose maintenant que la vitesse de décongélation est proportionnelle à la différence
de température entre les macarons et l'air ambiant (il s'agit de la loi de Newton).
On désigne par $\theta$ la température des macarons à l'instant $t$, et par $\theta'$ la vitesse de décongélation.
L'unité de temps est la minute et l'unité de température le degré Celsius.
\smallskip
On négligera la diminution de température de la pièce et on admettra donc qu'il existe un
nombre réel $a$ tel que, pour $t$ positif :
\[\theta'(t) = a [\theta(t) - 24]\quad (E)\]
\medskip
\begin{enumerate}
\item Vérifier que l'équation $(E)$ a pour solutions $\theta(t) = K e^{at} + 24$$K$ est un nombre réel.
Donner alors, en fonction de $a$, l'ensemble des solutions de $(E)$.
\end{enumerate}
On rappelle que la température des macarons à l'instant $t = 0$ est égale à $-20$~\degres C et que, au bout de $15$~min, elle est de $-3$~\degres C.
\begin{enumerate}
\setcounter{enumi}{1}
\item En utilisant la condition à $t=0$ démontrer que $K = -44$.
\item En utilisant la condition à $t=15$ démontrer que $a \approx -0.03$.
\item En déduire l'expression de la solution de l'équation différentielle puis étudier ses variations.
\item La température idéale de dégustation des macarons étant de $21$~\degres C, Marie estime que
celle-ci sera atteinte au bout de $30$~min. A-t-elle raison ? Justifier la réponse.
Sinon, combien de temps faudra-t-il attendre ?
\end{enumerate}
\end{exercise}
\end{document}
%%% Local Variables:
%%% mode: latex
%%% TeX-master: "master"
%%% End:

View File

@ -0,0 +1,136 @@
\documentclass[a4paper,10pt]{article}
\usepackage{myXsim}
% Title Page
\title{DS8 \hfill CLAIN Avinash}
\tribe{TST sti2d}
\date{\hfillÀ render pour le vendredi 9 avril à 10h au plus tard}
\xsimsetup{
solution/print = false
}
\begin{document}
\maketitle
\begin{exercise}[subtitle={Étude de fonction}]
On considère la fonction $f$ définie sur $\intOF{0}{+\infty}$ par $ f(x) = 3.5x^2 + - 42x + - 280\ln(x)$
\begin{enumerate}
\item Démontrer que la dérivée de $f$ est $f'(x) = \frac{7x^2 + - 42x + - 280}{x}$.
\item Étude du numérateur de $f'(x)$: $N(x) = 7x^2 - 42x - 280$
\begin{enumerate}
\item Démontrer que $x=10$ et $x=- 4$ sont deux racines de $N(x)$..
\item Proposer une forme factorisée de $N(x)$.
\item Proposer une forme factorisée de $f'(x)$.
\end{enumerate}
\item Étudier le signe de $f'$ et en déduire les variations de $f$.
\end{enumerate}
\end{exercise}
\begin{solution}
\begin{enumerate}
\item pas de correction disponible
\item
\begin{enumerate}
\item \[N(10) = 0\]
\[N(- 4) = 0\]
\item \[
N(x) = 7(x - 10)(x - - 4)
\]
\item
\[
f'(x) = \frac{7(x - 10)(x - - 4)}{x}
\]
\end{enumerate}
\item Pas de correction disponible
\end{enumerate}
\end{solution}
\begin{exercise}[subtitle={Complexes}]
\begin{enumerate}
\item Mettre le nombre complexe suivant sous forme algébrique $z_1 = \dfrac{3 + 8 i}{-10 + 5 i} $
\item Mettre le complexe suivante sous forme exponentielle $z_2 = 1 + \sqrt{3} i$
\item Mettre le complexe suivante sous forme exponentielle $z_3 = -3 + 3 \sqrt{3} i$
\item Calculer le produit $z_4=z_2\times z_3$ donner le résultat sous forme exponentielle puis algébrique.
\item Calculer le quotient $z_5=\frac{z_2}{z_3}$ donner le résultat sous forme exponentielle puis algébrique.
\end{enumerate}
\end{exercise}
\begin{solution}
\begin{enumerate}
\item $z_1 = \frac{2}{25} - \frac{19 i}{25}$
\item $z_2 = 2 e^{\frac{i \pi}{3}}$
\item $z_3 = 6 e^{\frac{2 i \pi}{3}}$
\item $z_4 = 12 e^{i \pi} = -12 = -12.0$
\item $z_5 = \frac{1}{3} e^{- \frac{i \pi}{3}} = \frac{1}{6} - \frac{\sqrt{3} i}{6} = 0.167 - 0.289 i$
\end{enumerate}
\end{solution}
\begin{exercise}[subtitle={Sortie du congélateur}]
Marie a invité quelques amis pour le thé. Elle souhaite leur proposer ses macarons maison.
Elle les sort de son congélateur à $-20$~\degres C et les place dans une pièce à $17$~\degres C.
Au bout de 15 minutes, la température des macarons est de $4$~\degres C.
\bigskip
\textbf{Premier modèle}
\medskip
On suppose que la vitesse de décongélation est constante : chaque minute la hausse de
température des macarons est la même.
Estimer dans ce cadre la température au bout de $30$~minutes, puis au bout de $45$~minutes.
Cette modélisation est-elle pertinente?
\bigskip
\textbf{Deuxième modèle}
\medskip
On suppose maintenant que la vitesse de décongélation est proportionnelle à la différence
de température entre les macarons et l'air ambiant (il s'agit de la loi de Newton).
On désigne par $\theta$ la température des macarons à l'instant $t$, et par $\theta'$ la vitesse de décongélation.
L'unité de temps est la minute et l'unité de température le degré Celsius.
\smallskip
On négligera la diminution de température de la pièce et on admettra donc qu'il existe un
nombre réel $a$ tel que, pour $t$ positif :
\[\theta'(t) = a [\theta(t) - 17]\quad (E)\]
\medskip
\begin{enumerate}
\item Vérifier que l'équation $(E)$ a pour solutions $\theta(t) = K e^{at} + 17$$K$ est un nombre réel.
Donner alors, en fonction de $a$, l'ensemble des solutions de $(E)$.
\end{enumerate}
On rappelle que la température des macarons à l'instant $t = 0$ est égale à $-20$~\degres C et que, au bout de $15$~min, elle est de $4$~\degres C.
\begin{enumerate}
\setcounter{enumi}{1}
\item En utilisant la condition à $t=0$ démontrer que $K = -37$.
\item En utilisant la condition à $t=15$ démontrer que $a \approx -0.07$.
\item En déduire l'expression de la solution de l'équation différentielle puis étudier ses variations.
\item La température idéale de dégustation des macarons étant de $14$~\degres C, Marie estime que
celle-ci sera atteinte au bout de $30$~min. A-t-elle raison ? Justifier la réponse.
Sinon, combien de temps faudra-t-il attendre ?
\end{enumerate}
\end{exercise}
\end{document}
%%% Local Variables:
%%% mode: latex
%%% TeX-master: "master"
%%% End:

View File

@ -0,0 +1,136 @@
\documentclass[a4paper,10pt]{article}
\usepackage{myXsim}
% Title Page
\title{DS8 \hfill COUBAT Alexis}
\tribe{TST sti2d}
\date{\hfillÀ render pour le vendredi 9 avril à 10h au plus tard}
\xsimsetup{
solution/print = false
}
\begin{document}
\maketitle
\begin{exercise}[subtitle={Étude de fonction}]
On considère la fonction $f$ définie sur $\intOF{0}{+\infty}$ par $ f(x) = 5x^2 + 70x + 60\ln(x)$
\begin{enumerate}
\item Démontrer que la dérivée de $f$ est $f'(x) = \frac{10x^2 + 70x + 60}{x}$.
\item Étude du numérateur de $f'(x)$: $N(x) = 10x^2 + 70x + 60$
\begin{enumerate}
\item Démontrer que $x=- 6$ et $x=- 1$ sont deux racines de $N(x)$..
\item Proposer une forme factorisée de $N(x)$.
\item Proposer une forme factorisée de $f'(x)$.
\end{enumerate}
\item Étudier le signe de $f'$ et en déduire les variations de $f$.
\end{enumerate}
\end{exercise}
\begin{solution}
\begin{enumerate}
\item pas de correction disponible
\item
\begin{enumerate}
\item \[N(- 6) = 0\]
\[N(- 1) = 0\]
\item \[
N(x) = 10(x - - 6)(x - - 1)
\]
\item
\[
f'(x) = \frac{10(x - - 6)(x - - 1)}{x}
\]
\end{enumerate}
\item Pas de correction disponible
\end{enumerate}
\end{solution}
\begin{exercise}[subtitle={Complexes}]
\begin{enumerate}
\item Mettre le nombre complexe suivant sous forme algébrique $z_1 = \dfrac{3 + 5 i}{-4 + 5 i} $
\item Mettre le complexe suivante sous forme exponentielle $z_2 = 5 + 5 \sqrt{3} i$
\item Mettre le complexe suivante sous forme exponentielle $z_3 = - 4 \sqrt{3} - 4 i$
\item Calculer le produit $z_4=z_2\times z_3$ donner le résultat sous forme exponentielle puis algébrique.
\item Calculer le quotient $z_5=\frac{z_2}{z_3}$ donner le résultat sous forme exponentielle puis algébrique.
\end{enumerate}
\end{exercise}
\begin{solution}
\begin{enumerate}
\item $z_1 = \frac{13}{41} - \frac{35 i}{41}$
\item $z_2 = 10 e^{\frac{i \pi}{3}}$
\item $z_3 = 8 e^{- \frac{5 i \pi}{6}}$
\item $z_4 = 80 e^{- \frac{i \pi}{2}} = - 80 i = - 80.0 i$
\item $z_5 = \frac{5}{4} e^{\frac{7 i \pi}{6}} = - \frac{5 \sqrt{3}}{8} - \frac{5 i}{8} = -1.08 - 0.625 i$
\end{enumerate}
\end{solution}
\begin{exercise}[subtitle={Sortie du congélateur}]
Marie a invité quelques amis pour le thé. Elle souhaite leur proposer ses macarons maison.
Elle les sort de son congélateur à $-20$~\degres C et les place dans une pièce à $21$~\degres C.
Au bout de 15 minutes, la température des macarons est de $4$~\degres C.
\bigskip
\textbf{Premier modèle}
\medskip
On suppose que la vitesse de décongélation est constante : chaque minute la hausse de
température des macarons est la même.
Estimer dans ce cadre la température au bout de $30$~minutes, puis au bout de $45$~minutes.
Cette modélisation est-elle pertinente?
\bigskip
\textbf{Deuxième modèle}
\medskip
On suppose maintenant que la vitesse de décongélation est proportionnelle à la différence
de température entre les macarons et l'air ambiant (il s'agit de la loi de Newton).
On désigne par $\theta$ la température des macarons à l'instant $t$, et par $\theta'$ la vitesse de décongélation.
L'unité de temps est la minute et l'unité de température le degré Celsius.
\smallskip
On négligera la diminution de température de la pièce et on admettra donc qu'il existe un
nombre réel $a$ tel que, pour $t$ positif :
\[\theta'(t) = a [\theta(t) - 21]\quad (E)\]
\medskip
\begin{enumerate}
\item Vérifier que l'équation $(E)$ a pour solutions $\theta(t) = K e^{at} + 21$$K$ est un nombre réel.
Donner alors, en fonction de $a$, l'ensemble des solutions de $(E)$.
\end{enumerate}
On rappelle que la température des macarons à l'instant $t = 0$ est égale à $-20$~\degres C et que, au bout de $15$~min, elle est de $4$~\degres C.
\begin{enumerate}
\setcounter{enumi}{1}
\item En utilisant la condition à $t=0$ démontrer que $K = -41$.
\item En utilisant la condition à $t=15$ démontrer que $a \approx -0.06$.
\item En déduire l'expression de la solution de l'équation différentielle puis étudier ses variations.
\item La température idéale de dégustation des macarons étant de $18$~\degres C, Marie estime que
celle-ci sera atteinte au bout de $30$~min. A-t-elle raison ? Justifier la réponse.
Sinon, combien de temps faudra-t-il attendre ?
\end{enumerate}
\end{exercise}
\end{document}
%%% Local Variables:
%%% mode: latex
%%% TeX-master: "master"
%%% End:

View File

@ -0,0 +1,136 @@
\documentclass[a4paper,10pt]{article}
\usepackage{myXsim}
% Title Page
\title{DS8 \hfill EVRARD Jules}
\tribe{TST sti2d}
\date{\hfillÀ render pour le vendredi 9 avril à 10h au plus tard}
\xsimsetup{
solution/print = false
}
\begin{document}
\maketitle
\begin{exercise}[subtitle={Étude de fonction}]
On considère la fonction $f$ définie sur $\intOF{0}{+\infty}$ par $ f(x) = 5x^2 + 0x + - 360\ln(x)$
\begin{enumerate}
\item Démontrer que la dérivée de $f$ est $f'(x) = \frac{10x^2 + 0x + - 360}{x}$.
\item Étude du numérateur de $f'(x)$: $N(x) = 10x^2 - 360$
\begin{enumerate}
\item Démontrer que $x=6$ et $x=- 6$ sont deux racines de $N(x)$..
\item Proposer une forme factorisée de $N(x)$.
\item Proposer une forme factorisée de $f'(x)$.
\end{enumerate}
\item Étudier le signe de $f'$ et en déduire les variations de $f$.
\end{enumerate}
\end{exercise}
\begin{solution}
\begin{enumerate}
\item pas de correction disponible
\item
\begin{enumerate}
\item \[N(6) = 0\]
\[N(- 6) = 0\]
\item \[
N(x) = 10(x - 6)(x - - 6)
\]
\item
\[
f'(x) = \frac{10(x - 6)(x - - 6)}{x}
\]
\end{enumerate}
\item Pas de correction disponible
\end{enumerate}
\end{solution}
\begin{exercise}[subtitle={Complexes}]
\begin{enumerate}
\item Mettre le nombre complexe suivant sous forme algébrique $z_1 = \dfrac{10 + 8 i}{-5 + 7 i} $
\item Mettre le complexe suivante sous forme exponentielle $z_2 = 9 \sqrt{2} - 9 \sqrt{2} i$
\item Mettre le complexe suivante sous forme exponentielle $z_3 = 7 \sqrt{3} + 7 i$
\item Calculer le produit $z_4=z_2\times z_3$ donner le résultat sous forme exponentielle puis algébrique.
\item Calculer le quotient $z_5=\frac{z_2}{z_3}$ donner le résultat sous forme exponentielle puis algébrique.
\end{enumerate}
\end{exercise}
\begin{solution}
\begin{enumerate}
\item $z_1 = \frac{3}{37} - \frac{55 i}{37}$
\item $z_2 = 18 e^{- \frac{i \pi}{4}}$
\item $z_3 = 14 e^{\frac{i \pi}{6}}$
\item $z_4 = 252 e^{- \frac{i \pi}{12}} = 63 \sqrt{2} + 63 \sqrt{6} + i \left(- 63 \sqrt{6} + 63 \sqrt{2}\right) = 243.0 - 65.2 i$
\item $z_5 = \frac{9}{7} e^{- \frac{5 i \pi}{12}} = - \frac{9 \sqrt{2}}{28} + \frac{9 \sqrt{6}}{28} + i \left(- \frac{9 \sqrt{6}}{28} - \frac{9 \sqrt{2}}{28}\right) = 0.333 - 1.24 i$
\end{enumerate}
\end{solution}
\begin{exercise}[subtitle={Sortie du congélateur}]
Marie a invité quelques amis pour le thé. Elle souhaite leur proposer ses macarons maison.
Elle les sort de son congélateur à $-15$~\degres C et les place dans une pièce à $17$~\degres C.
Au bout de 15 minutes, la température des macarons est de $-1$~\degres C.
\bigskip
\textbf{Premier modèle}
\medskip
On suppose que la vitesse de décongélation est constante : chaque minute la hausse de
température des macarons est la même.
Estimer dans ce cadre la température au bout de $30$~minutes, puis au bout de $45$~minutes.
Cette modélisation est-elle pertinente?
\bigskip
\textbf{Deuxième modèle}
\medskip
On suppose maintenant que la vitesse de décongélation est proportionnelle à la différence
de température entre les macarons et l'air ambiant (il s'agit de la loi de Newton).
On désigne par $\theta$ la température des macarons à l'instant $t$, et par $\theta'$ la vitesse de décongélation.
L'unité de temps est la minute et l'unité de température le degré Celsius.
\smallskip
On négligera la diminution de température de la pièce et on admettra donc qu'il existe un
nombre réel $a$ tel que, pour $t$ positif :
\[\theta'(t) = a [\theta(t) - 17]\quad (E)\]
\medskip
\begin{enumerate}
\item Vérifier que l'équation $(E)$ a pour solutions $\theta(t) = K e^{at} + 17$$K$ est un nombre réel.
Donner alors, en fonction de $a$, l'ensemble des solutions de $(E)$.
\end{enumerate}
On rappelle que la température des macarons à l'instant $t = 0$ est égale à $-15$~\degres C et que, au bout de $15$~min, elle est de $-1$~\degres C.
\begin{enumerate}
\setcounter{enumi}{1}
\item En utilisant la condition à $t=0$ démontrer que $K = -32$.
\item En utilisant la condition à $t=15$ démontrer que $a \approx -0.04$.
\item En déduire l'expression de la solution de l'équation différentielle puis étudier ses variations.
\item La température idéale de dégustation des macarons étant de $14$~\degres C, Marie estime que
celle-ci sera atteinte au bout de $30$~min. A-t-elle raison ? Justifier la réponse.
Sinon, combien de temps faudra-t-il attendre ?
\end{enumerate}
\end{exercise}
\end{document}
%%% Local Variables:
%%% mode: latex
%%% TeX-master: "master"
%%% End:

View File

@ -0,0 +1,136 @@
\documentclass[a4paper,10pt]{article}
\usepackage{myXsim}
% Title Page
\title{DS8 \hfill HADJRAS Mohcine}
\tribe{TST sti2d}
\date{\hfillÀ render pour le vendredi 9 avril à 10h au plus tard}
\xsimsetup{
solution/print = false
}
\begin{document}
\maketitle
\begin{exercise}[subtitle={Étude de fonction}]
On considère la fonction $f$ définie sur $\intOF{0}{+\infty}$ par $ f(x) = 3x^2 + 78x + 240\ln(x)$
\begin{enumerate}
\item Démontrer que la dérivée de $f$ est $f'(x) = \frac{6x^2 + 78x + 240}{x}$.
\item Étude du numérateur de $f'(x)$: $N(x) = 6x^2 + 78x + 240$
\begin{enumerate}
\item Démontrer que $x=- 5$ et $x=- 8$ sont deux racines de $N(x)$..
\item Proposer une forme factorisée de $N(x)$.
\item Proposer une forme factorisée de $f'(x)$.
\end{enumerate}
\item Étudier le signe de $f'$ et en déduire les variations de $f$.
\end{enumerate}
\end{exercise}
\begin{solution}
\begin{enumerate}
\item pas de correction disponible
\item
\begin{enumerate}
\item \[N(- 5) = 0\]
\[N(- 8) = 0\]
\item \[
N(x) = 6(x - - 5)(x - - 8)
\]
\item
\[
f'(x) = \frac{6(x - - 5)(x - - 8)}{x}
\]
\end{enumerate}
\item Pas de correction disponible
\end{enumerate}
\end{solution}
\begin{exercise}[subtitle={Complexes}]
\begin{enumerate}
\item Mettre le nombre complexe suivant sous forme algébrique $z_1 = \dfrac{10 + 5 i}{-6 + 4 i} $
\item Mettre le complexe suivante sous forme exponentielle $z_2 = 1 + \sqrt{3} i$
\item Mettre le complexe suivante sous forme exponentielle $z_3 = -7 - 7 \sqrt{3} i$
\item Calculer le produit $z_4=z_2\times z_3$ donner le résultat sous forme exponentielle puis algébrique.
\item Calculer le quotient $z_5=\frac{z_2}{z_3}$ donner le résultat sous forme exponentielle puis algébrique.
\end{enumerate}
\end{exercise}
\begin{solution}
\begin{enumerate}
\item $z_1 = - \frac{10}{13} - \frac{35 i}{26}$
\item $z_2 = 2 e^{\frac{i \pi}{3}}$
\item $z_3 = 14 e^{- \frac{2 i \pi}{3}}$
\item $z_4 = 28 e^{- \frac{i \pi}{3}} = 14 - 14 \sqrt{3} i = 14.0 - 24.3 i$
\item $z_5 = \frac{1}{7} e^{i \pi} = - \frac{1}{7} = -0.143$
\end{enumerate}
\end{solution}
\begin{exercise}[subtitle={Sortie du congélateur}]
Marie a invité quelques amis pour le thé. Elle souhaite leur proposer ses macarons maison.
Elle les sort de son congélateur à $-18$~\degres C et les place dans une pièce à $25$~\degres C.
Au bout de 15 minutes, la température des macarons est de $4$~\degres C.
\bigskip
\textbf{Premier modèle}
\medskip
On suppose que la vitesse de décongélation est constante : chaque minute la hausse de
température des macarons est la même.
Estimer dans ce cadre la température au bout de $30$~minutes, puis au bout de $45$~minutes.
Cette modélisation est-elle pertinente?
\bigskip
\textbf{Deuxième modèle}
\medskip
On suppose maintenant que la vitesse de décongélation est proportionnelle à la différence
de température entre les macarons et l'air ambiant (il s'agit de la loi de Newton).
On désigne par $\theta$ la température des macarons à l'instant $t$, et par $\theta'$ la vitesse de décongélation.
L'unité de temps est la minute et l'unité de température le degré Celsius.
\smallskip
On négligera la diminution de température de la pièce et on admettra donc qu'il existe un
nombre réel $a$ tel que, pour $t$ positif :
\[\theta'(t) = a [\theta(t) - 25]\quad (E)\]
\medskip
\begin{enumerate}
\item Vérifier que l'équation $(E)$ a pour solutions $\theta(t) = K e^{at} + 25$$K$ est un nombre réel.
Donner alors, en fonction de $a$, l'ensemble des solutions de $(E)$.
\end{enumerate}
On rappelle que la température des macarons à l'instant $t = 0$ est égale à $-18$~\degres C et que, au bout de $15$~min, elle est de $4$~\degres C.
\begin{enumerate}
\setcounter{enumi}{1}
\item En utilisant la condition à $t=0$ démontrer que $K = -43$.
\item En utilisant la condition à $t=15$ démontrer que $a \approx -0.05$.
\item En déduire l'expression de la solution de l'équation différentielle puis étudier ses variations.
\item La température idéale de dégustation des macarons étant de $22$~\degres C, Marie estime que
celle-ci sera atteinte au bout de $30$~min. A-t-elle raison ? Justifier la réponse.
Sinon, combien de temps faudra-t-il attendre ?
\end{enumerate}
\end{exercise}
\end{document}
%%% Local Variables:
%%% mode: latex
%%% TeX-master: "master"
%%% End:

View File

@ -0,0 +1,136 @@
\documentclass[a4paper,10pt]{article}
\usepackage{myXsim}
% Title Page
\title{DS8 \hfill HENRIST Maxime}
\tribe{TST sti2d}
\date{\hfillÀ render pour le vendredi 9 avril à 10h au plus tard}
\xsimsetup{
solution/print = false
}
\begin{document}
\maketitle
\begin{exercise}[subtitle={Étude de fonction}]
On considère la fonction $f$ définie sur $\intOF{0}{+\infty}$ par $ f(x) = 4.5x^2 + - 27x + - 36\ln(x)$
\begin{enumerate}
\item Démontrer que la dérivée de $f$ est $f'(x) = \frac{9x^2 + - 27x + - 36}{x}$.
\item Étude du numérateur de $f'(x)$: $N(x) = 9x^2 - 27x - 36$
\begin{enumerate}
\item Démontrer que $x=4$ et $x=- 1$ sont deux racines de $N(x)$..
\item Proposer une forme factorisée de $N(x)$.
\item Proposer une forme factorisée de $f'(x)$.
\end{enumerate}
\item Étudier le signe de $f'$ et en déduire les variations de $f$.
\end{enumerate}
\end{exercise}
\begin{solution}
\begin{enumerate}
\item pas de correction disponible
\item
\begin{enumerate}
\item \[N(4) = 0\]
\[N(- 1) = 0\]
\item \[
N(x) = 9(x - 4)(x - - 1)
\]
\item
\[
f'(x) = \frac{9(x - 4)(x - - 1)}{x}
\]
\end{enumerate}
\item Pas de correction disponible
\end{enumerate}
\end{solution}
\begin{exercise}[subtitle={Complexes}]
\begin{enumerate}
\item Mettre le nombre complexe suivant sous forme algébrique $z_1 = \dfrac{6 + 5 i}{-6 + 5 i} $
\item Mettre le complexe suivante sous forme exponentielle $z_2 = 8 \sqrt{3} - 8 i$
\item Mettre le complexe suivante sous forme exponentielle $z_3 = 5 - 5 \sqrt{3} i$
\item Calculer le produit $z_4=z_2\times z_3$ donner le résultat sous forme exponentielle puis algébrique.
\item Calculer le quotient $z_5=\frac{z_2}{z_3}$ donner le résultat sous forme exponentielle puis algébrique.
\end{enumerate}
\end{exercise}
\begin{solution}
\begin{enumerate}
\item $z_1 = - \frac{11}{61} - \frac{60 i}{61}$
\item $z_2 = 16 e^{- \frac{i \pi}{6}}$
\item $z_3 = 10 e^{- \frac{i \pi}{3}}$
\item $z_4 = 160 e^{- \frac{i \pi}{2}} = - 160 i = - 160.0 i$
\item $z_5 = \frac{8}{5} e^{\frac{i \pi}{6}} = \frac{4 \sqrt{3}}{5} + \frac{4 i}{5} = 1.39 + 0.8 i$
\end{enumerate}
\end{solution}
\begin{exercise}[subtitle={Sortie du congélateur}]
Marie a invité quelques amis pour le thé. Elle souhaite leur proposer ses macarons maison.
Elle les sort de son congélateur à $-16$~\degres C et les place dans une pièce à $23$~\degres C.
Au bout de 15 minutes, la température des macarons est de $0$~\degres C.
\bigskip
\textbf{Premier modèle}
\medskip
On suppose que la vitesse de décongélation est constante : chaque minute la hausse de
température des macarons est la même.
Estimer dans ce cadre la température au bout de $30$~minutes, puis au bout de $45$~minutes.
Cette modélisation est-elle pertinente?
\bigskip
\textbf{Deuxième modèle}
\medskip
On suppose maintenant que la vitesse de décongélation est proportionnelle à la différence
de température entre les macarons et l'air ambiant (il s'agit de la loi de Newton).
On désigne par $\theta$ la température des macarons à l'instant $t$, et par $\theta'$ la vitesse de décongélation.
L'unité de temps est la minute et l'unité de température le degré Celsius.
\smallskip
On négligera la diminution de température de la pièce et on admettra donc qu'il existe un
nombre réel $a$ tel que, pour $t$ positif :
\[\theta'(t) = a [\theta(t) - 23]\quad (E)\]
\medskip
\begin{enumerate}
\item Vérifier que l'équation $(E)$ a pour solutions $\theta(t) = K e^{at} + 23$$K$ est un nombre réel.
Donner alors, en fonction de $a$, l'ensemble des solutions de $(E)$.
\end{enumerate}
On rappelle que la température des macarons à l'instant $t = 0$ est égale à $-16$~\degres C et que, au bout de $15$~min, elle est de $0$~\degres C.
\begin{enumerate}
\setcounter{enumi}{1}
\item En utilisant la condition à $t=0$ démontrer que $K = -39$.
\item En utilisant la condition à $t=15$ démontrer que $a \approx -0.04$.
\item En déduire l'expression de la solution de l'équation différentielle puis étudier ses variations.
\item La température idéale de dégustation des macarons étant de $20$~\degres C, Marie estime que
celle-ci sera atteinte au bout de $30$~min. A-t-elle raison ? Justifier la réponse.
Sinon, combien de temps faudra-t-il attendre ?
\end{enumerate}
\end{exercise}
\end{document}
%%% Local Variables:
%%% mode: latex
%%% TeX-master: "master"
%%% End:

View File

@ -0,0 +1,136 @@
\documentclass[a4paper,10pt]{article}
\usepackage{myXsim}
% Title Page
\title{DS8 \hfill HUMBERT Rayan}
\tribe{TST sti2d}
\date{\hfillÀ render pour le vendredi 9 avril à 10h au plus tard}
\xsimsetup{
solution/print = false
}
\begin{document}
\maketitle
\begin{exercise}[subtitle={Étude de fonction}]
On considère la fonction $f$ définie sur $\intOF{0}{+\infty}$ par $ f(x) = 2.5x^2 + - 25x + - 180\ln(x)$
\begin{enumerate}
\item Démontrer que la dérivée de $f$ est $f'(x) = \frac{5x^2 + - 25x + - 180}{x}$.
\item Étude du numérateur de $f'(x)$: $N(x) = 5x^2 - 25x - 180$
\begin{enumerate}
\item Démontrer que $x=- 4$ et $x=9$ sont deux racines de $N(x)$..
\item Proposer une forme factorisée de $N(x)$.
\item Proposer une forme factorisée de $f'(x)$.
\end{enumerate}
\item Étudier le signe de $f'$ et en déduire les variations de $f$.
\end{enumerate}
\end{exercise}
\begin{solution}
\begin{enumerate}
\item pas de correction disponible
\item
\begin{enumerate}
\item \[N(- 4) = 0\]
\[N(9) = 0\]
\item \[
N(x) = 5(x - - 4)(x - 9)
\]
\item
\[
f'(x) = \frac{5(x - - 4)(x - 9)}{x}
\]
\end{enumerate}
\item Pas de correction disponible
\end{enumerate}
\end{solution}
\begin{exercise}[subtitle={Complexes}]
\begin{enumerate}
\item Mettre le nombre complexe suivant sous forme algébrique $z_1 = \dfrac{8 + 3 i}{-6 + 6 i} $
\item Mettre le complexe suivante sous forme exponentielle $z_2 = -4 - 4 \sqrt{3} i$
\item Mettre le complexe suivante sous forme exponentielle $z_3 = 8 \sqrt{3} + 8 i$
\item Calculer le produit $z_4=z_2\times z_3$ donner le résultat sous forme exponentielle puis algébrique.
\item Calculer le quotient $z_5=\frac{z_2}{z_3}$ donner le résultat sous forme exponentielle puis algébrique.
\end{enumerate}
\end{exercise}
\begin{solution}
\begin{enumerate}
\item $z_1 = - \frac{5}{12} - \frac{11 i}{12}$
\item $z_2 = 8 e^{- \frac{2 i \pi}{3}}$
\item $z_3 = 16 e^{\frac{i \pi}{6}}$
\item $z_4 = 128 e^{- \frac{i \pi}{2}} = - 128 i = - 128.0 i$
\item $z_5 = \frac{1}{2} e^{- \frac{5 i \pi}{6}} = - \frac{\sqrt{3}}{4} - \frac{i}{4} = -0.433 - 0.25 i$
\end{enumerate}
\end{solution}
\begin{exercise}[subtitle={Sortie du congélateur}]
Marie a invité quelques amis pour le thé. Elle souhaite leur proposer ses macarons maison.
Elle les sort de son congélateur à $-20$~\degres C et les place dans une pièce à $25$~\degres C.
Au bout de 15 minutes, la température des macarons est de $-3$~\degres C.
\bigskip
\textbf{Premier modèle}
\medskip
On suppose que la vitesse de décongélation est constante : chaque minute la hausse de
température des macarons est la même.
Estimer dans ce cadre la température au bout de $30$~minutes, puis au bout de $45$~minutes.
Cette modélisation est-elle pertinente?
\bigskip
\textbf{Deuxième modèle}
\medskip
On suppose maintenant que la vitesse de décongélation est proportionnelle à la différence
de température entre les macarons et l'air ambiant (il s'agit de la loi de Newton).
On désigne par $\theta$ la température des macarons à l'instant $t$, et par $\theta'$ la vitesse de décongélation.
L'unité de temps est la minute et l'unité de température le degré Celsius.
\smallskip
On négligera la diminution de température de la pièce et on admettra donc qu'il existe un
nombre réel $a$ tel que, pour $t$ positif :
\[\theta'(t) = a [\theta(t) - 25]\quad (E)\]
\medskip
\begin{enumerate}
\item Vérifier que l'équation $(E)$ a pour solutions $\theta(t) = K e^{at} + 25$$K$ est un nombre réel.
Donner alors, en fonction de $a$, l'ensemble des solutions de $(E)$.
\end{enumerate}
On rappelle que la température des macarons à l'instant $t = 0$ est égale à $-20$~\degres C et que, au bout de $15$~min, elle est de $-3$~\degres C.
\begin{enumerate}
\setcounter{enumi}{1}
\item En utilisant la condition à $t=0$ démontrer que $K = -45$.
\item En utilisant la condition à $t=15$ démontrer que $a \approx -0.03$.
\item En déduire l'expression de la solution de l'équation différentielle puis étudier ses variations.
\item La température idéale de dégustation des macarons étant de $22$~\degres C, Marie estime que
celle-ci sera atteinte au bout de $30$~min. A-t-elle raison ? Justifier la réponse.
Sinon, combien de temps faudra-t-il attendre ?
\end{enumerate}
\end{exercise}
\end{document}
%%% Local Variables:
%%% mode: latex
%%% TeX-master: "master"
%%% End:

View File

@ -0,0 +1,136 @@
\documentclass[a4paper,10pt]{article}
\usepackage{myXsim}
% Title Page
\title{DS8 \hfill KILINC Suleyman}
\tribe{TST sti2d}
\date{\hfillÀ render pour le vendredi 9 avril à 10h au plus tard}
\xsimsetup{
solution/print = false
}
\begin{document}
\maketitle
\begin{exercise}[subtitle={Étude de fonction}]
On considère la fonction $f$ définie sur $\intOF{0}{+\infty}$ par $ f(x) = 3.5x^2 + 21x + - 490\ln(x)$
\begin{enumerate}
\item Démontrer que la dérivée de $f$ est $f'(x) = \frac{7x^2 + 21x + - 490}{x}$.
\item Étude du numérateur de $f'(x)$: $N(x) = 7x^2 + 21x - 490$
\begin{enumerate}
\item Démontrer que $x=- 10$ et $x=7$ sont deux racines de $N(x)$..
\item Proposer une forme factorisée de $N(x)$.
\item Proposer une forme factorisée de $f'(x)$.
\end{enumerate}
\item Étudier le signe de $f'$ et en déduire les variations de $f$.
\end{enumerate}
\end{exercise}
\begin{solution}
\begin{enumerate}
\item pas de correction disponible
\item
\begin{enumerate}
\item \[N(- 10) = 0\]
\[N(7) = 0\]
\item \[
N(x) = 7(x - - 10)(x - 7)
\]
\item
\[
f'(x) = \frac{7(x - - 10)(x - 7)}{x}
\]
\end{enumerate}
\item Pas de correction disponible
\end{enumerate}
\end{solution}
\begin{exercise}[subtitle={Complexes}]
\begin{enumerate}
\item Mettre le nombre complexe suivant sous forme algébrique $z_1 = \dfrac{5 + 7 i}{-6 + 4 i} $
\item Mettre le complexe suivante sous forme exponentielle $z_2 = 1 - \sqrt{3} i$
\item Mettre le complexe suivante sous forme exponentielle $z_3 = - 8 \sqrt{2} + 8 \sqrt{2} i$
\item Calculer le produit $z_4=z_2\times z_3$ donner le résultat sous forme exponentielle puis algébrique.
\item Calculer le quotient $z_5=\frac{z_2}{z_3}$ donner le résultat sous forme exponentielle puis algébrique.
\end{enumerate}
\end{exercise}
\begin{solution}
\begin{enumerate}
\item $z_1 = - \frac{1}{26} - \frac{31 i}{26}$
\item $z_2 = 2 e^{- \frac{i \pi}{3}}$
\item $z_3 = 16 e^{\frac{3 i \pi}{4}}$
\item $z_4 = 32 e^{\frac{5 i \pi}{12}} = - 8 \sqrt{2} + 8 \sqrt{6} + i \left(8 \sqrt{2} + 8 \sqrt{6}\right) = 8.28 + 30.9 i$
\item $z_5 = \frac{1}{8} e^{- \frac{13 i \pi}{12}} = - \frac{\sqrt{6}}{32} - \frac{\sqrt{2}}{32} + i \left(- \frac{\sqrt{2}}{32} + \frac{\sqrt{6}}{32}\right) = -0.121 + 0.0323 i$
\end{enumerate}
\end{solution}
\begin{exercise}[subtitle={Sortie du congélateur}]
Marie a invité quelques amis pour le thé. Elle souhaite leur proposer ses macarons maison.
Elle les sort de son congélateur à $-19$~\degres C et les place dans une pièce à $15$~\degres C.
Au bout de 15 minutes, la température des macarons est de $-2$~\degres C.
\bigskip
\textbf{Premier modèle}
\medskip
On suppose que la vitesse de décongélation est constante : chaque minute la hausse de
température des macarons est la même.
Estimer dans ce cadre la température au bout de $30$~minutes, puis au bout de $45$~minutes.
Cette modélisation est-elle pertinente?
\bigskip
\textbf{Deuxième modèle}
\medskip
On suppose maintenant que la vitesse de décongélation est proportionnelle à la différence
de température entre les macarons et l'air ambiant (il s'agit de la loi de Newton).
On désigne par $\theta$ la température des macarons à l'instant $t$, et par $\theta'$ la vitesse de décongélation.
L'unité de temps est la minute et l'unité de température le degré Celsius.
\smallskip
On négligera la diminution de température de la pièce et on admettra donc qu'il existe un
nombre réel $a$ tel que, pour $t$ positif :
\[\theta'(t) = a [\theta(t) - 15]\quad (E)\]
\medskip
\begin{enumerate}
\item Vérifier que l'équation $(E)$ a pour solutions $\theta(t) = K e^{at} + 15$$K$ est un nombre réel.
Donner alors, en fonction de $a$, l'ensemble des solutions de $(E)$.
\end{enumerate}
On rappelle que la température des macarons à l'instant $t = 0$ est égale à $-19$~\degres C et que, au bout de $15$~min, elle est de $-2$~\degres C.
\begin{enumerate}
\setcounter{enumi}{1}
\item En utilisant la condition à $t=0$ démontrer que $K = -34$.
\item En utilisant la condition à $t=15$ démontrer que $a \approx -0.05$.
\item En déduire l'expression de la solution de l'équation différentielle puis étudier ses variations.
\item La température idéale de dégustation des macarons étant de $12$~\degres C, Marie estime que
celle-ci sera atteinte au bout de $30$~min. A-t-elle raison ? Justifier la réponse.
Sinon, combien de temps faudra-t-il attendre ?
\end{enumerate}
\end{exercise}
\end{document}
%%% Local Variables:
%%% mode: latex
%%% TeX-master: "master"
%%% End:

View File

@ -0,0 +1,136 @@
\documentclass[a4paper,10pt]{article}
\usepackage{myXsim}
% Title Page
\title{DS8 \hfill M'BAREK HASNAOUI Bilal}
\tribe{TST sti2d}
\date{\hfillÀ render pour le vendredi 9 avril à 10h au plus tard}
\xsimsetup{
solution/print = false
}
\begin{document}
\maketitle
\begin{exercise}[subtitle={Étude de fonction}]
On considère la fonction $f$ définie sur $\intOF{0}{+\infty}$ par $ f(x) = 5x^2 + - 40x + - 450\ln(x)$
\begin{enumerate}
\item Démontrer que la dérivée de $f$ est $f'(x) = \frac{10x^2 + - 40x + - 450}{x}$.
\item Étude du numérateur de $f'(x)$: $N(x) = 10x^2 - 40x - 450$
\begin{enumerate}
\item Démontrer que $x=9$ et $x=- 5$ sont deux racines de $N(x)$..
\item Proposer une forme factorisée de $N(x)$.
\item Proposer une forme factorisée de $f'(x)$.
\end{enumerate}
\item Étudier le signe de $f'$ et en déduire les variations de $f$.
\end{enumerate}
\end{exercise}
\begin{solution}
\begin{enumerate}
\item pas de correction disponible
\item
\begin{enumerate}
\item \[N(9) = 0\]
\[N(- 5) = 0\]
\item \[
N(x) = 10(x - 9)(x - - 5)
\]
\item
\[
f'(x) = \frac{10(x - 9)(x - - 5)}{x}
\]
\end{enumerate}
\item Pas de correction disponible
\end{enumerate}
\end{solution}
\begin{exercise}[subtitle={Complexes}]
\begin{enumerate}
\item Mettre le nombre complexe suivant sous forme algébrique $z_1 = \dfrac{2 + 4 i}{-5 + 7 i} $
\item Mettre le complexe suivante sous forme exponentielle $z_2 = 7 + 7 \sqrt{3} i$
\item Mettre le complexe suivante sous forme exponentielle $z_3 = 7 \sqrt{2} + 7 \sqrt{2} i$
\item Calculer le produit $z_4=z_2\times z_3$ donner le résultat sous forme exponentielle puis algébrique.
\item Calculer le quotient $z_5=\frac{z_2}{z_3}$ donner le résultat sous forme exponentielle puis algébrique.
\end{enumerate}
\end{exercise}
\begin{solution}
\begin{enumerate}
\item $z_1 = \frac{9}{37} - \frac{17 i}{37}$
\item $z_2 = 14 e^{\frac{i \pi}{3}}$
\item $z_3 = 14 e^{\frac{i \pi}{4}}$
\item $z_4 = 196 e^{\frac{7 i \pi}{12}} = - 49 \sqrt{6} + 49 \sqrt{2} + i \left(49 \sqrt{2} + 49 \sqrt{6}\right) = -50.7 + 189.0 i$
\item $z_5 = 1 e^{\frac{i \pi}{12}} = \frac{\sqrt{2}}{4} + \frac{\sqrt{6}}{4} + i \left(- \frac{\sqrt{2}}{4} + \frac{\sqrt{6}}{4}\right) = 0.966 + 0.259 i$
\end{enumerate}
\end{solution}
\begin{exercise}[subtitle={Sortie du congélateur}]
Marie a invité quelques amis pour le thé. Elle souhaite leur proposer ses macarons maison.
Elle les sort de son congélateur à $-15$~\degres C et les place dans une pièce à $25$~\degres C.
Au bout de 15 minutes, la température des macarons est de $0$~\degres C.
\bigskip
\textbf{Premier modèle}
\medskip
On suppose que la vitesse de décongélation est constante : chaque minute la hausse de
température des macarons est la même.
Estimer dans ce cadre la température au bout de $30$~minutes, puis au bout de $45$~minutes.
Cette modélisation est-elle pertinente?
\bigskip
\textbf{Deuxième modèle}
\medskip
On suppose maintenant que la vitesse de décongélation est proportionnelle à la différence
de température entre les macarons et l'air ambiant (il s'agit de la loi de Newton).
On désigne par $\theta$ la température des macarons à l'instant $t$, et par $\theta'$ la vitesse de décongélation.
L'unité de temps est la minute et l'unité de température le degré Celsius.
\smallskip
On négligera la diminution de température de la pièce et on admettra donc qu'il existe un
nombre réel $a$ tel que, pour $t$ positif :
\[\theta'(t) = a [\theta(t) - 25]\quad (E)\]
\medskip
\begin{enumerate}
\item Vérifier que l'équation $(E)$ a pour solutions $\theta(t) = K e^{at} + 25$$K$ est un nombre réel.
Donner alors, en fonction de $a$, l'ensemble des solutions de $(E)$.
\end{enumerate}
On rappelle que la température des macarons à l'instant $t = 0$ est égale à $-15$~\degres C et que, au bout de $15$~min, elle est de $0$~\degres C.
\begin{enumerate}
\setcounter{enumi}{1}
\item En utilisant la condition à $t=0$ démontrer que $K = -40$.
\item En utilisant la condition à $t=15$ démontrer que $a \approx -0.03$.
\item En déduire l'expression de la solution de l'équation différentielle puis étudier ses variations.
\item La température idéale de dégustation des macarons étant de $22$~\degres C, Marie estime que
celle-ci sera atteinte au bout de $30$~min. A-t-elle raison ? Justifier la réponse.
Sinon, combien de temps faudra-t-il attendre ?
\end{enumerate}
\end{exercise}
\end{document}
%%% Local Variables:
%%% mode: latex
%%% TeX-master: "master"
%%% End:

View File

@ -0,0 +1,136 @@
\documentclass[a4paper,10pt]{article}
\usepackage{myXsim}
% Title Page
\title{DS8 \hfill MERCIER Almandin}
\tribe{TST sti2d}
\date{\hfillÀ render pour le vendredi 9 avril à 10h au plus tard}
\xsimsetup{
solution/print = false
}
\begin{document}
\maketitle
\begin{exercise}[subtitle={Étude de fonction}]
On considère la fonction $f$ définie sur $\intOF{0}{+\infty}$ par $ f(x) = 2.5x^2 + - 50x + 120\ln(x)$
\begin{enumerate}
\item Démontrer que la dérivée de $f$ est $f'(x) = \frac{5x^2 + - 50x + 120}{x}$.
\item Étude du numérateur de $f'(x)$: $N(x) = 5x^2 - 50x + 120$
\begin{enumerate}
\item Démontrer que $x=6$ et $x=4$ sont deux racines de $N(x)$..
\item Proposer une forme factorisée de $N(x)$.
\item Proposer une forme factorisée de $f'(x)$.
\end{enumerate}
\item Étudier le signe de $f'$ et en déduire les variations de $f$.
\end{enumerate}
\end{exercise}
\begin{solution}
\begin{enumerate}
\item pas de correction disponible
\item
\begin{enumerate}
\item \[N(6) = 0\]
\[N(4) = 0\]
\item \[
N(x) = 5(x - 6)(x - 4)
\]
\item
\[
f'(x) = \frac{5(x - 6)(x - 4)}{x}
\]
\end{enumerate}
\item Pas de correction disponible
\end{enumerate}
\end{solution}
\begin{exercise}[subtitle={Complexes}]
\begin{enumerate}
\item Mettre le nombre complexe suivant sous forme algébrique $z_1 = \dfrac{10 + 10 i}{-2 + 3 i} $
\item Mettre le complexe suivante sous forme exponentielle $z_2 = - 7 \sqrt{2} - 7 \sqrt{2} i$
\item Mettre le complexe suivante sous forme exponentielle $z_3 = - 9 \sqrt{2} + 9 \sqrt{2} i$
\item Calculer le produit $z_4=z_2\times z_3$ donner le résultat sous forme exponentielle puis algébrique.
\item Calculer le quotient $z_5=\frac{z_2}{z_3}$ donner le résultat sous forme exponentielle puis algébrique.
\end{enumerate}
\end{exercise}
\begin{solution}
\begin{enumerate}
\item $z_1 = \frac{10}{13} - \frac{50 i}{13}$
\item $z_2 = 14 e^{- \frac{3 i \pi}{4}}$
\item $z_3 = 18 e^{\frac{3 i \pi}{4}}$
\item $z_4 = 252 e^{0} = 252 = 252.0$
\item $z_5 = \frac{7}{9} e^{- \frac{3 i \pi}{2}} = \frac{7 i}{9} = 0.778 i$
\end{enumerate}
\end{solution}
\begin{exercise}[subtitle={Sortie du congélateur}]
Marie a invité quelques amis pour le thé. Elle souhaite leur proposer ses macarons maison.
Elle les sort de son congélateur à $-16$~\degres C et les place dans une pièce à $22$~\degres C.
Au bout de 15 minutes, la température des macarons est de $3$~\degres C.
\bigskip
\textbf{Premier modèle}
\medskip
On suppose que la vitesse de décongélation est constante : chaque minute la hausse de
température des macarons est la même.
Estimer dans ce cadre la température au bout de $30$~minutes, puis au bout de $45$~minutes.
Cette modélisation est-elle pertinente?
\bigskip
\textbf{Deuxième modèle}
\medskip
On suppose maintenant que la vitesse de décongélation est proportionnelle à la différence
de température entre les macarons et l'air ambiant (il s'agit de la loi de Newton).
On désigne par $\theta$ la température des macarons à l'instant $t$, et par $\theta'$ la vitesse de décongélation.
L'unité de temps est la minute et l'unité de température le degré Celsius.
\smallskip
On négligera la diminution de température de la pièce et on admettra donc qu'il existe un
nombre réel $a$ tel que, pour $t$ positif :
\[\theta'(t) = a [\theta(t) - 22]\quad (E)\]
\medskip
\begin{enumerate}
\item Vérifier que l'équation $(E)$ a pour solutions $\theta(t) = K e^{at} + 22$$K$ est un nombre réel.
Donner alors, en fonction de $a$, l'ensemble des solutions de $(E)$.
\end{enumerate}
On rappelle que la température des macarons à l'instant $t = 0$ est égale à $-16$~\degres C et que, au bout de $15$~min, elle est de $3$~\degres C.
\begin{enumerate}
\setcounter{enumi}{1}
\item En utilisant la condition à $t=0$ démontrer que $K = -38$.
\item En utilisant la condition à $t=15$ démontrer que $a \approx -0.05$.
\item En déduire l'expression de la solution de l'équation différentielle puis étudier ses variations.
\item La température idéale de dégustation des macarons étant de $19$~\degres C, Marie estime que
celle-ci sera atteinte au bout de $30$~min. A-t-elle raison ? Justifier la réponse.
Sinon, combien de temps faudra-t-il attendre ?
\end{enumerate}
\end{exercise}
\end{document}
%%% Local Variables:
%%% mode: latex
%%% TeX-master: "master"
%%% End:

View File

@ -0,0 +1,136 @@
\documentclass[a4paper,10pt]{article}
\usepackage{myXsim}
% Title Page
\title{DS8 \hfill MOUFAQ Amine}
\tribe{TST sti2d}
\date{\hfillÀ render pour le vendredi 9 avril à 10h au plus tard}
\xsimsetup{
solution/print = false
}
\begin{document}
\maketitle
\begin{exercise}[subtitle={Étude de fonction}]
On considère la fonction $f$ définie sur $\intOF{0}{+\infty}$ par $ f(x) = 5x^2 + 0x + - 810\ln(x)$
\begin{enumerate}
\item Démontrer que la dérivée de $f$ est $f'(x) = \frac{10x^2 + 0x + - 810}{x}$.
\item Étude du numérateur de $f'(x)$: $N(x) = 10x^2 - 810$
\begin{enumerate}
\item Démontrer que $x=- 9$ et $x=9$ sont deux racines de $N(x)$..
\item Proposer une forme factorisée de $N(x)$.
\item Proposer une forme factorisée de $f'(x)$.
\end{enumerate}
\item Étudier le signe de $f'$ et en déduire les variations de $f$.
\end{enumerate}
\end{exercise}
\begin{solution}
\begin{enumerate}
\item pas de correction disponible
\item
\begin{enumerate}
\item \[N(- 9) = 0\]
\[N(9) = 0\]
\item \[
N(x) = 10(x - - 9)(x - 9)
\]
\item
\[
f'(x) = \frac{10(x - - 9)(x - 9)}{x}
\]
\end{enumerate}
\item Pas de correction disponible
\end{enumerate}
\end{solution}
\begin{exercise}[subtitle={Complexes}]
\begin{enumerate}
\item Mettre le nombre complexe suivant sous forme algébrique $z_1 = \dfrac{8 + 4 i}{-3 + 5 i} $
\item Mettre le complexe suivante sous forme exponentielle $z_2 = - 2 \sqrt{2} + 2 \sqrt{2} i$
\item Mettre le complexe suivante sous forme exponentielle $z_3 = 6 + 6 \sqrt{3} i$
\item Calculer le produit $z_4=z_2\times z_3$ donner le résultat sous forme exponentielle puis algébrique.
\item Calculer le quotient $z_5=\frac{z_2}{z_3}$ donner le résultat sous forme exponentielle puis algébrique.
\end{enumerate}
\end{exercise}
\begin{solution}
\begin{enumerate}
\item $z_1 = - \frac{2}{17} - \frac{26 i}{17}$
\item $z_2 = 4 e^{\frac{3 i \pi}{4}}$
\item $z_3 = 12 e^{\frac{i \pi}{3}}$
\item $z_4 = 48 e^{\frac{13 i \pi}{12}} = - 12 \sqrt{6} - 12 \sqrt{2} + i \left(- 12 \sqrt{6} + 12 \sqrt{2}\right) = -46.4 - 12.4 i$
\item $z_5 = \frac{1}{3} e^{\frac{5 i \pi}{12}} = - \frac{\sqrt{2}}{12} + \frac{\sqrt{6}}{12} + i \left(\frac{\sqrt{2}}{12} + \frac{\sqrt{6}}{12}\right) = 0.0863 + 0.322 i$
\end{enumerate}
\end{solution}
\begin{exercise}[subtitle={Sortie du congélateur}]
Marie a invité quelques amis pour le thé. Elle souhaite leur proposer ses macarons maison.
Elle les sort de son congélateur à $-15$~\degres C et les place dans une pièce à $20$~\degres C.
Au bout de 15 minutes, la température des macarons est de $-3$~\degres C.
\bigskip
\textbf{Premier modèle}
\medskip
On suppose que la vitesse de décongélation est constante : chaque minute la hausse de
température des macarons est la même.
Estimer dans ce cadre la température au bout de $30$~minutes, puis au bout de $45$~minutes.
Cette modélisation est-elle pertinente?
\bigskip
\textbf{Deuxième modèle}
\medskip
On suppose maintenant que la vitesse de décongélation est proportionnelle à la différence
de température entre les macarons et l'air ambiant (il s'agit de la loi de Newton).
On désigne par $\theta$ la température des macarons à l'instant $t$, et par $\theta'$ la vitesse de décongélation.
L'unité de temps est la minute et l'unité de température le degré Celsius.
\smallskip
On négligera la diminution de température de la pièce et on admettra donc qu'il existe un
nombre réel $a$ tel que, pour $t$ positif :
\[\theta'(t) = a [\theta(t) - 20]\quad (E)\]
\medskip
\begin{enumerate}
\item Vérifier que l'équation $(E)$ a pour solutions $\theta(t) = K e^{at} + 20$$K$ est un nombre réel.
Donner alors, en fonction de $a$, l'ensemble des solutions de $(E)$.
\end{enumerate}
On rappelle que la température des macarons à l'instant $t = 0$ est égale à $-15$~\degres C et que, au bout de $15$~min, elle est de $-3$~\degres C.
\begin{enumerate}
\setcounter{enumi}{1}
\item En utilisant la condition à $t=0$ démontrer que $K = -35$.
\item En utilisant la condition à $t=15$ démontrer que $a \approx -0.03$.
\item En déduire l'expression de la solution de l'équation différentielle puis étudier ses variations.
\item La température idéale de dégustation des macarons étant de $17$~\degres C, Marie estime que
celle-ci sera atteinte au bout de $30$~min. A-t-elle raison ? Justifier la réponse.
Sinon, combien de temps faudra-t-il attendre ?
\end{enumerate}
\end{exercise}
\end{document}
%%% Local Variables:
%%% mode: latex
%%% TeX-master: "master"
%%% End:

View File

@ -0,0 +1,136 @@
\documentclass[a4paper,10pt]{article}
\usepackage{myXsim}
% Title Page
\title{DS8 \hfill NARDINI Kakary}
\tribe{TST sti2d}
\date{\hfillÀ render pour le vendredi 9 avril à 10h au plus tard}
\xsimsetup{
solution/print = false
}
\begin{document}
\maketitle
\begin{exercise}[subtitle={Étude de fonction}]
On considère la fonction $f$ définie sur $\intOF{0}{+\infty}$ par $ f(x) = 3x^2 + - 18x + - 108\ln(x)$
\begin{enumerate}
\item Démontrer que la dérivée de $f$ est $f'(x) = \frac{6x^2 + - 18x + - 108}{x}$.
\item Étude du numérateur de $f'(x)$: $N(x) = 6x^2 - 18x - 108$
\begin{enumerate}
\item Démontrer que $x=6$ et $x=- 3$ sont deux racines de $N(x)$..
\item Proposer une forme factorisée de $N(x)$.
\item Proposer une forme factorisée de $f'(x)$.
\end{enumerate}
\item Étudier le signe de $f'$ et en déduire les variations de $f$.
\end{enumerate}
\end{exercise}
\begin{solution}
\begin{enumerate}
\item pas de correction disponible
\item
\begin{enumerate}
\item \[N(6) = 0\]
\[N(- 3) = 0\]
\item \[
N(x) = 6(x - 6)(x - - 3)
\]
\item
\[
f'(x) = \frac{6(x - 6)(x - - 3)}{x}
\]
\end{enumerate}
\item Pas de correction disponible
\end{enumerate}
\end{solution}
\begin{exercise}[subtitle={Complexes}]
\begin{enumerate}
\item Mettre le nombre complexe suivant sous forme algébrique $z_1 = \dfrac{9 + 6 i}{-3 + 6 i} $
\item Mettre le complexe suivante sous forme exponentielle $z_2 = -4 - 4 \sqrt{3} i$
\item Mettre le complexe suivante sous forme exponentielle $z_3 = 9 \sqrt{2} + 9 \sqrt{2} i$
\item Calculer le produit $z_4=z_2\times z_3$ donner le résultat sous forme exponentielle puis algébrique.
\item Calculer le quotient $z_5=\frac{z_2}{z_3}$ donner le résultat sous forme exponentielle puis algébrique.
\end{enumerate}
\end{exercise}
\begin{solution}
\begin{enumerate}
\item $z_1 = \frac{1}{5} - \frac{8 i}{5}$
\item $z_2 = 8 e^{- \frac{2 i \pi}{3}}$
\item $z_3 = 18 e^{\frac{i \pi}{4}}$
\item $z_4 = 144 e^{- \frac{5 i \pi}{12}} = - 36 \sqrt{2} + 36 \sqrt{6} + i \left(- 36 \sqrt{6} - 36 \sqrt{2}\right) = 37.3 - 139.0 i$
\item $z_5 = \frac{4}{9} e^{- \frac{11 i \pi}{12}} = - \frac{\sqrt{6}}{9} - \frac{\sqrt{2}}{9} + i \left(- \frac{\sqrt{6}}{9} + \frac{\sqrt{2}}{9}\right) = -0.429 - 0.115 i$
\end{enumerate}
\end{solution}
\begin{exercise}[subtitle={Sortie du congélateur}]
Marie a invité quelques amis pour le thé. Elle souhaite leur proposer ses macarons maison.
Elle les sort de son congélateur à $-18$~\degres C et les place dans une pièce à $17$~\degres C.
Au bout de 15 minutes, la température des macarons est de $2$~\degres C.
\bigskip
\textbf{Premier modèle}
\medskip
On suppose que la vitesse de décongélation est constante : chaque minute la hausse de
température des macarons est la même.
Estimer dans ce cadre la température au bout de $30$~minutes, puis au bout de $45$~minutes.
Cette modélisation est-elle pertinente?
\bigskip
\textbf{Deuxième modèle}
\medskip
On suppose maintenant que la vitesse de décongélation est proportionnelle à la différence
de température entre les macarons et l'air ambiant (il s'agit de la loi de Newton).
On désigne par $\theta$ la température des macarons à l'instant $t$, et par $\theta'$ la vitesse de décongélation.
L'unité de temps est la minute et l'unité de température le degré Celsius.
\smallskip
On négligera la diminution de température de la pièce et on admettra donc qu'il existe un
nombre réel $a$ tel que, pour $t$ positif :
\[\theta'(t) = a [\theta(t) - 17]\quad (E)\]
\medskip
\begin{enumerate}
\item Vérifier que l'équation $(E)$ a pour solutions $\theta(t) = K e^{at} + 17$$K$ est un nombre réel.
Donner alors, en fonction de $a$, l'ensemble des solutions de $(E)$.
\end{enumerate}
On rappelle que la température des macarons à l'instant $t = 0$ est égale à $-18$~\degres C et que, au bout de $15$~min, elle est de $2$~\degres C.
\begin{enumerate}
\setcounter{enumi}{1}
\item En utilisant la condition à $t=0$ démontrer que $K = -35$.
\item En utilisant la condition à $t=15$ démontrer que $a \approx -0.06$.
\item En déduire l'expression de la solution de l'équation différentielle puis étudier ses variations.
\item La température idéale de dégustation des macarons étant de $14$~\degres C, Marie estime que
celle-ci sera atteinte au bout de $30$~min. A-t-elle raison ? Justifier la réponse.
Sinon, combien de temps faudra-t-il attendre ?
\end{enumerate}
\end{exercise}
\end{document}
%%% Local Variables:
%%% mode: latex
%%% TeX-master: "master"
%%% End:

View File

@ -0,0 +1,136 @@
\documentclass[a4paper,10pt]{article}
\usepackage{myXsim}
% Title Page
\title{DS8 \hfill ONAL Yakub}
\tribe{TST sti2d}
\date{\hfillÀ render pour le vendredi 9 avril à 10h au plus tard}
\xsimsetup{
solution/print = false
}
\begin{document}
\maketitle
\begin{exercise}[subtitle={Étude de fonction}]
On considère la fonction $f$ définie sur $\intOF{0}{+\infty}$ par $ f(x) = 5x^2 + - 40x + - 450\ln(x)$
\begin{enumerate}
\item Démontrer que la dérivée de $f$ est $f'(x) = \frac{10x^2 + - 40x + - 450}{x}$.
\item Étude du numérateur de $f'(x)$: $N(x) = 10x^2 - 40x - 450$
\begin{enumerate}
\item Démontrer que $x=9$ et $x=- 5$ sont deux racines de $N(x)$..
\item Proposer une forme factorisée de $N(x)$.
\item Proposer une forme factorisée de $f'(x)$.
\end{enumerate}
\item Étudier le signe de $f'$ et en déduire les variations de $f$.
\end{enumerate}
\end{exercise}
\begin{solution}
\begin{enumerate}
\item pas de correction disponible
\item
\begin{enumerate}
\item \[N(9) = 0\]
\[N(- 5) = 0\]
\item \[
N(x) = 10(x - 9)(x - - 5)
\]
\item
\[
f'(x) = \frac{10(x - 9)(x - - 5)}{x}
\]
\end{enumerate}
\item Pas de correction disponible
\end{enumerate}
\end{solution}
\begin{exercise}[subtitle={Complexes}]
\begin{enumerate}
\item Mettre le nombre complexe suivant sous forme algébrique $z_1 = \dfrac{5 + 2 i}{-10 + 2 i} $
\item Mettre le complexe suivante sous forme exponentielle $z_2 = 6 - 6 \sqrt{3} i$
\item Mettre le complexe suivante sous forme exponentielle $z_3 = 10 \sqrt{3} + 10 i$
\item Calculer le produit $z_4=z_2\times z_3$ donner le résultat sous forme exponentielle puis algébrique.
\item Calculer le quotient $z_5=\frac{z_2}{z_3}$ donner le résultat sous forme exponentielle puis algébrique.
\end{enumerate}
\end{exercise}
\begin{solution}
\begin{enumerate}
\item $z_1 = - \frac{23}{52} - \frac{15 i}{52}$
\item $z_2 = 12 e^{- \frac{i \pi}{3}}$
\item $z_3 = 20 e^{\frac{i \pi}{6}}$
\item $z_4 = 240 e^{- \frac{i \pi}{6}} = 120 \sqrt{3} - 120 i = 208.0 - 120.0 i$
\item $z_5 = \frac{3}{5} e^{- \frac{i \pi}{2}} = - \frac{3 i}{5} = - 0.6 i$
\end{enumerate}
\end{solution}
\begin{exercise}[subtitle={Sortie du congélateur}]
Marie a invité quelques amis pour le thé. Elle souhaite leur proposer ses macarons maison.
Elle les sort de son congélateur à $-18$~\degres C et les place dans une pièce à $19$~\degres C.
Au bout de 15 minutes, la température des macarons est de $-2$~\degres C.
\bigskip
\textbf{Premier modèle}
\medskip
On suppose que la vitesse de décongélation est constante : chaque minute la hausse de
température des macarons est la même.
Estimer dans ce cadre la température au bout de $30$~minutes, puis au bout de $45$~minutes.
Cette modélisation est-elle pertinente?
\bigskip
\textbf{Deuxième modèle}
\medskip
On suppose maintenant que la vitesse de décongélation est proportionnelle à la différence
de température entre les macarons et l'air ambiant (il s'agit de la loi de Newton).
On désigne par $\theta$ la température des macarons à l'instant $t$, et par $\theta'$ la vitesse de décongélation.
L'unité de temps est la minute et l'unité de température le degré Celsius.
\smallskip
On négligera la diminution de température de la pièce et on admettra donc qu'il existe un
nombre réel $a$ tel que, pour $t$ positif :
\[\theta'(t) = a [\theta(t) - 19]\quad (E)\]
\medskip
\begin{enumerate}
\item Vérifier que l'équation $(E)$ a pour solutions $\theta(t) = K e^{at} + 19$$K$ est un nombre réel.
Donner alors, en fonction de $a$, l'ensemble des solutions de $(E)$.
\end{enumerate}
On rappelle que la température des macarons à l'instant $t = 0$ est égale à $-18$~\degres C et que, au bout de $15$~min, elle est de $-2$~\degres C.
\begin{enumerate}
\setcounter{enumi}{1}
\item En utilisant la condition à $t=0$ démontrer que $K = -37$.
\item En utilisant la condition à $t=15$ démontrer que $a \approx -0.04$.
\item En déduire l'expression de la solution de l'équation différentielle puis étudier ses variations.
\item La température idéale de dégustation des macarons étant de $16$~\degres C, Marie estime que
celle-ci sera atteinte au bout de $30$~min. A-t-elle raison ? Justifier la réponse.
Sinon, combien de temps faudra-t-il attendre ?
\end{enumerate}
\end{exercise}
\end{document}
%%% Local Variables:
%%% mode: latex
%%% TeX-master: "master"
%%% End:

View File

@ -0,0 +1,136 @@
\documentclass[a4paper,10pt]{article}
\usepackage{myXsim}
% Title Page
\title{DS8 \hfill RADOUAA Saleh}
\tribe{TST sti2d}
\date{\hfillÀ render pour le vendredi 9 avril à 10h au plus tard}
\xsimsetup{
solution/print = false
}
\begin{document}
\maketitle
\begin{exercise}[subtitle={Étude de fonction}]
On considère la fonction $f$ définie sur $\intOF{0}{+\infty}$ par $ f(x) = 3x^2 + - 54x + 108\ln(x)$
\begin{enumerate}
\item Démontrer que la dérivée de $f$ est $f'(x) = \frac{6x^2 + - 54x + 108}{x}$.
\item Étude du numérateur de $f'(x)$: $N(x) = 6x^2 - 54x + 108$
\begin{enumerate}
\item Démontrer que $x=3$ et $x=6$ sont deux racines de $N(x)$..
\item Proposer une forme factorisée de $N(x)$.
\item Proposer une forme factorisée de $f'(x)$.
\end{enumerate}
\item Étudier le signe de $f'$ et en déduire les variations de $f$.
\end{enumerate}
\end{exercise}
\begin{solution}
\begin{enumerate}
\item pas de correction disponible
\item
\begin{enumerate}
\item \[N(3) = 0\]
\[N(6) = 0\]
\item \[
N(x) = 6(x - 3)(x - 6)
\]
\item
\[
f'(x) = \frac{6(x - 3)(x - 6)}{x}
\]
\end{enumerate}
\item Pas de correction disponible
\end{enumerate}
\end{solution}
\begin{exercise}[subtitle={Complexes}]
\begin{enumerate}
\item Mettre le nombre complexe suivant sous forme algébrique $z_1 = \dfrac{7 + 9 i}{-3 + 7 i} $
\item Mettre le complexe suivante sous forme exponentielle $z_2 = -6 + 6 \sqrt{3} i$
\item Mettre le complexe suivante sous forme exponentielle $z_3 = 10 + 10 \sqrt{3} i$
\item Calculer le produit $z_4=z_2\times z_3$ donner le résultat sous forme exponentielle puis algébrique.
\item Calculer le quotient $z_5=\frac{z_2}{z_3}$ donner le résultat sous forme exponentielle puis algébrique.
\end{enumerate}
\end{exercise}
\begin{solution}
\begin{enumerate}
\item $z_1 = \frac{21}{29} - \frac{38 i}{29}$
\item $z_2 = 12 e^{\frac{2 i \pi}{3}}$
\item $z_3 = 20 e^{\frac{i \pi}{3}}$
\item $z_4 = 240 e^{i \pi} = -240 = -240.0$
\item $z_5 = \frac{3}{5} e^{\frac{i \pi}{3}} = \frac{3}{10} + \frac{3 \sqrt{3} i}{10} = 0.3 + 0.52 i$
\end{enumerate}
\end{solution}
\begin{exercise}[subtitle={Sortie du congélateur}]
Marie a invité quelques amis pour le thé. Elle souhaite leur proposer ses macarons maison.
Elle les sort de son congélateur à $-20$~\degres C et les place dans une pièce à $16$~\degres C.
Au bout de 15 minutes, la température des macarons est de $4$~\degres C.
\bigskip
\textbf{Premier modèle}
\medskip
On suppose que la vitesse de décongélation est constante : chaque minute la hausse de
température des macarons est la même.
Estimer dans ce cadre la température au bout de $30$~minutes, puis au bout de $45$~minutes.
Cette modélisation est-elle pertinente?
\bigskip
\textbf{Deuxième modèle}
\medskip
On suppose maintenant que la vitesse de décongélation est proportionnelle à la différence
de température entre les macarons et l'air ambiant (il s'agit de la loi de Newton).
On désigne par $\theta$ la température des macarons à l'instant $t$, et par $\theta'$ la vitesse de décongélation.
L'unité de temps est la minute et l'unité de température le degré Celsius.
\smallskip
On négligera la diminution de température de la pièce et on admettra donc qu'il existe un
nombre réel $a$ tel que, pour $t$ positif :
\[\theta'(t) = a [\theta(t) - 16]\quad (E)\]
\medskip
\begin{enumerate}
\item Vérifier que l'équation $(E)$ a pour solutions $\theta(t) = K e^{at} + 16$$K$ est un nombre réel.
Donner alors, en fonction de $a$, l'ensemble des solutions de $(E)$.
\end{enumerate}
On rappelle que la température des macarons à l'instant $t = 0$ est égale à $-20$~\degres C et que, au bout de $15$~min, elle est de $4$~\degres C.
\begin{enumerate}
\setcounter{enumi}{1}
\item En utilisant la condition à $t=0$ démontrer que $K = -36$.
\item En utilisant la condition à $t=15$ démontrer que $a \approx -0.07$.
\item En déduire l'expression de la solution de l'équation différentielle puis étudier ses variations.
\item La température idéale de dégustation des macarons étant de $13$~\degres C, Marie estime que
celle-ci sera atteinte au bout de $30$~min. A-t-elle raison ? Justifier la réponse.
Sinon, combien de temps faudra-t-il attendre ?
\end{enumerate}
\end{exercise}
\end{document}
%%% Local Variables:
%%% mode: latex
%%% TeX-master: "master"
%%% End:

View File

@ -0,0 +1,136 @@
\documentclass[a4paper,10pt]{article}
\usepackage{myXsim}
% Title Page
\title{DS8 \hfill TAVERNIER Joanny}
\tribe{TST sti2d}
\date{\hfillÀ render pour le vendredi 9 avril à 10h au plus tard}
\xsimsetup{
solution/print = false
}
\begin{document}
\maketitle
\begin{exercise}[subtitle={Étude de fonction}]
On considère la fonction $f$ définie sur $\intOF{0}{+\infty}$ par $ f(x) = 4.5x^2 + - 144x + 567\ln(x)$
\begin{enumerate}
\item Démontrer que la dérivée de $f$ est $f'(x) = \frac{9x^2 + - 144x + 567}{x}$.
\item Étude du numérateur de $f'(x)$: $N(x) = 9x^2 - 144x + 567$
\begin{enumerate}
\item Démontrer que $x=9$ et $x=7$ sont deux racines de $N(x)$..
\item Proposer une forme factorisée de $N(x)$.
\item Proposer une forme factorisée de $f'(x)$.
\end{enumerate}
\item Étudier le signe de $f'$ et en déduire les variations de $f$.
\end{enumerate}
\end{exercise}
\begin{solution}
\begin{enumerate}
\item pas de correction disponible
\item
\begin{enumerate}
\item \[N(9) = 0\]
\[N(7) = 0\]
\item \[
N(x) = 9(x - 9)(x - 7)
\]
\item
\[
f'(x) = \frac{9(x - 9)(x - 7)}{x}
\]
\end{enumerate}
\item Pas de correction disponible
\end{enumerate}
\end{solution}
\begin{exercise}[subtitle={Complexes}]
\begin{enumerate}
\item Mettre le nombre complexe suivant sous forme algébrique $z_1 = \dfrac{10 + 6 i}{-4 + 5 i} $
\item Mettre le complexe suivante sous forme exponentielle $z_2 = - 8 \sqrt{3} + 8 i$
\item Mettre le complexe suivante sous forme exponentielle $z_3 = 5 - 5 \sqrt{3} i$
\item Calculer le produit $z_4=z_2\times z_3$ donner le résultat sous forme exponentielle puis algébrique.
\item Calculer le quotient $z_5=\frac{z_2}{z_3}$ donner le résultat sous forme exponentielle puis algébrique.
\end{enumerate}
\end{exercise}
\begin{solution}
\begin{enumerate}
\item $z_1 = - \frac{10}{41} - \frac{74 i}{41}$
\item $z_2 = 16 e^{\frac{5 i \pi}{6}}$
\item $z_3 = 10 e^{- \frac{i \pi}{3}}$
\item $z_4 = 160 e^{\frac{i \pi}{2}} = 160 i = 160.0 i$
\item $z_5 = \frac{8}{5} e^{\frac{7 i \pi}{6}} = - \frac{4 \sqrt{3}}{5} - \frac{4 i}{5} = -1.39 - 0.8 i$
\end{enumerate}
\end{solution}
\begin{exercise}[subtitle={Sortie du congélateur}]
Marie a invité quelques amis pour le thé. Elle souhaite leur proposer ses macarons maison.
Elle les sort de son congélateur à $-19$~\degres C et les place dans une pièce à $21$~\degres C.
Au bout de 15 minutes, la température des macarons est de $-4$~\degres C.
\bigskip
\textbf{Premier modèle}
\medskip
On suppose que la vitesse de décongélation est constante : chaque minute la hausse de
température des macarons est la même.
Estimer dans ce cadre la température au bout de $30$~minutes, puis au bout de $45$~minutes.
Cette modélisation est-elle pertinente?
\bigskip
\textbf{Deuxième modèle}
\medskip
On suppose maintenant que la vitesse de décongélation est proportionnelle à la différence
de température entre les macarons et l'air ambiant (il s'agit de la loi de Newton).
On désigne par $\theta$ la température des macarons à l'instant $t$, et par $\theta'$ la vitesse de décongélation.
L'unité de temps est la minute et l'unité de température le degré Celsius.
\smallskip
On négligera la diminution de température de la pièce et on admettra donc qu'il existe un
nombre réel $a$ tel que, pour $t$ positif :
\[\theta'(t) = a [\theta(t) - 21]\quad (E)\]
\medskip
\begin{enumerate}
\item Vérifier que l'équation $(E)$ a pour solutions $\theta(t) = K e^{at} + 21$$K$ est un nombre réel.
Donner alors, en fonction de $a$, l'ensemble des solutions de $(E)$.
\end{enumerate}
On rappelle que la température des macarons à l'instant $t = 0$ est égale à $-19$~\degres C et que, au bout de $15$~min, elle est de $-4$~\degres C.
\begin{enumerate}
\setcounter{enumi}{1}
\item En utilisant la condition à $t=0$ démontrer que $K = -40$.
\item En utilisant la condition à $t=15$ démontrer que $a \approx -0.03$.
\item En déduire l'expression de la solution de l'équation différentielle puis étudier ses variations.
\item La température idéale de dégustation des macarons étant de $18$~\degres C, Marie estime que
celle-ci sera atteinte au bout de $30$~min. A-t-elle raison ? Justifier la réponse.
Sinon, combien de temps faudra-t-il attendre ?
\end{enumerate}
\end{exercise}
\end{document}
%%% Local Variables:
%%% mode: latex
%%% TeX-master: "master"
%%% End:

View File

@ -0,0 +1,136 @@
\documentclass[a4paper,10pt]{article}
\usepackage{myXsim}
% Title Page
\title{DS8 \hfill ZAHORE Zahiri}
\tribe{TST sti2d}
\date{\hfillÀ render pour le vendredi 9 avril à 10h au plus tard}
\xsimsetup{
solution/print = false
}
\begin{document}
\maketitle
\begin{exercise}[subtitle={Étude de fonction}]
On considère la fonction $f$ définie sur $\intOF{0}{+\infty}$ par $ f(x) = 4.5x^2 + - 54x + - 63\ln(x)$
\begin{enumerate}
\item Démontrer que la dérivée de $f$ est $f'(x) = \frac{9x^2 + - 54x + - 63}{x}$.
\item Étude du numérateur de $f'(x)$: $N(x) = 9x^2 - 54x - 63$
\begin{enumerate}
\item Démontrer que $x=7$ et $x=- 1$ sont deux racines de $N(x)$..
\item Proposer une forme factorisée de $N(x)$.
\item Proposer une forme factorisée de $f'(x)$.
\end{enumerate}
\item Étudier le signe de $f'$ et en déduire les variations de $f$.
\end{enumerate}
\end{exercise}
\begin{solution}
\begin{enumerate}
\item pas de correction disponible
\item
\begin{enumerate}
\item \[N(7) = 0\]
\[N(- 1) = 0\]
\item \[
N(x) = 9(x - 7)(x - - 1)
\]
\item
\[
f'(x) = \frac{9(x - 7)(x - - 1)}{x}
\]
\end{enumerate}
\item Pas de correction disponible
\end{enumerate}
\end{solution}
\begin{exercise}[subtitle={Complexes}]
\begin{enumerate}
\item Mettre le nombre complexe suivant sous forme algébrique $z_1 = \dfrac{7 + 4 i}{-4 + 8 i} $
\item Mettre le complexe suivante sous forme exponentielle $z_2 = 4 \sqrt{2} + 4 \sqrt{2} i$
\item Mettre le complexe suivante sous forme exponentielle $z_3 = - 4 \sqrt{2} - 4 \sqrt{2} i$
\item Calculer le produit $z_4=z_2\times z_3$ donner le résultat sous forme exponentielle puis algébrique.
\item Calculer le quotient $z_5=\frac{z_2}{z_3}$ donner le résultat sous forme exponentielle puis algébrique.
\end{enumerate}
\end{exercise}
\begin{solution}
\begin{enumerate}
\item $z_1 = \frac{1}{20} - \frac{9 i}{10}$
\item $z_2 = 8 e^{\frac{i \pi}{4}}$
\item $z_3 = 8 e^{- \frac{3 i \pi}{4}}$
\item $z_4 = 64 e^{- \frac{i \pi}{2}} = - 64 i = - 64.0 i$
\item $z_5 = 1 e^{i \pi} = -1 = -1.0$
\end{enumerate}
\end{solution}
\begin{exercise}[subtitle={Sortie du congélateur}]
Marie a invité quelques amis pour le thé. Elle souhaite leur proposer ses macarons maison.
Elle les sort de son congélateur à $-18$~\degres C et les place dans une pièce à $21$~\degres C.
Au bout de 15 minutes, la température des macarons est de $4$~\degres C.
\bigskip
\textbf{Premier modèle}
\medskip
On suppose que la vitesse de décongélation est constante : chaque minute la hausse de
température des macarons est la même.
Estimer dans ce cadre la température au bout de $30$~minutes, puis au bout de $45$~minutes.
Cette modélisation est-elle pertinente?
\bigskip
\textbf{Deuxième modèle}
\medskip
On suppose maintenant que la vitesse de décongélation est proportionnelle à la différence
de température entre les macarons et l'air ambiant (il s'agit de la loi de Newton).
On désigne par $\theta$ la température des macarons à l'instant $t$, et par $\theta'$ la vitesse de décongélation.
L'unité de temps est la minute et l'unité de température le degré Celsius.
\smallskip
On négligera la diminution de température de la pièce et on admettra donc qu'il existe un
nombre réel $a$ tel que, pour $t$ positif :
\[\theta'(t) = a [\theta(t) - 21]\quad (E)\]
\medskip
\begin{enumerate}
\item Vérifier que l'équation $(E)$ a pour solutions $\theta(t) = K e^{at} + 21$$K$ est un nombre réel.
Donner alors, en fonction de $a$, l'ensemble des solutions de $(E)$.
\end{enumerate}
On rappelle que la température des macarons à l'instant $t = 0$ est égale à $-18$~\degres C et que, au bout de $15$~min, elle est de $4$~\degres C.
\begin{enumerate}
\setcounter{enumi}{1}
\item En utilisant la condition à $t=0$ démontrer que $K = -39$.
\item En utilisant la condition à $t=15$ démontrer que $a \approx -0.06$.
\item En déduire l'expression de la solution de l'équation différentielle puis étudier ses variations.
\item La température idéale de dégustation des macarons étant de $18$~\degres C, Marie estime que
celle-ci sera atteinte au bout de $30$~min. A-t-elle raison ? Justifier la réponse.
Sinon, combien de temps faudra-t-il attendre ?
\end{enumerate}
\end{exercise}
\end{document}
%%% Local Variables:
%%% mode: latex
%%% TeX-master: "master"
%%% End:

Binary file not shown.

View File

@ -0,0 +1,136 @@
\documentclass[a4paper,10pt]{article}
\usepackage{myXsim}
% Title Page
\title{DS8 \hfill BAHBAH Zakaria}
\tribe{TST sti2d}
\date{\hfillÀ render pour le vendredi 9 avril à 10h au plus tard}
\xsimsetup{
solution/print = true
}
\begin{document}
\maketitle
\begin{exercise}[subtitle={Étude de fonction}]
On considère la fonction $f$ définie sur $\intOF{0}{+\infty}$ par $ f(x) = 4x^2 + 72x + 160\ln(x)$
\begin{enumerate}
\item Démontrer que la dérivée de $f$ est $f'(x) = \frac{8x^2 + 72x + 160}{x}$.
\item Étude du numérateur de $f'(x)$: $N(x) = 8x^2 + 72x + 160$
\begin{enumerate}
\item Démontrer que $x=- 4$ et $x=- 5$ sont deux racines de $N(x)$..
\item Proposer une forme factorisée de $N(x)$.
\item Proposer une forme factorisée de $f'(x)$.
\end{enumerate}
\item Étudier le signe de $f'$ et en déduire les variations de $f$.
\end{enumerate}
\end{exercise}
\begin{solution}
\begin{enumerate}
\item pas de correction disponible
\item
\begin{enumerate}
\item \[N(- 4) = 0\]
\[N(- 5) = 0\]
\item \[
N(x) = 8(x - - 4)(x - - 5)
\]
\item
\[
f'(x) = \frac{8(x - - 4)(x - - 5)}{x}
\]
\end{enumerate}
\item Pas de correction disponible
\end{enumerate}
\end{solution}
\begin{exercise}[subtitle={Complexes}]
\begin{enumerate}
\item Mettre le nombre complexe suivant sous forme algébrique $z_1 = \dfrac{2 + 8 i}{-9 + 3 i} $
\item Mettre le complexe suivante sous forme exponentielle $z_2 = 10 \sqrt{3} - 10 i$
\item Mettre le complexe suivante sous forme exponentielle $z_3 = 7 + 7 \sqrt{3} i$
\item Calculer le produit $z_4=z_2\times z_3$ donner le résultat sous forme exponentielle puis algébrique.
\item Calculer le quotient $z_5=\frac{z_2}{z_3}$ donner le résultat sous forme exponentielle puis algébrique.
\end{enumerate}
\end{exercise}
\begin{solution}
\begin{enumerate}
\item $z_1 = \frac{1}{15} - \frac{13 i}{15}$
\item $z_2 = 20 e^{- \frac{i \pi}{6}}$
\item $z_3 = 14 e^{\frac{i \pi}{3}}$
\item $z_4 = 280 e^{\frac{i \pi}{6}} = 140 \sqrt{3} + 140 i = 243.0 + 140.0 i$
\item $z_5 = \frac{10}{7} e^{- \frac{i \pi}{2}} = - \frac{10 i}{7} = - 1.43 i$
\end{enumerate}
\end{solution}
\begin{exercise}[subtitle={Sortie du congélateur}]
Marie a invité quelques amis pour le thé. Elle souhaite leur proposer ses macarons maison.
Elle les sort de son congélateur à $-17$~\degres C et les place dans une pièce à $16$~\degres C.
Au bout de 15 minutes, la température des macarons est de $-3$~\degres C.
\bigskip
\textbf{Premier modèle}
\medskip
On suppose que la vitesse de décongélation est constante : chaque minute la hausse de
température des macarons est la même.
Estimer dans ce cadre la température au bout de $30$~minutes, puis au bout de $45$~minutes.
Cette modélisation est-elle pertinente?
\bigskip
\textbf{Deuxième modèle}
\medskip
On suppose maintenant que la vitesse de décongélation est proportionnelle à la différence
de température entre les macarons et l'air ambiant (il s'agit de la loi de Newton).
On désigne par $\theta$ la température des macarons à l'instant $t$, et par $\theta'$ la vitesse de décongélation.
L'unité de temps est la minute et l'unité de température le degré Celsius.
\smallskip
On négligera la diminution de température de la pièce et on admettra donc qu'il existe un
nombre réel $a$ tel que, pour $t$ positif :
\[\theta'(t) = a [\theta(t) - 16]\quad (E)\]
\medskip
\begin{enumerate}
\item Vérifier que l'équation $(E)$ a pour solutions $\theta(t) = K e^{at} + 16$$K$ est un nombre réel.
Donner alors, en fonction de $a$, l'ensemble des solutions de $(E)$.
\end{enumerate}
On rappelle que la température des macarons à l'instant $t = 0$ est égale à $-17$~\degres C et que, au bout de $15$~min, elle est de $-3$~\degres C.
\begin{enumerate}
\setcounter{enumi}{1}
\item En utilisant la condition à $t=0$ démontrer que $K = -33$.
\item En utilisant la condition à $t=15$ démontrer que $a \approx -0.04$.
\item En déduire l'expression de la solution de l'équation différentielle puis étudier ses variations.
\item La température idéale de dégustation des macarons étant de $13$~\degres C, Marie estime que
celle-ci sera atteinte au bout de $30$~min. A-t-elle raison ? Justifier la réponse.
Sinon, combien de temps faudra-t-il attendre ?
\end{enumerate}
\end{exercise}
\end{document}
%%% Local Variables:
%%% mode: latex
%%% TeX-master: "master"
%%% End:

View File

@ -0,0 +1,136 @@
\documentclass[a4paper,10pt]{article}
\usepackage{myXsim}
% Title Page
\title{DS8 \hfill BENALI Ilyas}
\tribe{TST sti2d}
\date{\hfillÀ render pour le vendredi 9 avril à 10h au plus tard}
\xsimsetup{
solution/print = true
}
\begin{document}
\maketitle
\begin{exercise}[subtitle={Étude de fonction}]
On considère la fonction $f$ définie sur $\intOF{0}{+\infty}$ par $ f(x) = 2.5x^2 + - 95x + 450\ln(x)$
\begin{enumerate}
\item Démontrer que la dérivée de $f$ est $f'(x) = \frac{5x^2 + - 95x + 450}{x}$.
\item Étude du numérateur de $f'(x)$: $N(x) = 5x^2 - 95x + 450$
\begin{enumerate}
\item Démontrer que $x=9$ et $x=10$ sont deux racines de $N(x)$..
\item Proposer une forme factorisée de $N(x)$.
\item Proposer une forme factorisée de $f'(x)$.
\end{enumerate}
\item Étudier le signe de $f'$ et en déduire les variations de $f$.
\end{enumerate}
\end{exercise}
\begin{solution}
\begin{enumerate}
\item pas de correction disponible
\item
\begin{enumerate}
\item \[N(9) = 0\]
\[N(10) = 0\]
\item \[
N(x) = 5(x - 9)(x - 10)
\]
\item
\[
f'(x) = \frac{5(x - 9)(x - 10)}{x}
\]
\end{enumerate}
\item Pas de correction disponible
\end{enumerate}
\end{solution}
\begin{exercise}[subtitle={Complexes}]
\begin{enumerate}
\item Mettre le nombre complexe suivant sous forme algébrique $z_1 = \dfrac{10 + 5 i}{-3 + 10 i} $
\item Mettre le complexe suivante sous forme exponentielle $z_2 = - 5 \sqrt{3} + 5 i$
\item Mettre le complexe suivante sous forme exponentielle $z_3 = - 8 \sqrt{2} - 8 \sqrt{2} i$
\item Calculer le produit $z_4=z_2\times z_3$ donner le résultat sous forme exponentielle puis algébrique.
\item Calculer le quotient $z_5=\frac{z_2}{z_3}$ donner le résultat sous forme exponentielle puis algébrique.
\end{enumerate}
\end{exercise}
\begin{solution}
\begin{enumerate}
\item $z_1 = \frac{20}{109} - \frac{115 i}{109}$
\item $z_2 = 10 e^{\frac{5 i \pi}{6}}$
\item $z_3 = 16 e^{- \frac{3 i \pi}{4}}$
\item $z_4 = 160 e^{\frac{i \pi}{12}} = 40 \sqrt{2} + 40 \sqrt{6} + i \left(- 40 \sqrt{2} + 40 \sqrt{6}\right) = 155.0 + 41.4 i$
\item $z_5 = \frac{5}{8} e^{\frac{19 i \pi}{12}} = - \frac{5 \sqrt{2}}{32} + \frac{5 \sqrt{6}}{32} + i \left(- \frac{5 \sqrt{6}}{32} - \frac{5 \sqrt{2}}{32}\right) = 0.162 - 0.604 i$
\end{enumerate}
\end{solution}
\begin{exercise}[subtitle={Sortie du congélateur}]
Marie a invité quelques amis pour le thé. Elle souhaite leur proposer ses macarons maison.
Elle les sort de son congélateur à $-17$~\degres C et les place dans une pièce à $17$~\degres C.
Au bout de 15 minutes, la température des macarons est de $-1$~\degres C.
\bigskip
\textbf{Premier modèle}
\medskip
On suppose que la vitesse de décongélation est constante : chaque minute la hausse de
température des macarons est la même.
Estimer dans ce cadre la température au bout de $30$~minutes, puis au bout de $45$~minutes.
Cette modélisation est-elle pertinente?
\bigskip
\textbf{Deuxième modèle}
\medskip
On suppose maintenant que la vitesse de décongélation est proportionnelle à la différence
de température entre les macarons et l'air ambiant (il s'agit de la loi de Newton).
On désigne par $\theta$ la température des macarons à l'instant $t$, et par $\theta'$ la vitesse de décongélation.
L'unité de temps est la minute et l'unité de température le degré Celsius.
\smallskip
On négligera la diminution de température de la pièce et on admettra donc qu'il existe un
nombre réel $a$ tel que, pour $t$ positif :
\[\theta'(t) = a [\theta(t) - 17]\quad (E)\]
\medskip
\begin{enumerate}
\item Vérifier que l'équation $(E)$ a pour solutions $\theta(t) = K e^{at} + 17$$K$ est un nombre réel.
Donner alors, en fonction de $a$, l'ensemble des solutions de $(E)$.
\end{enumerate}
On rappelle que la température des macarons à l'instant $t = 0$ est égale à $-17$~\degres C et que, au bout de $15$~min, elle est de $-1$~\degres C.
\begin{enumerate}
\setcounter{enumi}{1}
\item En utilisant la condition à $t=0$ démontrer que $K = -34$.
\item En utilisant la condition à $t=15$ démontrer que $a \approx -0.04$.
\item En déduire l'expression de la solution de l'équation différentielle puis étudier ses variations.
\item La température idéale de dégustation des macarons étant de $14$~\degres C, Marie estime que
celle-ci sera atteinte au bout de $30$~min. A-t-elle raison ? Justifier la réponse.
Sinon, combien de temps faudra-t-il attendre ?
\end{enumerate}
\end{exercise}
\end{document}
%%% Local Variables:
%%% mode: latex
%%% TeX-master: "master"
%%% End:

View File

@ -0,0 +1,136 @@
\documentclass[a4paper,10pt]{article}
\usepackage{myXsim}
% Title Page
\title{DS8 \hfill BERNADAT Noah}
\tribe{TST sti2d}
\date{\hfillÀ render pour le vendredi 9 avril à 10h au plus tard}
\xsimsetup{
solution/print = true
}
\begin{document}
\maketitle
\begin{exercise}[subtitle={Étude de fonction}]
On considère la fonction $f$ définie sur $\intOF{0}{+\infty}$ par $ f(x) = 3.5x^2 + - 21x + - 196\ln(x)$
\begin{enumerate}
\item Démontrer que la dérivée de $f$ est $f'(x) = \frac{7x^2 + - 21x + - 196}{x}$.
\item Étude du numérateur de $f'(x)$: $N(x) = 7x^2 - 21x - 196$
\begin{enumerate}
\item Démontrer que $x=- 4$ et $x=7$ sont deux racines de $N(x)$..
\item Proposer une forme factorisée de $N(x)$.
\item Proposer une forme factorisée de $f'(x)$.
\end{enumerate}
\item Étudier le signe de $f'$ et en déduire les variations de $f$.
\end{enumerate}
\end{exercise}
\begin{solution}
\begin{enumerate}
\item pas de correction disponible
\item
\begin{enumerate}
\item \[N(- 4) = 0\]
\[N(7) = 0\]
\item \[
N(x) = 7(x - - 4)(x - 7)
\]
\item
\[
f'(x) = \frac{7(x - - 4)(x - 7)}{x}
\]
\end{enumerate}
\item Pas de correction disponible
\end{enumerate}
\end{solution}
\begin{exercise}[subtitle={Complexes}]
\begin{enumerate}
\item Mettre le nombre complexe suivant sous forme algébrique $z_1 = \dfrac{8 + 9 i}{-9 + 5 i} $
\item Mettre le complexe suivante sous forme exponentielle $z_2 = - 8 \sqrt{3} + 8 i$
\item Mettre le complexe suivante sous forme exponentielle $z_3 = -5 - 5 \sqrt{3} i$
\item Calculer le produit $z_4=z_2\times z_3$ donner le résultat sous forme exponentielle puis algébrique.
\item Calculer le quotient $z_5=\frac{z_2}{z_3}$ donner le résultat sous forme exponentielle puis algébrique.
\end{enumerate}
\end{exercise}
\begin{solution}
\begin{enumerate}
\item $z_1 = - \frac{27}{106} - \frac{121 i}{106}$
\item $z_2 = 16 e^{\frac{5 i \pi}{6}}$
\item $z_3 = 10 e^{- \frac{2 i \pi}{3}}$
\item $z_4 = 160 e^{\frac{i \pi}{6}} = 80 \sqrt{3} + 80 i = 139.0 + 80.0 i$
\item $z_5 = \frac{8}{5} e^{\frac{3 i \pi}{2}} = - \frac{8 i}{5} = - 1.6 i$
\end{enumerate}
\end{solution}
\begin{exercise}[subtitle={Sortie du congélateur}]
Marie a invité quelques amis pour le thé. Elle souhaite leur proposer ses macarons maison.
Elle les sort de son congélateur à $-15$~\degres C et les place dans une pièce à $22$~\degres C.
Au bout de 15 minutes, la température des macarons est de $-1$~\degres C.
\bigskip
\textbf{Premier modèle}
\medskip
On suppose que la vitesse de décongélation est constante : chaque minute la hausse de
température des macarons est la même.
Estimer dans ce cadre la température au bout de $30$~minutes, puis au bout de $45$~minutes.
Cette modélisation est-elle pertinente?
\bigskip
\textbf{Deuxième modèle}
\medskip
On suppose maintenant que la vitesse de décongélation est proportionnelle à la différence
de température entre les macarons et l'air ambiant (il s'agit de la loi de Newton).
On désigne par $\theta$ la température des macarons à l'instant $t$, et par $\theta'$ la vitesse de décongélation.
L'unité de temps est la minute et l'unité de température le degré Celsius.
\smallskip
On négligera la diminution de température de la pièce et on admettra donc qu'il existe un
nombre réel $a$ tel que, pour $t$ positif :
\[\theta'(t) = a [\theta(t) - 22]\quad (E)\]
\medskip
\begin{enumerate}
\item Vérifier que l'équation $(E)$ a pour solutions $\theta(t) = K e^{at} + 22$$K$ est un nombre réel.
Donner alors, en fonction de $a$, l'ensemble des solutions de $(E)$.
\end{enumerate}
On rappelle que la température des macarons à l'instant $t = 0$ est égale à $-15$~\degres C et que, au bout de $15$~min, elle est de $-1$~\degres C.
\begin{enumerate}
\setcounter{enumi}{1}
\item En utilisant la condition à $t=0$ démontrer que $K = -37$.
\item En utilisant la condition à $t=15$ démontrer que $a \approx -0.03$.
\item En déduire l'expression de la solution de l'équation différentielle puis étudier ses variations.
\item La température idéale de dégustation des macarons étant de $19$~\degres C, Marie estime que
celle-ci sera atteinte au bout de $30$~min. A-t-elle raison ? Justifier la réponse.
Sinon, combien de temps faudra-t-il attendre ?
\end{enumerate}
\end{exercise}
\end{document}
%%% Local Variables:
%%% mode: latex
%%% TeX-master: "master"
%%% End:

View File

@ -0,0 +1,136 @@
\documentclass[a4paper,10pt]{article}
\usepackage{myXsim}
% Title Page
\title{DS8 \hfill BUDIN Nathan}
\tribe{TST sti2d}
\date{\hfillÀ render pour le vendredi 9 avril à 10h au plus tard}
\xsimsetup{
solution/print = true
}
\begin{document}
\maketitle
\begin{exercise}[subtitle={Étude de fonction}]
On considère la fonction $f$ définie sur $\intOF{0}{+\infty}$ par $ f(x) = 3.5x^2 + - 84x + 140\ln(x)$
\begin{enumerate}
\item Démontrer que la dérivée de $f$ est $f'(x) = \frac{7x^2 + - 84x + 140}{x}$.
\item Étude du numérateur de $f'(x)$: $N(x) = 7x^2 - 84x + 140$
\begin{enumerate}
\item Démontrer que $x=10$ et $x=2$ sont deux racines de $N(x)$..
\item Proposer une forme factorisée de $N(x)$.
\item Proposer une forme factorisée de $f'(x)$.
\end{enumerate}
\item Étudier le signe de $f'$ et en déduire les variations de $f$.
\end{enumerate}
\end{exercise}
\begin{solution}
\begin{enumerate}
\item pas de correction disponible
\item
\begin{enumerate}
\item \[N(10) = 0\]
\[N(2) = 0\]
\item \[
N(x) = 7(x - 10)(x - 2)
\]
\item
\[
f'(x) = \frac{7(x - 10)(x - 2)}{x}
\]
\end{enumerate}
\item Pas de correction disponible
\end{enumerate}
\end{solution}
\begin{exercise}[subtitle={Complexes}]
\begin{enumerate}
\item Mettre le nombre complexe suivant sous forme algébrique $z_1 = \dfrac{8 + 8 i}{-7 + 4 i} $
\item Mettre le complexe suivante sous forme exponentielle $z_2 = 7 \sqrt{3} - 7 i$
\item Mettre le complexe suivante sous forme exponentielle $z_3 = - 2 \sqrt{3} + 2 i$
\item Calculer le produit $z_4=z_2\times z_3$ donner le résultat sous forme exponentielle puis algébrique.
\item Calculer le quotient $z_5=\frac{z_2}{z_3}$ donner le résultat sous forme exponentielle puis algébrique.
\end{enumerate}
\end{exercise}
\begin{solution}
\begin{enumerate}
\item $z_1 = - \frac{24}{65} - \frac{88 i}{65}$
\item $z_2 = 14 e^{- \frac{i \pi}{6}}$
\item $z_3 = 4 e^{\frac{5 i \pi}{6}}$
\item $z_4 = 56 e^{\frac{2 i \pi}{3}} = -28 + 28 \sqrt{3} i = -28.0 + 48.5 i$
\item $z_5 = \frac{7}{2} e^{- i \pi} = - \frac{7}{2} = -3.5$
\end{enumerate}
\end{solution}
\begin{exercise}[subtitle={Sortie du congélateur}]
Marie a invité quelques amis pour le thé. Elle souhaite leur proposer ses macarons maison.
Elle les sort de son congélateur à $-19$~\degres C et les place dans une pièce à $23$~\degres C.
Au bout de 15 minutes, la température des macarons est de $3$~\degres C.
\bigskip
\textbf{Premier modèle}
\medskip
On suppose que la vitesse de décongélation est constante : chaque minute la hausse de
température des macarons est la même.
Estimer dans ce cadre la température au bout de $30$~minutes, puis au bout de $45$~minutes.
Cette modélisation est-elle pertinente?
\bigskip
\textbf{Deuxième modèle}
\medskip
On suppose maintenant que la vitesse de décongélation est proportionnelle à la différence
de température entre les macarons et l'air ambiant (il s'agit de la loi de Newton).
On désigne par $\theta$ la température des macarons à l'instant $t$, et par $\theta'$ la vitesse de décongélation.
L'unité de temps est la minute et l'unité de température le degré Celsius.
\smallskip
On négligera la diminution de température de la pièce et on admettra donc qu'il existe un
nombre réel $a$ tel que, pour $t$ positif :
\[\theta'(t) = a [\theta(t) - 23]\quad (E)\]
\medskip
\begin{enumerate}
\item Vérifier que l'équation $(E)$ a pour solutions $\theta(t) = K e^{at} + 23$$K$ est un nombre réel.
Donner alors, en fonction de $a$, l'ensemble des solutions de $(E)$.
\end{enumerate}
On rappelle que la température des macarons à l'instant $t = 0$ est égale à $-19$~\degres C et que, au bout de $15$~min, elle est de $3$~\degres C.
\begin{enumerate}
\setcounter{enumi}{1}
\item En utilisant la condition à $t=0$ démontrer que $K = -42$.
\item En utilisant la condition à $t=15$ démontrer que $a \approx -0.05$.
\item En déduire l'expression de la solution de l'équation différentielle puis étudier ses variations.
\item La température idéale de dégustation des macarons étant de $20$~\degres C, Marie estime que
celle-ci sera atteinte au bout de $30$~min. A-t-elle raison ? Justifier la réponse.
Sinon, combien de temps faudra-t-il attendre ?
\end{enumerate}
\end{exercise}
\end{document}
%%% Local Variables:
%%% mode: latex
%%% TeX-master: "master"
%%% End:

View File

@ -0,0 +1,136 @@
\documentclass[a4paper,10pt]{article}
\usepackage{myXsim}
% Title Page
\title{DS8 \hfill CHION Léa}
\tribe{TST sti2d}
\date{\hfillÀ render pour le vendredi 9 avril à 10h au plus tard}
\xsimsetup{
solution/print = true
}
\begin{document}
\maketitle
\begin{exercise}[subtitle={Étude de fonction}]
On considère la fonction $f$ définie sur $\intOF{0}{+\infty}$ par $ f(x) = 5x^2 + 130x + 300\ln(x)$
\begin{enumerate}
\item Démontrer que la dérivée de $f$ est $f'(x) = \frac{10x^2 + 130x + 300}{x}$.
\item Étude du numérateur de $f'(x)$: $N(x) = 10x^2 + 130x + 300$
\begin{enumerate}
\item Démontrer que $x=- 10$ et $x=- 3$ sont deux racines de $N(x)$..
\item Proposer une forme factorisée de $N(x)$.
\item Proposer une forme factorisée de $f'(x)$.
\end{enumerate}
\item Étudier le signe de $f'$ et en déduire les variations de $f$.
\end{enumerate}
\end{exercise}
\begin{solution}
\begin{enumerate}
\item pas de correction disponible
\item
\begin{enumerate}
\item \[N(- 10) = 0\]
\[N(- 3) = 0\]
\item \[
N(x) = 10(x - - 10)(x - - 3)
\]
\item
\[
f'(x) = \frac{10(x - - 10)(x - - 3)}{x}
\]
\end{enumerate}
\item Pas de correction disponible
\end{enumerate}
\end{solution}
\begin{exercise}[subtitle={Complexes}]
\begin{enumerate}
\item Mettre le nombre complexe suivant sous forme algébrique $z_1 = \dfrac{8 + 10 i}{-3 + 2 i} $
\item Mettre le complexe suivante sous forme exponentielle $z_2 = - \sqrt{2} - \sqrt{2} i$
\item Mettre le complexe suivante sous forme exponentielle $z_3 = - 10 \sqrt{2} + 10 \sqrt{2} i$
\item Calculer le produit $z_4=z_2\times z_3$ donner le résultat sous forme exponentielle puis algébrique.
\item Calculer le quotient $z_5=\frac{z_2}{z_3}$ donner le résultat sous forme exponentielle puis algébrique.
\end{enumerate}
\end{exercise}
\begin{solution}
\begin{enumerate}
\item $z_1 = - \frac{4}{13} - \frac{46 i}{13}$
\item $z_2 = 2 e^{- \frac{3 i \pi}{4}}$
\item $z_3 = 20 e^{\frac{3 i \pi}{4}}$
\item $z_4 = 40 e^{0} = 40 = 40.0$
\item $z_5 = \frac{1}{10} e^{- \frac{3 i \pi}{2}} = \frac{i}{10} = 0.1 i$
\end{enumerate}
\end{solution}
\begin{exercise}[subtitle={Sortie du congélateur}]
Marie a invité quelques amis pour le thé. Elle souhaite leur proposer ses macarons maison.
Elle les sort de son congélateur à $-20$~\degres C et les place dans une pièce à $24$~\degres C.
Au bout de 15 minutes, la température des macarons est de $-3$~\degres C.
\bigskip
\textbf{Premier modèle}
\medskip
On suppose que la vitesse de décongélation est constante : chaque minute la hausse de
température des macarons est la même.
Estimer dans ce cadre la température au bout de $30$~minutes, puis au bout de $45$~minutes.
Cette modélisation est-elle pertinente?
\bigskip
\textbf{Deuxième modèle}
\medskip
On suppose maintenant que la vitesse de décongélation est proportionnelle à la différence
de température entre les macarons et l'air ambiant (il s'agit de la loi de Newton).
On désigne par $\theta$ la température des macarons à l'instant $t$, et par $\theta'$ la vitesse de décongélation.
L'unité de temps est la minute et l'unité de température le degré Celsius.
\smallskip
On négligera la diminution de température de la pièce et on admettra donc qu'il existe un
nombre réel $a$ tel que, pour $t$ positif :
\[\theta'(t) = a [\theta(t) - 24]\quad (E)\]
\medskip
\begin{enumerate}
\item Vérifier que l'équation $(E)$ a pour solutions $\theta(t) = K e^{at} + 24$$K$ est un nombre réel.
Donner alors, en fonction de $a$, l'ensemble des solutions de $(E)$.
\end{enumerate}
On rappelle que la température des macarons à l'instant $t = 0$ est égale à $-20$~\degres C et que, au bout de $15$~min, elle est de $-3$~\degres C.
\begin{enumerate}
\setcounter{enumi}{1}
\item En utilisant la condition à $t=0$ démontrer que $K = -44$.
\item En utilisant la condition à $t=15$ démontrer que $a \approx -0.03$.
\item En déduire l'expression de la solution de l'équation différentielle puis étudier ses variations.
\item La température idéale de dégustation des macarons étant de $21$~\degres C, Marie estime que
celle-ci sera atteinte au bout de $30$~min. A-t-elle raison ? Justifier la réponse.
Sinon, combien de temps faudra-t-il attendre ?
\end{enumerate}
\end{exercise}
\end{document}
%%% Local Variables:
%%% mode: latex
%%% TeX-master: "master"
%%% End:

View File

@ -0,0 +1,136 @@
\documentclass[a4paper,10pt]{article}
\usepackage{myXsim}
% Title Page
\title{DS8 \hfill CLAIN Avinash}
\tribe{TST sti2d}
\date{\hfillÀ render pour le vendredi 9 avril à 10h au plus tard}
\xsimsetup{
solution/print = true
}
\begin{document}
\maketitle
\begin{exercise}[subtitle={Étude de fonction}]
On considère la fonction $f$ définie sur $\intOF{0}{+\infty}$ par $ f(x) = 3.5x^2 + - 42x + - 280\ln(x)$
\begin{enumerate}
\item Démontrer que la dérivée de $f$ est $f'(x) = \frac{7x^2 + - 42x + - 280}{x}$.
\item Étude du numérateur de $f'(x)$: $N(x) = 7x^2 - 42x - 280$
\begin{enumerate}
\item Démontrer que $x=10$ et $x=- 4$ sont deux racines de $N(x)$..
\item Proposer une forme factorisée de $N(x)$.
\item Proposer une forme factorisée de $f'(x)$.
\end{enumerate}
\item Étudier le signe de $f'$ et en déduire les variations de $f$.
\end{enumerate}
\end{exercise}
\begin{solution}
\begin{enumerate}
\item pas de correction disponible
\item
\begin{enumerate}
\item \[N(10) = 0\]
\[N(- 4) = 0\]
\item \[
N(x) = 7(x - 10)(x - - 4)
\]
\item
\[
f'(x) = \frac{7(x - 10)(x - - 4)}{x}
\]
\end{enumerate}
\item Pas de correction disponible
\end{enumerate}
\end{solution}
\begin{exercise}[subtitle={Complexes}]
\begin{enumerate}
\item Mettre le nombre complexe suivant sous forme algébrique $z_1 = \dfrac{3 + 8 i}{-10 + 5 i} $
\item Mettre le complexe suivante sous forme exponentielle $z_2 = 1 + \sqrt{3} i$
\item Mettre le complexe suivante sous forme exponentielle $z_3 = -3 + 3 \sqrt{3} i$
\item Calculer le produit $z_4=z_2\times z_3$ donner le résultat sous forme exponentielle puis algébrique.
\item Calculer le quotient $z_5=\frac{z_2}{z_3}$ donner le résultat sous forme exponentielle puis algébrique.
\end{enumerate}
\end{exercise}
\begin{solution}
\begin{enumerate}
\item $z_1 = \frac{2}{25} - \frac{19 i}{25}$
\item $z_2 = 2 e^{\frac{i \pi}{3}}$
\item $z_3 = 6 e^{\frac{2 i \pi}{3}}$
\item $z_4 = 12 e^{i \pi} = -12 = -12.0$
\item $z_5 = \frac{1}{3} e^{- \frac{i \pi}{3}} = \frac{1}{6} - \frac{\sqrt{3} i}{6} = 0.167 - 0.289 i$
\end{enumerate}
\end{solution}
\begin{exercise}[subtitle={Sortie du congélateur}]
Marie a invité quelques amis pour le thé. Elle souhaite leur proposer ses macarons maison.
Elle les sort de son congélateur à $-20$~\degres C et les place dans une pièce à $17$~\degres C.
Au bout de 15 minutes, la température des macarons est de $4$~\degres C.
\bigskip
\textbf{Premier modèle}
\medskip
On suppose que la vitesse de décongélation est constante : chaque minute la hausse de
température des macarons est la même.
Estimer dans ce cadre la température au bout de $30$~minutes, puis au bout de $45$~minutes.
Cette modélisation est-elle pertinente?
\bigskip
\textbf{Deuxième modèle}
\medskip
On suppose maintenant que la vitesse de décongélation est proportionnelle à la différence
de température entre les macarons et l'air ambiant (il s'agit de la loi de Newton).
On désigne par $\theta$ la température des macarons à l'instant $t$, et par $\theta'$ la vitesse de décongélation.
L'unité de temps est la minute et l'unité de température le degré Celsius.
\smallskip
On négligera la diminution de température de la pièce et on admettra donc qu'il existe un
nombre réel $a$ tel que, pour $t$ positif :
\[\theta'(t) = a [\theta(t) - 17]\quad (E)\]
\medskip
\begin{enumerate}
\item Vérifier que l'équation $(E)$ a pour solutions $\theta(t) = K e^{at} + 17$$K$ est un nombre réel.
Donner alors, en fonction de $a$, l'ensemble des solutions de $(E)$.
\end{enumerate}
On rappelle que la température des macarons à l'instant $t = 0$ est égale à $-20$~\degres C et que, au bout de $15$~min, elle est de $4$~\degres C.
\begin{enumerate}
\setcounter{enumi}{1}
\item En utilisant la condition à $t=0$ démontrer que $K = -37$.
\item En utilisant la condition à $t=15$ démontrer que $a \approx -0.07$.
\item En déduire l'expression de la solution de l'équation différentielle puis étudier ses variations.
\item La température idéale de dégustation des macarons étant de $14$~\degres C, Marie estime que
celle-ci sera atteinte au bout de $30$~min. A-t-elle raison ? Justifier la réponse.
Sinon, combien de temps faudra-t-il attendre ?
\end{enumerate}
\end{exercise}
\end{document}
%%% Local Variables:
%%% mode: latex
%%% TeX-master: "master"
%%% End:

View File

@ -0,0 +1,136 @@
\documentclass[a4paper,10pt]{article}
\usepackage{myXsim}
% Title Page
\title{DS8 \hfill COUBAT Alexis}
\tribe{TST sti2d}
\date{\hfillÀ render pour le vendredi 9 avril à 10h au plus tard}
\xsimsetup{
solution/print = true
}
\begin{document}
\maketitle
\begin{exercise}[subtitle={Étude de fonction}]
On considère la fonction $f$ définie sur $\intOF{0}{+\infty}$ par $ f(x) = 5x^2 + 70x + 60\ln(x)$
\begin{enumerate}
\item Démontrer que la dérivée de $f$ est $f'(x) = \frac{10x^2 + 70x + 60}{x}$.
\item Étude du numérateur de $f'(x)$: $N(x) = 10x^2 + 70x + 60$
\begin{enumerate}
\item Démontrer que $x=- 6$ et $x=- 1$ sont deux racines de $N(x)$..
\item Proposer une forme factorisée de $N(x)$.
\item Proposer une forme factorisée de $f'(x)$.
\end{enumerate}
\item Étudier le signe de $f'$ et en déduire les variations de $f$.
\end{enumerate}
\end{exercise}
\begin{solution}
\begin{enumerate}
\item pas de correction disponible
\item
\begin{enumerate}
\item \[N(- 6) = 0\]
\[N(- 1) = 0\]
\item \[
N(x) = 10(x - - 6)(x - - 1)
\]
\item
\[
f'(x) = \frac{10(x - - 6)(x - - 1)}{x}
\]
\end{enumerate}
\item Pas de correction disponible
\end{enumerate}
\end{solution}
\begin{exercise}[subtitle={Complexes}]
\begin{enumerate}
\item Mettre le nombre complexe suivant sous forme algébrique $z_1 = \dfrac{3 + 5 i}{-4 + 5 i} $
\item Mettre le complexe suivante sous forme exponentielle $z_2 = 5 + 5 \sqrt{3} i$
\item Mettre le complexe suivante sous forme exponentielle $z_3 = - 4 \sqrt{3} - 4 i$
\item Calculer le produit $z_4=z_2\times z_3$ donner le résultat sous forme exponentielle puis algébrique.
\item Calculer le quotient $z_5=\frac{z_2}{z_3}$ donner le résultat sous forme exponentielle puis algébrique.
\end{enumerate}
\end{exercise}
\begin{solution}
\begin{enumerate}
\item $z_1 = \frac{13}{41} - \frac{35 i}{41}$
\item $z_2 = 10 e^{\frac{i \pi}{3}}$
\item $z_3 = 8 e^{- \frac{5 i \pi}{6}}$
\item $z_4 = 80 e^{- \frac{i \pi}{2}} = - 80 i = - 80.0 i$
\item $z_5 = \frac{5}{4} e^{\frac{7 i \pi}{6}} = - \frac{5 \sqrt{3}}{8} - \frac{5 i}{8} = -1.08 - 0.625 i$
\end{enumerate}
\end{solution}
\begin{exercise}[subtitle={Sortie du congélateur}]
Marie a invité quelques amis pour le thé. Elle souhaite leur proposer ses macarons maison.
Elle les sort de son congélateur à $-20$~\degres C et les place dans une pièce à $21$~\degres C.
Au bout de 15 minutes, la température des macarons est de $4$~\degres C.
\bigskip
\textbf{Premier modèle}
\medskip
On suppose que la vitesse de décongélation est constante : chaque minute la hausse de
température des macarons est la même.
Estimer dans ce cadre la température au bout de $30$~minutes, puis au bout de $45$~minutes.
Cette modélisation est-elle pertinente?
\bigskip
\textbf{Deuxième modèle}
\medskip
On suppose maintenant que la vitesse de décongélation est proportionnelle à la différence
de température entre les macarons et l'air ambiant (il s'agit de la loi de Newton).
On désigne par $\theta$ la température des macarons à l'instant $t$, et par $\theta'$ la vitesse de décongélation.
L'unité de temps est la minute et l'unité de température le degré Celsius.
\smallskip
On négligera la diminution de température de la pièce et on admettra donc qu'il existe un
nombre réel $a$ tel que, pour $t$ positif :
\[\theta'(t) = a [\theta(t) - 21]\quad (E)\]
\medskip
\begin{enumerate}
\item Vérifier que l'équation $(E)$ a pour solutions $\theta(t) = K e^{at} + 21$$K$ est un nombre réel.
Donner alors, en fonction de $a$, l'ensemble des solutions de $(E)$.
\end{enumerate}
On rappelle que la température des macarons à l'instant $t = 0$ est égale à $-20$~\degres C et que, au bout de $15$~min, elle est de $4$~\degres C.
\begin{enumerate}
\setcounter{enumi}{1}
\item En utilisant la condition à $t=0$ démontrer que $K = -41$.
\item En utilisant la condition à $t=15$ démontrer que $a \approx -0.06$.
\item En déduire l'expression de la solution de l'équation différentielle puis étudier ses variations.
\item La température idéale de dégustation des macarons étant de $18$~\degres C, Marie estime que
celle-ci sera atteinte au bout de $30$~min. A-t-elle raison ? Justifier la réponse.
Sinon, combien de temps faudra-t-il attendre ?
\end{enumerate}
\end{exercise}
\end{document}
%%% Local Variables:
%%% mode: latex
%%% TeX-master: "master"
%%% End:

View File

@ -0,0 +1,136 @@
\documentclass[a4paper,10pt]{article}
\usepackage{myXsim}
% Title Page
\title{DS8 \hfill EVRARD Jules}
\tribe{TST sti2d}
\date{\hfillÀ render pour le vendredi 9 avril à 10h au plus tard}
\xsimsetup{
solution/print = true
}
\begin{document}
\maketitle
\begin{exercise}[subtitle={Étude de fonction}]
On considère la fonction $f$ définie sur $\intOF{0}{+\infty}$ par $ f(x) = 5x^2 + 0x + - 360\ln(x)$
\begin{enumerate}
\item Démontrer que la dérivée de $f$ est $f'(x) = \frac{10x^2 + 0x + - 360}{x}$.
\item Étude du numérateur de $f'(x)$: $N(x) = 10x^2 - 360$
\begin{enumerate}
\item Démontrer que $x=6$ et $x=- 6$ sont deux racines de $N(x)$..
\item Proposer une forme factorisée de $N(x)$.
\item Proposer une forme factorisée de $f'(x)$.
\end{enumerate}
\item Étudier le signe de $f'$ et en déduire les variations de $f$.
\end{enumerate}
\end{exercise}
\begin{solution}
\begin{enumerate}
\item pas de correction disponible
\item
\begin{enumerate}
\item \[N(6) = 0\]
\[N(- 6) = 0\]
\item \[
N(x) = 10(x - 6)(x - - 6)
\]
\item
\[
f'(x) = \frac{10(x - 6)(x - - 6)}{x}
\]
\end{enumerate}
\item Pas de correction disponible
\end{enumerate}
\end{solution}
\begin{exercise}[subtitle={Complexes}]
\begin{enumerate}
\item Mettre le nombre complexe suivant sous forme algébrique $z_1 = \dfrac{10 + 8 i}{-5 + 7 i} $
\item Mettre le complexe suivante sous forme exponentielle $z_2 = 9 \sqrt{2} - 9 \sqrt{2} i$
\item Mettre le complexe suivante sous forme exponentielle $z_3 = 7 \sqrt{3} + 7 i$
\item Calculer le produit $z_4=z_2\times z_3$ donner le résultat sous forme exponentielle puis algébrique.
\item Calculer le quotient $z_5=\frac{z_2}{z_3}$ donner le résultat sous forme exponentielle puis algébrique.
\end{enumerate}
\end{exercise}
\begin{solution}
\begin{enumerate}
\item $z_1 = \frac{3}{37} - \frac{55 i}{37}$
\item $z_2 = 18 e^{- \frac{i \pi}{4}}$
\item $z_3 = 14 e^{\frac{i \pi}{6}}$
\item $z_4 = 252 e^{- \frac{i \pi}{12}} = 63 \sqrt{2} + 63 \sqrt{6} + i \left(- 63 \sqrt{6} + 63 \sqrt{2}\right) = 243.0 - 65.2 i$
\item $z_5 = \frac{9}{7} e^{- \frac{5 i \pi}{12}} = - \frac{9 \sqrt{2}}{28} + \frac{9 \sqrt{6}}{28} + i \left(- \frac{9 \sqrt{6}}{28} - \frac{9 \sqrt{2}}{28}\right) = 0.333 - 1.24 i$
\end{enumerate}
\end{solution}
\begin{exercise}[subtitle={Sortie du congélateur}]
Marie a invité quelques amis pour le thé. Elle souhaite leur proposer ses macarons maison.
Elle les sort de son congélateur à $-15$~\degres C et les place dans une pièce à $17$~\degres C.
Au bout de 15 minutes, la température des macarons est de $-1$~\degres C.
\bigskip
\textbf{Premier modèle}
\medskip
On suppose que la vitesse de décongélation est constante : chaque minute la hausse de
température des macarons est la même.
Estimer dans ce cadre la température au bout de $30$~minutes, puis au bout de $45$~minutes.
Cette modélisation est-elle pertinente?
\bigskip
\textbf{Deuxième modèle}
\medskip
On suppose maintenant que la vitesse de décongélation est proportionnelle à la différence
de température entre les macarons et l'air ambiant (il s'agit de la loi de Newton).
On désigne par $\theta$ la température des macarons à l'instant $t$, et par $\theta'$ la vitesse de décongélation.
L'unité de temps est la minute et l'unité de température le degré Celsius.
\smallskip
On négligera la diminution de température de la pièce et on admettra donc qu'il existe un
nombre réel $a$ tel que, pour $t$ positif :
\[\theta'(t) = a [\theta(t) - 17]\quad (E)\]
\medskip
\begin{enumerate}
\item Vérifier que l'équation $(E)$ a pour solutions $\theta(t) = K e^{at} + 17$$K$ est un nombre réel.
Donner alors, en fonction de $a$, l'ensemble des solutions de $(E)$.
\end{enumerate}
On rappelle que la température des macarons à l'instant $t = 0$ est égale à $-15$~\degres C et que, au bout de $15$~min, elle est de $-1$~\degres C.
\begin{enumerate}
\setcounter{enumi}{1}
\item En utilisant la condition à $t=0$ démontrer que $K = -32$.
\item En utilisant la condition à $t=15$ démontrer que $a \approx -0.04$.
\item En déduire l'expression de la solution de l'équation différentielle puis étudier ses variations.
\item La température idéale de dégustation des macarons étant de $14$~\degres C, Marie estime que
celle-ci sera atteinte au bout de $30$~min. A-t-elle raison ? Justifier la réponse.
Sinon, combien de temps faudra-t-il attendre ?
\end{enumerate}
\end{exercise}
\end{document}
%%% Local Variables:
%%% mode: latex
%%% TeX-master: "master"
%%% End:

View File

@ -0,0 +1,136 @@
\documentclass[a4paper,10pt]{article}
\usepackage{myXsim}
% Title Page
\title{DS8 \hfill HADJRAS Mohcine}
\tribe{TST sti2d}
\date{\hfillÀ render pour le vendredi 9 avril à 10h au plus tard}
\xsimsetup{
solution/print = true
}
\begin{document}
\maketitle
\begin{exercise}[subtitle={Étude de fonction}]
On considère la fonction $f$ définie sur $\intOF{0}{+\infty}$ par $ f(x) = 3x^2 + 78x + 240\ln(x)$
\begin{enumerate}
\item Démontrer que la dérivée de $f$ est $f'(x) = \frac{6x^2 + 78x + 240}{x}$.
\item Étude du numérateur de $f'(x)$: $N(x) = 6x^2 + 78x + 240$
\begin{enumerate}
\item Démontrer que $x=- 5$ et $x=- 8$ sont deux racines de $N(x)$..
\item Proposer une forme factorisée de $N(x)$.
\item Proposer une forme factorisée de $f'(x)$.
\end{enumerate}
\item Étudier le signe de $f'$ et en déduire les variations de $f$.
\end{enumerate}
\end{exercise}
\begin{solution}
\begin{enumerate}
\item pas de correction disponible
\item
\begin{enumerate}
\item \[N(- 5) = 0\]
\[N(- 8) = 0\]
\item \[
N(x) = 6(x - - 5)(x - - 8)
\]
\item
\[
f'(x) = \frac{6(x - - 5)(x - - 8)}{x}
\]
\end{enumerate}
\item Pas de correction disponible
\end{enumerate}
\end{solution}
\begin{exercise}[subtitle={Complexes}]
\begin{enumerate}
\item Mettre le nombre complexe suivant sous forme algébrique $z_1 = \dfrac{10 + 5 i}{-6 + 4 i} $
\item Mettre le complexe suivante sous forme exponentielle $z_2 = 1 + \sqrt{3} i$
\item Mettre le complexe suivante sous forme exponentielle $z_3 = -7 - 7 \sqrt{3} i$
\item Calculer le produit $z_4=z_2\times z_3$ donner le résultat sous forme exponentielle puis algébrique.
\item Calculer le quotient $z_5=\frac{z_2}{z_3}$ donner le résultat sous forme exponentielle puis algébrique.
\end{enumerate}
\end{exercise}
\begin{solution}
\begin{enumerate}
\item $z_1 = - \frac{10}{13} - \frac{35 i}{26}$
\item $z_2 = 2 e^{\frac{i \pi}{3}}$
\item $z_3 = 14 e^{- \frac{2 i \pi}{3}}$
\item $z_4 = 28 e^{- \frac{i \pi}{3}} = 14 - 14 \sqrt{3} i = 14.0 - 24.3 i$
\item $z_5 = \frac{1}{7} e^{i \pi} = - \frac{1}{7} = -0.143$
\end{enumerate}
\end{solution}
\begin{exercise}[subtitle={Sortie du congélateur}]
Marie a invité quelques amis pour le thé. Elle souhaite leur proposer ses macarons maison.
Elle les sort de son congélateur à $-18$~\degres C et les place dans une pièce à $25$~\degres C.
Au bout de 15 minutes, la température des macarons est de $4$~\degres C.
\bigskip
\textbf{Premier modèle}
\medskip
On suppose que la vitesse de décongélation est constante : chaque minute la hausse de
température des macarons est la même.
Estimer dans ce cadre la température au bout de $30$~minutes, puis au bout de $45$~minutes.
Cette modélisation est-elle pertinente?
\bigskip
\textbf{Deuxième modèle}
\medskip
On suppose maintenant que la vitesse de décongélation est proportionnelle à la différence
de température entre les macarons et l'air ambiant (il s'agit de la loi de Newton).
On désigne par $\theta$ la température des macarons à l'instant $t$, et par $\theta'$ la vitesse de décongélation.
L'unité de temps est la minute et l'unité de température le degré Celsius.
\smallskip
On négligera la diminution de température de la pièce et on admettra donc qu'il existe un
nombre réel $a$ tel que, pour $t$ positif :
\[\theta'(t) = a [\theta(t) - 25]\quad (E)\]
\medskip
\begin{enumerate}
\item Vérifier que l'équation $(E)$ a pour solutions $\theta(t) = K e^{at} + 25$$K$ est un nombre réel.
Donner alors, en fonction de $a$, l'ensemble des solutions de $(E)$.
\end{enumerate}
On rappelle que la température des macarons à l'instant $t = 0$ est égale à $-18$~\degres C et que, au bout de $15$~min, elle est de $4$~\degres C.
\begin{enumerate}
\setcounter{enumi}{1}
\item En utilisant la condition à $t=0$ démontrer que $K = -43$.
\item En utilisant la condition à $t=15$ démontrer que $a \approx -0.05$.
\item En déduire l'expression de la solution de l'équation différentielle puis étudier ses variations.
\item La température idéale de dégustation des macarons étant de $22$~\degres C, Marie estime que
celle-ci sera atteinte au bout de $30$~min. A-t-elle raison ? Justifier la réponse.
Sinon, combien de temps faudra-t-il attendre ?
\end{enumerate}
\end{exercise}
\end{document}
%%% Local Variables:
%%% mode: latex
%%% TeX-master: "master"
%%% End:

View File

@ -0,0 +1,136 @@
\documentclass[a4paper,10pt]{article}
\usepackage{myXsim}
% Title Page
\title{DS8 \hfill HENRIST Maxime}
\tribe{TST sti2d}
\date{\hfillÀ render pour le vendredi 9 avril à 10h au plus tard}
\xsimsetup{
solution/print = true
}
\begin{document}
\maketitle
\begin{exercise}[subtitle={Étude de fonction}]
On considère la fonction $f$ définie sur $\intOF{0}{+\infty}$ par $ f(x) = 4.5x^2 + - 27x + - 36\ln(x)$
\begin{enumerate}
\item Démontrer que la dérivée de $f$ est $f'(x) = \frac{9x^2 + - 27x + - 36}{x}$.
\item Étude du numérateur de $f'(x)$: $N(x) = 9x^2 - 27x - 36$
\begin{enumerate}
\item Démontrer que $x=4$ et $x=- 1$ sont deux racines de $N(x)$..
\item Proposer une forme factorisée de $N(x)$.
\item Proposer une forme factorisée de $f'(x)$.
\end{enumerate}
\item Étudier le signe de $f'$ et en déduire les variations de $f$.
\end{enumerate}
\end{exercise}
\begin{solution}
\begin{enumerate}
\item pas de correction disponible
\item
\begin{enumerate}
\item \[N(4) = 0\]
\[N(- 1) = 0\]
\item \[
N(x) = 9(x - 4)(x - - 1)
\]
\item
\[
f'(x) = \frac{9(x - 4)(x - - 1)}{x}
\]
\end{enumerate}
\item Pas de correction disponible
\end{enumerate}
\end{solution}
\begin{exercise}[subtitle={Complexes}]
\begin{enumerate}
\item Mettre le nombre complexe suivant sous forme algébrique $z_1 = \dfrac{6 + 5 i}{-6 + 5 i} $
\item Mettre le complexe suivante sous forme exponentielle $z_2 = 8 \sqrt{3} - 8 i$
\item Mettre le complexe suivante sous forme exponentielle $z_3 = 5 - 5 \sqrt{3} i$
\item Calculer le produit $z_4=z_2\times z_3$ donner le résultat sous forme exponentielle puis algébrique.
\item Calculer le quotient $z_5=\frac{z_2}{z_3}$ donner le résultat sous forme exponentielle puis algébrique.
\end{enumerate}
\end{exercise}
\begin{solution}
\begin{enumerate}
\item $z_1 = - \frac{11}{61} - \frac{60 i}{61}$
\item $z_2 = 16 e^{- \frac{i \pi}{6}}$
\item $z_3 = 10 e^{- \frac{i \pi}{3}}$
\item $z_4 = 160 e^{- \frac{i \pi}{2}} = - 160 i = - 160.0 i$
\item $z_5 = \frac{8}{5} e^{\frac{i \pi}{6}} = \frac{4 \sqrt{3}}{5} + \frac{4 i}{5} = 1.39 + 0.8 i$
\end{enumerate}
\end{solution}
\begin{exercise}[subtitle={Sortie du congélateur}]
Marie a invité quelques amis pour le thé. Elle souhaite leur proposer ses macarons maison.
Elle les sort de son congélateur à $-16$~\degres C et les place dans une pièce à $23$~\degres C.
Au bout de 15 minutes, la température des macarons est de $0$~\degres C.
\bigskip
\textbf{Premier modèle}
\medskip
On suppose que la vitesse de décongélation est constante : chaque minute la hausse de
température des macarons est la même.
Estimer dans ce cadre la température au bout de $30$~minutes, puis au bout de $45$~minutes.
Cette modélisation est-elle pertinente?
\bigskip
\textbf{Deuxième modèle}
\medskip
On suppose maintenant que la vitesse de décongélation est proportionnelle à la différence
de température entre les macarons et l'air ambiant (il s'agit de la loi de Newton).
On désigne par $\theta$ la température des macarons à l'instant $t$, et par $\theta'$ la vitesse de décongélation.
L'unité de temps est la minute et l'unité de température le degré Celsius.
\smallskip
On négligera la diminution de température de la pièce et on admettra donc qu'il existe un
nombre réel $a$ tel que, pour $t$ positif :
\[\theta'(t) = a [\theta(t) - 23]\quad (E)\]
\medskip
\begin{enumerate}
\item Vérifier que l'équation $(E)$ a pour solutions $\theta(t) = K e^{at} + 23$$K$ est un nombre réel.
Donner alors, en fonction de $a$, l'ensemble des solutions de $(E)$.
\end{enumerate}
On rappelle que la température des macarons à l'instant $t = 0$ est égale à $-16$~\degres C et que, au bout de $15$~min, elle est de $0$~\degres C.
\begin{enumerate}
\setcounter{enumi}{1}
\item En utilisant la condition à $t=0$ démontrer que $K = -39$.
\item En utilisant la condition à $t=15$ démontrer que $a \approx -0.04$.
\item En déduire l'expression de la solution de l'équation différentielle puis étudier ses variations.
\item La température idéale de dégustation des macarons étant de $20$~\degres C, Marie estime que
celle-ci sera atteinte au bout de $30$~min. A-t-elle raison ? Justifier la réponse.
Sinon, combien de temps faudra-t-il attendre ?
\end{enumerate}
\end{exercise}
\end{document}
%%% Local Variables:
%%% mode: latex
%%% TeX-master: "master"
%%% End:

View File

@ -0,0 +1,136 @@
\documentclass[a4paper,10pt]{article}
\usepackage{myXsim}
% Title Page
\title{DS8 \hfill HUMBERT Rayan}
\tribe{TST sti2d}
\date{\hfillÀ render pour le vendredi 9 avril à 10h au plus tard}
\xsimsetup{
solution/print = true
}
\begin{document}
\maketitle
\begin{exercise}[subtitle={Étude de fonction}]
On considère la fonction $f$ définie sur $\intOF{0}{+\infty}$ par $ f(x) = 2.5x^2 + - 25x + - 180\ln(x)$
\begin{enumerate}
\item Démontrer que la dérivée de $f$ est $f'(x) = \frac{5x^2 + - 25x + - 180}{x}$.
\item Étude du numérateur de $f'(x)$: $N(x) = 5x^2 - 25x - 180$
\begin{enumerate}
\item Démontrer que $x=- 4$ et $x=9$ sont deux racines de $N(x)$..
\item Proposer une forme factorisée de $N(x)$.
\item Proposer une forme factorisée de $f'(x)$.
\end{enumerate}
\item Étudier le signe de $f'$ et en déduire les variations de $f$.
\end{enumerate}
\end{exercise}
\begin{solution}
\begin{enumerate}
\item pas de correction disponible
\item
\begin{enumerate}
\item \[N(- 4) = 0\]
\[N(9) = 0\]
\item \[
N(x) = 5(x - - 4)(x - 9)
\]
\item
\[
f'(x) = \frac{5(x - - 4)(x - 9)}{x}
\]
\end{enumerate}
\item Pas de correction disponible
\end{enumerate}
\end{solution}
\begin{exercise}[subtitle={Complexes}]
\begin{enumerate}
\item Mettre le nombre complexe suivant sous forme algébrique $z_1 = \dfrac{8 + 3 i}{-6 + 6 i} $
\item Mettre le complexe suivante sous forme exponentielle $z_2 = -4 - 4 \sqrt{3} i$
\item Mettre le complexe suivante sous forme exponentielle $z_3 = 8 \sqrt{3} + 8 i$
\item Calculer le produit $z_4=z_2\times z_3$ donner le résultat sous forme exponentielle puis algébrique.
\item Calculer le quotient $z_5=\frac{z_2}{z_3}$ donner le résultat sous forme exponentielle puis algébrique.
\end{enumerate}
\end{exercise}
\begin{solution}
\begin{enumerate}
\item $z_1 = - \frac{5}{12} - \frac{11 i}{12}$
\item $z_2 = 8 e^{- \frac{2 i \pi}{3}}$
\item $z_3 = 16 e^{\frac{i \pi}{6}}$
\item $z_4 = 128 e^{- \frac{i \pi}{2}} = - 128 i = - 128.0 i$
\item $z_5 = \frac{1}{2} e^{- \frac{5 i \pi}{6}} = - \frac{\sqrt{3}}{4} - \frac{i}{4} = -0.433 - 0.25 i$
\end{enumerate}
\end{solution}
\begin{exercise}[subtitle={Sortie du congélateur}]
Marie a invité quelques amis pour le thé. Elle souhaite leur proposer ses macarons maison.
Elle les sort de son congélateur à $-20$~\degres C et les place dans une pièce à $25$~\degres C.
Au bout de 15 minutes, la température des macarons est de $-3$~\degres C.
\bigskip
\textbf{Premier modèle}
\medskip
On suppose que la vitesse de décongélation est constante : chaque minute la hausse de
température des macarons est la même.
Estimer dans ce cadre la température au bout de $30$~minutes, puis au bout de $45$~minutes.
Cette modélisation est-elle pertinente?
\bigskip
\textbf{Deuxième modèle}
\medskip
On suppose maintenant que la vitesse de décongélation est proportionnelle à la différence
de température entre les macarons et l'air ambiant (il s'agit de la loi de Newton).
On désigne par $\theta$ la température des macarons à l'instant $t$, et par $\theta'$ la vitesse de décongélation.
L'unité de temps est la minute et l'unité de température le degré Celsius.
\smallskip
On négligera la diminution de température de la pièce et on admettra donc qu'il existe un
nombre réel $a$ tel que, pour $t$ positif :
\[\theta'(t) = a [\theta(t) - 25]\quad (E)\]
\medskip
\begin{enumerate}
\item Vérifier que l'équation $(E)$ a pour solutions $\theta(t) = K e^{at} + 25$$K$ est un nombre réel.
Donner alors, en fonction de $a$, l'ensemble des solutions de $(E)$.
\end{enumerate}
On rappelle que la température des macarons à l'instant $t = 0$ est égale à $-20$~\degres C et que, au bout de $15$~min, elle est de $-3$~\degres C.
\begin{enumerate}
\setcounter{enumi}{1}
\item En utilisant la condition à $t=0$ démontrer que $K = -45$.
\item En utilisant la condition à $t=15$ démontrer que $a \approx -0.03$.
\item En déduire l'expression de la solution de l'équation différentielle puis étudier ses variations.
\item La température idéale de dégustation des macarons étant de $22$~\degres C, Marie estime que
celle-ci sera atteinte au bout de $30$~min. A-t-elle raison ? Justifier la réponse.
Sinon, combien de temps faudra-t-il attendre ?
\end{enumerate}
\end{exercise}
\end{document}
%%% Local Variables:
%%% mode: latex
%%% TeX-master: "master"
%%% End:

View File

@ -0,0 +1,136 @@
\documentclass[a4paper,10pt]{article}
\usepackage{myXsim}
% Title Page
\title{DS8 \hfill KILINC Suleyman}
\tribe{TST sti2d}
\date{\hfillÀ render pour le vendredi 9 avril à 10h au plus tard}
\xsimsetup{
solution/print = true
}
\begin{document}
\maketitle
\begin{exercise}[subtitle={Étude de fonction}]
On considère la fonction $f$ définie sur $\intOF{0}{+\infty}$ par $ f(x) = 3.5x^2 + 21x + - 490\ln(x)$
\begin{enumerate}
\item Démontrer que la dérivée de $f$ est $f'(x) = \frac{7x^2 + 21x + - 490}{x}$.
\item Étude du numérateur de $f'(x)$: $N(x) = 7x^2 + 21x - 490$
\begin{enumerate}
\item Démontrer que $x=- 10$ et $x=7$ sont deux racines de $N(x)$..
\item Proposer une forme factorisée de $N(x)$.
\item Proposer une forme factorisée de $f'(x)$.
\end{enumerate}
\item Étudier le signe de $f'$ et en déduire les variations de $f$.
\end{enumerate}
\end{exercise}
\begin{solution}
\begin{enumerate}
\item pas de correction disponible
\item
\begin{enumerate}
\item \[N(- 10) = 0\]
\[N(7) = 0\]
\item \[
N(x) = 7(x - - 10)(x - 7)
\]
\item
\[
f'(x) = \frac{7(x - - 10)(x - 7)}{x}
\]
\end{enumerate}
\item Pas de correction disponible
\end{enumerate}
\end{solution}
\begin{exercise}[subtitle={Complexes}]
\begin{enumerate}
\item Mettre le nombre complexe suivant sous forme algébrique $z_1 = \dfrac{5 + 7 i}{-6 + 4 i} $
\item Mettre le complexe suivante sous forme exponentielle $z_2 = 1 - \sqrt{3} i$
\item Mettre le complexe suivante sous forme exponentielle $z_3 = - 8 \sqrt{2} + 8 \sqrt{2} i$
\item Calculer le produit $z_4=z_2\times z_3$ donner le résultat sous forme exponentielle puis algébrique.
\item Calculer le quotient $z_5=\frac{z_2}{z_3}$ donner le résultat sous forme exponentielle puis algébrique.
\end{enumerate}
\end{exercise}
\begin{solution}
\begin{enumerate}
\item $z_1 = - \frac{1}{26} - \frac{31 i}{26}$
\item $z_2 = 2 e^{- \frac{i \pi}{3}}$
\item $z_3 = 16 e^{\frac{3 i \pi}{4}}$
\item $z_4 = 32 e^{\frac{5 i \pi}{12}} = - 8 \sqrt{2} + 8 \sqrt{6} + i \left(8 \sqrt{2} + 8 \sqrt{6}\right) = 8.28 + 30.9 i$
\item $z_5 = \frac{1}{8} e^{- \frac{13 i \pi}{12}} = - \frac{\sqrt{6}}{32} - \frac{\sqrt{2}}{32} + i \left(- \frac{\sqrt{2}}{32} + \frac{\sqrt{6}}{32}\right) = -0.121 + 0.0323 i$
\end{enumerate}
\end{solution}
\begin{exercise}[subtitle={Sortie du congélateur}]
Marie a invité quelques amis pour le thé. Elle souhaite leur proposer ses macarons maison.
Elle les sort de son congélateur à $-19$~\degres C et les place dans une pièce à $15$~\degres C.
Au bout de 15 minutes, la température des macarons est de $-2$~\degres C.
\bigskip
\textbf{Premier modèle}
\medskip
On suppose que la vitesse de décongélation est constante : chaque minute la hausse de
température des macarons est la même.
Estimer dans ce cadre la température au bout de $30$~minutes, puis au bout de $45$~minutes.
Cette modélisation est-elle pertinente?
\bigskip
\textbf{Deuxième modèle}
\medskip
On suppose maintenant que la vitesse de décongélation est proportionnelle à la différence
de température entre les macarons et l'air ambiant (il s'agit de la loi de Newton).
On désigne par $\theta$ la température des macarons à l'instant $t$, et par $\theta'$ la vitesse de décongélation.
L'unité de temps est la minute et l'unité de température le degré Celsius.
\smallskip
On négligera la diminution de température de la pièce et on admettra donc qu'il existe un
nombre réel $a$ tel que, pour $t$ positif :
\[\theta'(t) = a [\theta(t) - 15]\quad (E)\]
\medskip
\begin{enumerate}
\item Vérifier que l'équation $(E)$ a pour solutions $\theta(t) = K e^{at} + 15$$K$ est un nombre réel.
Donner alors, en fonction de $a$, l'ensemble des solutions de $(E)$.
\end{enumerate}
On rappelle que la température des macarons à l'instant $t = 0$ est égale à $-19$~\degres C et que, au bout de $15$~min, elle est de $-2$~\degres C.
\begin{enumerate}
\setcounter{enumi}{1}
\item En utilisant la condition à $t=0$ démontrer que $K = -34$.
\item En utilisant la condition à $t=15$ démontrer que $a \approx -0.05$.
\item En déduire l'expression de la solution de l'équation différentielle puis étudier ses variations.
\item La température idéale de dégustation des macarons étant de $12$~\degres C, Marie estime que
celle-ci sera atteinte au bout de $30$~min. A-t-elle raison ? Justifier la réponse.
Sinon, combien de temps faudra-t-il attendre ?
\end{enumerate}
\end{exercise}
\end{document}
%%% Local Variables:
%%% mode: latex
%%% TeX-master: "master"
%%% End:

View File

@ -0,0 +1,136 @@
\documentclass[a4paper,10pt]{article}
\usepackage{myXsim}
% Title Page
\title{DS8 \hfill M'BAREK HASNAOUI Bilal}
\tribe{TST sti2d}
\date{\hfillÀ render pour le vendredi 9 avril à 10h au plus tard}
\xsimsetup{
solution/print = true
}
\begin{document}
\maketitle
\begin{exercise}[subtitle={Étude de fonction}]
On considère la fonction $f$ définie sur $\intOF{0}{+\infty}$ par $ f(x) = 5x^2 + - 40x + - 450\ln(x)$
\begin{enumerate}
\item Démontrer que la dérivée de $f$ est $f'(x) = \frac{10x^2 + - 40x + - 450}{x}$.
\item Étude du numérateur de $f'(x)$: $N(x) = 10x^2 - 40x - 450$
\begin{enumerate}
\item Démontrer que $x=9$ et $x=- 5$ sont deux racines de $N(x)$..
\item Proposer une forme factorisée de $N(x)$.
\item Proposer une forme factorisée de $f'(x)$.
\end{enumerate}
\item Étudier le signe de $f'$ et en déduire les variations de $f$.
\end{enumerate}
\end{exercise}
\begin{solution}
\begin{enumerate}
\item pas de correction disponible
\item
\begin{enumerate}
\item \[N(9) = 0\]
\[N(- 5) = 0\]
\item \[
N(x) = 10(x - 9)(x - - 5)
\]
\item
\[
f'(x) = \frac{10(x - 9)(x - - 5)}{x}
\]
\end{enumerate}
\item Pas de correction disponible
\end{enumerate}
\end{solution}
\begin{exercise}[subtitle={Complexes}]
\begin{enumerate}
\item Mettre le nombre complexe suivant sous forme algébrique $z_1 = \dfrac{2 + 4 i}{-5 + 7 i} $
\item Mettre le complexe suivante sous forme exponentielle $z_2 = 7 + 7 \sqrt{3} i$
\item Mettre le complexe suivante sous forme exponentielle $z_3 = 7 \sqrt{2} + 7 \sqrt{2} i$
\item Calculer le produit $z_4=z_2\times z_3$ donner le résultat sous forme exponentielle puis algébrique.
\item Calculer le quotient $z_5=\frac{z_2}{z_3}$ donner le résultat sous forme exponentielle puis algébrique.
\end{enumerate}
\end{exercise}
\begin{solution}
\begin{enumerate}
\item $z_1 = \frac{9}{37} - \frac{17 i}{37}$
\item $z_2 = 14 e^{\frac{i \pi}{3}}$
\item $z_3 = 14 e^{\frac{i \pi}{4}}$
\item $z_4 = 196 e^{\frac{7 i \pi}{12}} = - 49 \sqrt{6} + 49 \sqrt{2} + i \left(49 \sqrt{2} + 49 \sqrt{6}\right) = -50.7 + 189.0 i$
\item $z_5 = 1 e^{\frac{i \pi}{12}} = \frac{\sqrt{2}}{4} + \frac{\sqrt{6}}{4} + i \left(- \frac{\sqrt{2}}{4} + \frac{\sqrt{6}}{4}\right) = 0.966 + 0.259 i$
\end{enumerate}
\end{solution}
\begin{exercise}[subtitle={Sortie du congélateur}]
Marie a invité quelques amis pour le thé. Elle souhaite leur proposer ses macarons maison.
Elle les sort de son congélateur à $-15$~\degres C et les place dans une pièce à $25$~\degres C.
Au bout de 15 minutes, la température des macarons est de $0$~\degres C.
\bigskip
\textbf{Premier modèle}
\medskip
On suppose que la vitesse de décongélation est constante : chaque minute la hausse de
température des macarons est la même.
Estimer dans ce cadre la température au bout de $30$~minutes, puis au bout de $45$~minutes.
Cette modélisation est-elle pertinente?
\bigskip
\textbf{Deuxième modèle}
\medskip
On suppose maintenant que la vitesse de décongélation est proportionnelle à la différence
de température entre les macarons et l'air ambiant (il s'agit de la loi de Newton).
On désigne par $\theta$ la température des macarons à l'instant $t$, et par $\theta'$ la vitesse de décongélation.
L'unité de temps est la minute et l'unité de température le degré Celsius.
\smallskip
On négligera la diminution de température de la pièce et on admettra donc qu'il existe un
nombre réel $a$ tel que, pour $t$ positif :
\[\theta'(t) = a [\theta(t) - 25]\quad (E)\]
\medskip
\begin{enumerate}
\item Vérifier que l'équation $(E)$ a pour solutions $\theta(t) = K e^{at} + 25$$K$ est un nombre réel.
Donner alors, en fonction de $a$, l'ensemble des solutions de $(E)$.
\end{enumerate}
On rappelle que la température des macarons à l'instant $t = 0$ est égale à $-15$~\degres C et que, au bout de $15$~min, elle est de $0$~\degres C.
\begin{enumerate}
\setcounter{enumi}{1}
\item En utilisant la condition à $t=0$ démontrer que $K = -40$.
\item En utilisant la condition à $t=15$ démontrer que $a \approx -0.03$.
\item En déduire l'expression de la solution de l'équation différentielle puis étudier ses variations.
\item La température idéale de dégustation des macarons étant de $22$~\degres C, Marie estime que
celle-ci sera atteinte au bout de $30$~min. A-t-elle raison ? Justifier la réponse.
Sinon, combien de temps faudra-t-il attendre ?
\end{enumerate}
\end{exercise}
\end{document}
%%% Local Variables:
%%% mode: latex
%%% TeX-master: "master"
%%% End:

View File

@ -0,0 +1,136 @@
\documentclass[a4paper,10pt]{article}
\usepackage{myXsim}
% Title Page
\title{DS8 \hfill MERCIER Almandin}
\tribe{TST sti2d}
\date{\hfillÀ render pour le vendredi 9 avril à 10h au plus tard}
\xsimsetup{
solution/print = true
}
\begin{document}
\maketitle
\begin{exercise}[subtitle={Étude de fonction}]
On considère la fonction $f$ définie sur $\intOF{0}{+\infty}$ par $ f(x) = 2.5x^2 + - 50x + 120\ln(x)$
\begin{enumerate}
\item Démontrer que la dérivée de $f$ est $f'(x) = \frac{5x^2 + - 50x + 120}{x}$.
\item Étude du numérateur de $f'(x)$: $N(x) = 5x^2 - 50x + 120$
\begin{enumerate}
\item Démontrer que $x=6$ et $x=4$ sont deux racines de $N(x)$..
\item Proposer une forme factorisée de $N(x)$.
\item Proposer une forme factorisée de $f'(x)$.
\end{enumerate}
\item Étudier le signe de $f'$ et en déduire les variations de $f$.
\end{enumerate}
\end{exercise}
\begin{solution}
\begin{enumerate}
\item pas de correction disponible
\item
\begin{enumerate}
\item \[N(6) = 0\]
\[N(4) = 0\]
\item \[
N(x) = 5(x - 6)(x - 4)
\]
\item
\[
f'(x) = \frac{5(x - 6)(x - 4)}{x}
\]
\end{enumerate}
\item Pas de correction disponible
\end{enumerate}
\end{solution}
\begin{exercise}[subtitle={Complexes}]
\begin{enumerate}
\item Mettre le nombre complexe suivant sous forme algébrique $z_1 = \dfrac{10 + 10 i}{-2 + 3 i} $
\item Mettre le complexe suivante sous forme exponentielle $z_2 = - 7 \sqrt{2} - 7 \sqrt{2} i$
\item Mettre le complexe suivante sous forme exponentielle $z_3 = - 9 \sqrt{2} + 9 \sqrt{2} i$
\item Calculer le produit $z_4=z_2\times z_3$ donner le résultat sous forme exponentielle puis algébrique.
\item Calculer le quotient $z_5=\frac{z_2}{z_3}$ donner le résultat sous forme exponentielle puis algébrique.
\end{enumerate}
\end{exercise}
\begin{solution}
\begin{enumerate}
\item $z_1 = \frac{10}{13} - \frac{50 i}{13}$
\item $z_2 = 14 e^{- \frac{3 i \pi}{4}}$
\item $z_3 = 18 e^{\frac{3 i \pi}{4}}$
\item $z_4 = 252 e^{0} = 252 = 252.0$
\item $z_5 = \frac{7}{9} e^{- \frac{3 i \pi}{2}} = \frac{7 i}{9} = 0.778 i$
\end{enumerate}
\end{solution}
\begin{exercise}[subtitle={Sortie du congélateur}]
Marie a invité quelques amis pour le thé. Elle souhaite leur proposer ses macarons maison.
Elle les sort de son congélateur à $-16$~\degres C et les place dans une pièce à $22$~\degres C.
Au bout de 15 minutes, la température des macarons est de $3$~\degres C.
\bigskip
\textbf{Premier modèle}
\medskip
On suppose que la vitesse de décongélation est constante : chaque minute la hausse de
température des macarons est la même.
Estimer dans ce cadre la température au bout de $30$~minutes, puis au bout de $45$~minutes.
Cette modélisation est-elle pertinente?
\bigskip
\textbf{Deuxième modèle}
\medskip
On suppose maintenant que la vitesse de décongélation est proportionnelle à la différence
de température entre les macarons et l'air ambiant (il s'agit de la loi de Newton).
On désigne par $\theta$ la température des macarons à l'instant $t$, et par $\theta'$ la vitesse de décongélation.
L'unité de temps est la minute et l'unité de température le degré Celsius.
\smallskip
On négligera la diminution de température de la pièce et on admettra donc qu'il existe un
nombre réel $a$ tel que, pour $t$ positif :
\[\theta'(t) = a [\theta(t) - 22]\quad (E)\]
\medskip
\begin{enumerate}
\item Vérifier que l'équation $(E)$ a pour solutions $\theta(t) = K e^{at} + 22$$K$ est un nombre réel.
Donner alors, en fonction de $a$, l'ensemble des solutions de $(E)$.
\end{enumerate}
On rappelle que la température des macarons à l'instant $t = 0$ est égale à $-16$~\degres C et que, au bout de $15$~min, elle est de $3$~\degres C.
\begin{enumerate}
\setcounter{enumi}{1}
\item En utilisant la condition à $t=0$ démontrer que $K = -38$.
\item En utilisant la condition à $t=15$ démontrer que $a \approx -0.05$.
\item En déduire l'expression de la solution de l'équation différentielle puis étudier ses variations.
\item La température idéale de dégustation des macarons étant de $19$~\degres C, Marie estime que
celle-ci sera atteinte au bout de $30$~min. A-t-elle raison ? Justifier la réponse.
Sinon, combien de temps faudra-t-il attendre ?
\end{enumerate}
\end{exercise}
\end{document}
%%% Local Variables:
%%% mode: latex
%%% TeX-master: "master"
%%% End:

View File

@ -0,0 +1,136 @@
\documentclass[a4paper,10pt]{article}
\usepackage{myXsim}
% Title Page
\title{DS8 \hfill MOUFAQ Amine}
\tribe{TST sti2d}
\date{\hfillÀ render pour le vendredi 9 avril à 10h au plus tard}
\xsimsetup{
solution/print = true
}
\begin{document}
\maketitle
\begin{exercise}[subtitle={Étude de fonction}]
On considère la fonction $f$ définie sur $\intOF{0}{+\infty}$ par $ f(x) = 5x^2 + 0x + - 810\ln(x)$
\begin{enumerate}
\item Démontrer que la dérivée de $f$ est $f'(x) = \frac{10x^2 + 0x + - 810}{x}$.
\item Étude du numérateur de $f'(x)$: $N(x) = 10x^2 - 810$
\begin{enumerate}
\item Démontrer que $x=- 9$ et $x=9$ sont deux racines de $N(x)$..
\item Proposer une forme factorisée de $N(x)$.
\item Proposer une forme factorisée de $f'(x)$.
\end{enumerate}
\item Étudier le signe de $f'$ et en déduire les variations de $f$.
\end{enumerate}
\end{exercise}
\begin{solution}
\begin{enumerate}
\item pas de correction disponible
\item
\begin{enumerate}
\item \[N(- 9) = 0\]
\[N(9) = 0\]
\item \[
N(x) = 10(x - - 9)(x - 9)
\]
\item
\[
f'(x) = \frac{10(x - - 9)(x - 9)}{x}
\]
\end{enumerate}
\item Pas de correction disponible
\end{enumerate}
\end{solution}
\begin{exercise}[subtitle={Complexes}]
\begin{enumerate}
\item Mettre le nombre complexe suivant sous forme algébrique $z_1 = \dfrac{8 + 4 i}{-3 + 5 i} $
\item Mettre le complexe suivante sous forme exponentielle $z_2 = - 2 \sqrt{2} + 2 \sqrt{2} i$
\item Mettre le complexe suivante sous forme exponentielle $z_3 = 6 + 6 \sqrt{3} i$
\item Calculer le produit $z_4=z_2\times z_3$ donner le résultat sous forme exponentielle puis algébrique.
\item Calculer le quotient $z_5=\frac{z_2}{z_3}$ donner le résultat sous forme exponentielle puis algébrique.
\end{enumerate}
\end{exercise}
\begin{solution}
\begin{enumerate}
\item $z_1 = - \frac{2}{17} - \frac{26 i}{17}$
\item $z_2 = 4 e^{\frac{3 i \pi}{4}}$
\item $z_3 = 12 e^{\frac{i \pi}{3}}$
\item $z_4 = 48 e^{\frac{13 i \pi}{12}} = - 12 \sqrt{6} - 12 \sqrt{2} + i \left(- 12 \sqrt{6} + 12 \sqrt{2}\right) = -46.4 - 12.4 i$
\item $z_5 = \frac{1}{3} e^{\frac{5 i \pi}{12}} = - \frac{\sqrt{2}}{12} + \frac{\sqrt{6}}{12} + i \left(\frac{\sqrt{2}}{12} + \frac{\sqrt{6}}{12}\right) = 0.0863 + 0.322 i$
\end{enumerate}
\end{solution}
\begin{exercise}[subtitle={Sortie du congélateur}]
Marie a invité quelques amis pour le thé. Elle souhaite leur proposer ses macarons maison.
Elle les sort de son congélateur à $-15$~\degres C et les place dans une pièce à $20$~\degres C.
Au bout de 15 minutes, la température des macarons est de $-3$~\degres C.
\bigskip
\textbf{Premier modèle}
\medskip
On suppose que la vitesse de décongélation est constante : chaque minute la hausse de
température des macarons est la même.
Estimer dans ce cadre la température au bout de $30$~minutes, puis au bout de $45$~minutes.
Cette modélisation est-elle pertinente?
\bigskip
\textbf{Deuxième modèle}
\medskip
On suppose maintenant que la vitesse de décongélation est proportionnelle à la différence
de température entre les macarons et l'air ambiant (il s'agit de la loi de Newton).
On désigne par $\theta$ la température des macarons à l'instant $t$, et par $\theta'$ la vitesse de décongélation.
L'unité de temps est la minute et l'unité de température le degré Celsius.
\smallskip
On négligera la diminution de température de la pièce et on admettra donc qu'il existe un
nombre réel $a$ tel que, pour $t$ positif :
\[\theta'(t) = a [\theta(t) - 20]\quad (E)\]
\medskip
\begin{enumerate}
\item Vérifier que l'équation $(E)$ a pour solutions $\theta(t) = K e^{at} + 20$$K$ est un nombre réel.
Donner alors, en fonction de $a$, l'ensemble des solutions de $(E)$.
\end{enumerate}
On rappelle que la température des macarons à l'instant $t = 0$ est égale à $-15$~\degres C et que, au bout de $15$~min, elle est de $-3$~\degres C.
\begin{enumerate}
\setcounter{enumi}{1}
\item En utilisant la condition à $t=0$ démontrer que $K = -35$.
\item En utilisant la condition à $t=15$ démontrer que $a \approx -0.03$.
\item En déduire l'expression de la solution de l'équation différentielle puis étudier ses variations.
\item La température idéale de dégustation des macarons étant de $17$~\degres C, Marie estime que
celle-ci sera atteinte au bout de $30$~min. A-t-elle raison ? Justifier la réponse.
Sinon, combien de temps faudra-t-il attendre ?
\end{enumerate}
\end{exercise}
\end{document}
%%% Local Variables:
%%% mode: latex
%%% TeX-master: "master"
%%% End:

View File

@ -0,0 +1,136 @@
\documentclass[a4paper,10pt]{article}
\usepackage{myXsim}
% Title Page
\title{DS8 \hfill NARDINI Kakary}
\tribe{TST sti2d}
\date{\hfillÀ render pour le vendredi 9 avril à 10h au plus tard}
\xsimsetup{
solution/print = true
}
\begin{document}
\maketitle
\begin{exercise}[subtitle={Étude de fonction}]
On considère la fonction $f$ définie sur $\intOF{0}{+\infty}$ par $ f(x) = 3x^2 + - 18x + - 108\ln(x)$
\begin{enumerate}
\item Démontrer que la dérivée de $f$ est $f'(x) = \frac{6x^2 + - 18x + - 108}{x}$.
\item Étude du numérateur de $f'(x)$: $N(x) = 6x^2 - 18x - 108$
\begin{enumerate}
\item Démontrer que $x=6$ et $x=- 3$ sont deux racines de $N(x)$..
\item Proposer une forme factorisée de $N(x)$.
\item Proposer une forme factorisée de $f'(x)$.
\end{enumerate}
\item Étudier le signe de $f'$ et en déduire les variations de $f$.
\end{enumerate}
\end{exercise}
\begin{solution}
\begin{enumerate}
\item pas de correction disponible
\item
\begin{enumerate}
\item \[N(6) = 0\]
\[N(- 3) = 0\]
\item \[
N(x) = 6(x - 6)(x - - 3)
\]
\item
\[
f'(x) = \frac{6(x - 6)(x - - 3)}{x}
\]
\end{enumerate}
\item Pas de correction disponible
\end{enumerate}
\end{solution}
\begin{exercise}[subtitle={Complexes}]
\begin{enumerate}
\item Mettre le nombre complexe suivant sous forme algébrique $z_1 = \dfrac{9 + 6 i}{-3 + 6 i} $
\item Mettre le complexe suivante sous forme exponentielle $z_2 = -4 - 4 \sqrt{3} i$
\item Mettre le complexe suivante sous forme exponentielle $z_3 = 9 \sqrt{2} + 9 \sqrt{2} i$
\item Calculer le produit $z_4=z_2\times z_3$ donner le résultat sous forme exponentielle puis algébrique.
\item Calculer le quotient $z_5=\frac{z_2}{z_3}$ donner le résultat sous forme exponentielle puis algébrique.
\end{enumerate}
\end{exercise}
\begin{solution}
\begin{enumerate}
\item $z_1 = \frac{1}{5} - \frac{8 i}{5}$
\item $z_2 = 8 e^{- \frac{2 i \pi}{3}}$
\item $z_3 = 18 e^{\frac{i \pi}{4}}$
\item $z_4 = 144 e^{- \frac{5 i \pi}{12}} = - 36 \sqrt{2} + 36 \sqrt{6} + i \left(- 36 \sqrt{6} - 36 \sqrt{2}\right) = 37.3 - 139.0 i$
\item $z_5 = \frac{4}{9} e^{- \frac{11 i \pi}{12}} = - \frac{\sqrt{6}}{9} - \frac{\sqrt{2}}{9} + i \left(- \frac{\sqrt{6}}{9} + \frac{\sqrt{2}}{9}\right) = -0.429 - 0.115 i$
\end{enumerate}
\end{solution}
\begin{exercise}[subtitle={Sortie du congélateur}]
Marie a invité quelques amis pour le thé. Elle souhaite leur proposer ses macarons maison.
Elle les sort de son congélateur à $-18$~\degres C et les place dans une pièce à $17$~\degres C.
Au bout de 15 minutes, la température des macarons est de $2$~\degres C.
\bigskip
\textbf{Premier modèle}
\medskip
On suppose que la vitesse de décongélation est constante : chaque minute la hausse de
température des macarons est la même.
Estimer dans ce cadre la température au bout de $30$~minutes, puis au bout de $45$~minutes.
Cette modélisation est-elle pertinente?
\bigskip
\textbf{Deuxième modèle}
\medskip
On suppose maintenant que la vitesse de décongélation est proportionnelle à la différence
de température entre les macarons et l'air ambiant (il s'agit de la loi de Newton).
On désigne par $\theta$ la température des macarons à l'instant $t$, et par $\theta'$ la vitesse de décongélation.
L'unité de temps est la minute et l'unité de température le degré Celsius.
\smallskip
On négligera la diminution de température de la pièce et on admettra donc qu'il existe un
nombre réel $a$ tel que, pour $t$ positif :
\[\theta'(t) = a [\theta(t) - 17]\quad (E)\]
\medskip
\begin{enumerate}
\item Vérifier que l'équation $(E)$ a pour solutions $\theta(t) = K e^{at} + 17$$K$ est un nombre réel.
Donner alors, en fonction de $a$, l'ensemble des solutions de $(E)$.
\end{enumerate}
On rappelle que la température des macarons à l'instant $t = 0$ est égale à $-18$~\degres C et que, au bout de $15$~min, elle est de $2$~\degres C.
\begin{enumerate}
\setcounter{enumi}{1}
\item En utilisant la condition à $t=0$ démontrer que $K = -35$.
\item En utilisant la condition à $t=15$ démontrer que $a \approx -0.06$.
\item En déduire l'expression de la solution de l'équation différentielle puis étudier ses variations.
\item La température idéale de dégustation des macarons étant de $14$~\degres C, Marie estime que
celle-ci sera atteinte au bout de $30$~min. A-t-elle raison ? Justifier la réponse.
Sinon, combien de temps faudra-t-il attendre ?
\end{enumerate}
\end{exercise}
\end{document}
%%% Local Variables:
%%% mode: latex
%%% TeX-master: "master"
%%% End:

View File

@ -0,0 +1,136 @@
\documentclass[a4paper,10pt]{article}
\usepackage{myXsim}
% Title Page
\title{DS8 \hfill ONAL Yakub}
\tribe{TST sti2d}
\date{\hfillÀ render pour le vendredi 9 avril à 10h au plus tard}
\xsimsetup{
solution/print = true
}
\begin{document}
\maketitle
\begin{exercise}[subtitle={Étude de fonction}]
On considère la fonction $f$ définie sur $\intOF{0}{+\infty}$ par $ f(x) = 5x^2 + - 40x + - 450\ln(x)$
\begin{enumerate}
\item Démontrer que la dérivée de $f$ est $f'(x) = \frac{10x^2 + - 40x + - 450}{x}$.
\item Étude du numérateur de $f'(x)$: $N(x) = 10x^2 - 40x - 450$
\begin{enumerate}
\item Démontrer que $x=9$ et $x=- 5$ sont deux racines de $N(x)$..
\item Proposer une forme factorisée de $N(x)$.
\item Proposer une forme factorisée de $f'(x)$.
\end{enumerate}
\item Étudier le signe de $f'$ et en déduire les variations de $f$.
\end{enumerate}
\end{exercise}
\begin{solution}
\begin{enumerate}
\item pas de correction disponible
\item
\begin{enumerate}
\item \[N(9) = 0\]
\[N(- 5) = 0\]
\item \[
N(x) = 10(x - 9)(x - - 5)
\]
\item
\[
f'(x) = \frac{10(x - 9)(x - - 5)}{x}
\]
\end{enumerate}
\item Pas de correction disponible
\end{enumerate}
\end{solution}
\begin{exercise}[subtitle={Complexes}]
\begin{enumerate}
\item Mettre le nombre complexe suivant sous forme algébrique $z_1 = \dfrac{5 + 2 i}{-10 + 2 i} $
\item Mettre le complexe suivante sous forme exponentielle $z_2 = 6 - 6 \sqrt{3} i$
\item Mettre le complexe suivante sous forme exponentielle $z_3 = 10 \sqrt{3} + 10 i$
\item Calculer le produit $z_4=z_2\times z_3$ donner le résultat sous forme exponentielle puis algébrique.
\item Calculer le quotient $z_5=\frac{z_2}{z_3}$ donner le résultat sous forme exponentielle puis algébrique.
\end{enumerate}
\end{exercise}
\begin{solution}
\begin{enumerate}
\item $z_1 = - \frac{23}{52} - \frac{15 i}{52}$
\item $z_2 = 12 e^{- \frac{i \pi}{3}}$
\item $z_3 = 20 e^{\frac{i \pi}{6}}$
\item $z_4 = 240 e^{- \frac{i \pi}{6}} = 120 \sqrt{3} - 120 i = 208.0 - 120.0 i$
\item $z_5 = \frac{3}{5} e^{- \frac{i \pi}{2}} = - \frac{3 i}{5} = - 0.6 i$
\end{enumerate}
\end{solution}
\begin{exercise}[subtitle={Sortie du congélateur}]
Marie a invité quelques amis pour le thé. Elle souhaite leur proposer ses macarons maison.
Elle les sort de son congélateur à $-18$~\degres C et les place dans une pièce à $19$~\degres C.
Au bout de 15 minutes, la température des macarons est de $-2$~\degres C.
\bigskip
\textbf{Premier modèle}
\medskip
On suppose que la vitesse de décongélation est constante : chaque minute la hausse de
température des macarons est la même.
Estimer dans ce cadre la température au bout de $30$~minutes, puis au bout de $45$~minutes.
Cette modélisation est-elle pertinente?
\bigskip
\textbf{Deuxième modèle}
\medskip
On suppose maintenant que la vitesse de décongélation est proportionnelle à la différence
de température entre les macarons et l'air ambiant (il s'agit de la loi de Newton).
On désigne par $\theta$ la température des macarons à l'instant $t$, et par $\theta'$ la vitesse de décongélation.
L'unité de temps est la minute et l'unité de température le degré Celsius.
\smallskip
On négligera la diminution de température de la pièce et on admettra donc qu'il existe un
nombre réel $a$ tel que, pour $t$ positif :
\[\theta'(t) = a [\theta(t) - 19]\quad (E)\]
\medskip
\begin{enumerate}
\item Vérifier que l'équation $(E)$ a pour solutions $\theta(t) = K e^{at} + 19$$K$ est un nombre réel.
Donner alors, en fonction de $a$, l'ensemble des solutions de $(E)$.
\end{enumerate}
On rappelle que la température des macarons à l'instant $t = 0$ est égale à $-18$~\degres C et que, au bout de $15$~min, elle est de $-2$~\degres C.
\begin{enumerate}
\setcounter{enumi}{1}
\item En utilisant la condition à $t=0$ démontrer que $K = -37$.
\item En utilisant la condition à $t=15$ démontrer que $a \approx -0.04$.
\item En déduire l'expression de la solution de l'équation différentielle puis étudier ses variations.
\item La température idéale de dégustation des macarons étant de $16$~\degres C, Marie estime que
celle-ci sera atteinte au bout de $30$~min. A-t-elle raison ? Justifier la réponse.
Sinon, combien de temps faudra-t-il attendre ?
\end{enumerate}
\end{exercise}
\end{document}
%%% Local Variables:
%%% mode: latex
%%% TeX-master: "master"
%%% End:

View File

@ -0,0 +1,136 @@
\documentclass[a4paper,10pt]{article}
\usepackage{myXsim}
% Title Page
\title{DS8 \hfill RADOUAA Saleh}
\tribe{TST sti2d}
\date{\hfillÀ render pour le vendredi 9 avril à 10h au plus tard}
\xsimsetup{
solution/print = true
}
\begin{document}
\maketitle
\begin{exercise}[subtitle={Étude de fonction}]
On considère la fonction $f$ définie sur $\intOF{0}{+\infty}$ par $ f(x) = 3x^2 + - 54x + 108\ln(x)$
\begin{enumerate}
\item Démontrer que la dérivée de $f$ est $f'(x) = \frac{6x^2 + - 54x + 108}{x}$.
\item Étude du numérateur de $f'(x)$: $N(x) = 6x^2 - 54x + 108$
\begin{enumerate}
\item Démontrer que $x=3$ et $x=6$ sont deux racines de $N(x)$..
\item Proposer une forme factorisée de $N(x)$.
\item Proposer une forme factorisée de $f'(x)$.
\end{enumerate}
\item Étudier le signe de $f'$ et en déduire les variations de $f$.
\end{enumerate}
\end{exercise}
\begin{solution}
\begin{enumerate}
\item pas de correction disponible
\item
\begin{enumerate}
\item \[N(3) = 0\]
\[N(6) = 0\]
\item \[
N(x) = 6(x - 3)(x - 6)
\]
\item
\[
f'(x) = \frac{6(x - 3)(x - 6)}{x}
\]
\end{enumerate}
\item Pas de correction disponible
\end{enumerate}
\end{solution}
\begin{exercise}[subtitle={Complexes}]
\begin{enumerate}
\item Mettre le nombre complexe suivant sous forme algébrique $z_1 = \dfrac{7 + 9 i}{-3 + 7 i} $
\item Mettre le complexe suivante sous forme exponentielle $z_2 = -6 + 6 \sqrt{3} i$
\item Mettre le complexe suivante sous forme exponentielle $z_3 = 10 + 10 \sqrt{3} i$
\item Calculer le produit $z_4=z_2\times z_3$ donner le résultat sous forme exponentielle puis algébrique.
\item Calculer le quotient $z_5=\frac{z_2}{z_3}$ donner le résultat sous forme exponentielle puis algébrique.
\end{enumerate}
\end{exercise}
\begin{solution}
\begin{enumerate}
\item $z_1 = \frac{21}{29} - \frac{38 i}{29}$
\item $z_2 = 12 e^{\frac{2 i \pi}{3}}$
\item $z_3 = 20 e^{\frac{i \pi}{3}}$
\item $z_4 = 240 e^{i \pi} = -240 = -240.0$
\item $z_5 = \frac{3}{5} e^{\frac{i \pi}{3}} = \frac{3}{10} + \frac{3 \sqrt{3} i}{10} = 0.3 + 0.52 i$
\end{enumerate}
\end{solution}
\begin{exercise}[subtitle={Sortie du congélateur}]
Marie a invité quelques amis pour le thé. Elle souhaite leur proposer ses macarons maison.
Elle les sort de son congélateur à $-20$~\degres C et les place dans une pièce à $16$~\degres C.
Au bout de 15 minutes, la température des macarons est de $4$~\degres C.
\bigskip
\textbf{Premier modèle}
\medskip
On suppose que la vitesse de décongélation est constante : chaque minute la hausse de
température des macarons est la même.
Estimer dans ce cadre la température au bout de $30$~minutes, puis au bout de $45$~minutes.
Cette modélisation est-elle pertinente?
\bigskip
\textbf{Deuxième modèle}
\medskip
On suppose maintenant que la vitesse de décongélation est proportionnelle à la différence
de température entre les macarons et l'air ambiant (il s'agit de la loi de Newton).
On désigne par $\theta$ la température des macarons à l'instant $t$, et par $\theta'$ la vitesse de décongélation.
L'unité de temps est la minute et l'unité de température le degré Celsius.
\smallskip
On négligera la diminution de température de la pièce et on admettra donc qu'il existe un
nombre réel $a$ tel que, pour $t$ positif :
\[\theta'(t) = a [\theta(t) - 16]\quad (E)\]
\medskip
\begin{enumerate}
\item Vérifier que l'équation $(E)$ a pour solutions $\theta(t) = K e^{at} + 16$$K$ est un nombre réel.
Donner alors, en fonction de $a$, l'ensemble des solutions de $(E)$.
\end{enumerate}
On rappelle que la température des macarons à l'instant $t = 0$ est égale à $-20$~\degres C et que, au bout de $15$~min, elle est de $4$~\degres C.
\begin{enumerate}
\setcounter{enumi}{1}
\item En utilisant la condition à $t=0$ démontrer que $K = -36$.
\item En utilisant la condition à $t=15$ démontrer que $a \approx -0.07$.
\item En déduire l'expression de la solution de l'équation différentielle puis étudier ses variations.
\item La température idéale de dégustation des macarons étant de $13$~\degres C, Marie estime que
celle-ci sera atteinte au bout de $30$~min. A-t-elle raison ? Justifier la réponse.
Sinon, combien de temps faudra-t-il attendre ?
\end{enumerate}
\end{exercise}
\end{document}
%%% Local Variables:
%%% mode: latex
%%% TeX-master: "master"
%%% End:

View File

@ -0,0 +1,136 @@
\documentclass[a4paper,10pt]{article}
\usepackage{myXsim}
% Title Page
\title{DS8 \hfill TAVERNIER Joanny}
\tribe{TST sti2d}
\date{\hfillÀ render pour le vendredi 9 avril à 10h au plus tard}
\xsimsetup{
solution/print = true
}
\begin{document}
\maketitle
\begin{exercise}[subtitle={Étude de fonction}]
On considère la fonction $f$ définie sur $\intOF{0}{+\infty}$ par $ f(x) = 4.5x^2 + - 144x + 567\ln(x)$
\begin{enumerate}
\item Démontrer que la dérivée de $f$ est $f'(x) = \frac{9x^2 + - 144x + 567}{x}$.
\item Étude du numérateur de $f'(x)$: $N(x) = 9x^2 - 144x + 567$
\begin{enumerate}
\item Démontrer que $x=9$ et $x=7$ sont deux racines de $N(x)$..
\item Proposer une forme factorisée de $N(x)$.
\item Proposer une forme factorisée de $f'(x)$.
\end{enumerate}
\item Étudier le signe de $f'$ et en déduire les variations de $f$.
\end{enumerate}
\end{exercise}
\begin{solution}
\begin{enumerate}
\item pas de correction disponible
\item
\begin{enumerate}
\item \[N(9) = 0\]
\[N(7) = 0\]
\item \[
N(x) = 9(x - 9)(x - 7)
\]
\item
\[
f'(x) = \frac{9(x - 9)(x - 7)}{x}
\]
\end{enumerate}
\item Pas de correction disponible
\end{enumerate}
\end{solution}
\begin{exercise}[subtitle={Complexes}]
\begin{enumerate}
\item Mettre le nombre complexe suivant sous forme algébrique $z_1 = \dfrac{10 + 6 i}{-4 + 5 i} $
\item Mettre le complexe suivante sous forme exponentielle $z_2 = - 8 \sqrt{3} + 8 i$
\item Mettre le complexe suivante sous forme exponentielle $z_3 = 5 - 5 \sqrt{3} i$
\item Calculer le produit $z_4=z_2\times z_3$ donner le résultat sous forme exponentielle puis algébrique.
\item Calculer le quotient $z_5=\frac{z_2}{z_3}$ donner le résultat sous forme exponentielle puis algébrique.
\end{enumerate}
\end{exercise}
\begin{solution}
\begin{enumerate}
\item $z_1 = - \frac{10}{41} - \frac{74 i}{41}$
\item $z_2 = 16 e^{\frac{5 i \pi}{6}}$
\item $z_3 = 10 e^{- \frac{i \pi}{3}}$
\item $z_4 = 160 e^{\frac{i \pi}{2}} = 160 i = 160.0 i$
\item $z_5 = \frac{8}{5} e^{\frac{7 i \pi}{6}} = - \frac{4 \sqrt{3}}{5} - \frac{4 i}{5} = -1.39 - 0.8 i$
\end{enumerate}
\end{solution}
\begin{exercise}[subtitle={Sortie du congélateur}]
Marie a invité quelques amis pour le thé. Elle souhaite leur proposer ses macarons maison.
Elle les sort de son congélateur à $-19$~\degres C et les place dans une pièce à $21$~\degres C.
Au bout de 15 minutes, la température des macarons est de $-4$~\degres C.
\bigskip
\textbf{Premier modèle}
\medskip
On suppose que la vitesse de décongélation est constante : chaque minute la hausse de
température des macarons est la même.
Estimer dans ce cadre la température au bout de $30$~minutes, puis au bout de $45$~minutes.
Cette modélisation est-elle pertinente?
\bigskip
\textbf{Deuxième modèle}
\medskip
On suppose maintenant que la vitesse de décongélation est proportionnelle à la différence
de température entre les macarons et l'air ambiant (il s'agit de la loi de Newton).
On désigne par $\theta$ la température des macarons à l'instant $t$, et par $\theta'$ la vitesse de décongélation.
L'unité de temps est la minute et l'unité de température le degré Celsius.
\smallskip
On négligera la diminution de température de la pièce et on admettra donc qu'il existe un
nombre réel $a$ tel que, pour $t$ positif :
\[\theta'(t) = a [\theta(t) - 21]\quad (E)\]
\medskip
\begin{enumerate}
\item Vérifier que l'équation $(E)$ a pour solutions $\theta(t) = K e^{at} + 21$$K$ est un nombre réel.
Donner alors, en fonction de $a$, l'ensemble des solutions de $(E)$.
\end{enumerate}
On rappelle que la température des macarons à l'instant $t = 0$ est égale à $-19$~\degres C et que, au bout de $15$~min, elle est de $-4$~\degres C.
\begin{enumerate}
\setcounter{enumi}{1}
\item En utilisant la condition à $t=0$ démontrer que $K = -40$.
\item En utilisant la condition à $t=15$ démontrer que $a \approx -0.03$.
\item En déduire l'expression de la solution de l'équation différentielle puis étudier ses variations.
\item La température idéale de dégustation des macarons étant de $18$~\degres C, Marie estime que
celle-ci sera atteinte au bout de $30$~min. A-t-elle raison ? Justifier la réponse.
Sinon, combien de temps faudra-t-il attendre ?
\end{enumerate}
\end{exercise}
\end{document}
%%% Local Variables:
%%% mode: latex
%%% TeX-master: "master"
%%% End:

View File

@ -0,0 +1,136 @@
\documentclass[a4paper,10pt]{article}
\usepackage{myXsim}
% Title Page
\title{DS8 \hfill ZAHORE Zahiri}
\tribe{TST sti2d}
\date{\hfillÀ render pour le vendredi 9 avril à 10h au plus tard}
\xsimsetup{
solution/print = true
}
\begin{document}
\maketitle
\begin{exercise}[subtitle={Étude de fonction}]
On considère la fonction $f$ définie sur $\intOF{0}{+\infty}$ par $ f(x) = 4.5x^2 + - 54x + - 63\ln(x)$
\begin{enumerate}
\item Démontrer que la dérivée de $f$ est $f'(x) = \frac{9x^2 + - 54x + - 63}{x}$.
\item Étude du numérateur de $f'(x)$: $N(x) = 9x^2 - 54x - 63$
\begin{enumerate}
\item Démontrer que $x=7$ et $x=- 1$ sont deux racines de $N(x)$..
\item Proposer une forme factorisée de $N(x)$.
\item Proposer une forme factorisée de $f'(x)$.
\end{enumerate}
\item Étudier le signe de $f'$ et en déduire les variations de $f$.
\end{enumerate}
\end{exercise}
\begin{solution}
\begin{enumerate}
\item pas de correction disponible
\item
\begin{enumerate}
\item \[N(7) = 0\]
\[N(- 1) = 0\]
\item \[
N(x) = 9(x - 7)(x - - 1)
\]
\item
\[
f'(x) = \frac{9(x - 7)(x - - 1)}{x}
\]
\end{enumerate}
\item Pas de correction disponible
\end{enumerate}
\end{solution}
\begin{exercise}[subtitle={Complexes}]
\begin{enumerate}
\item Mettre le nombre complexe suivant sous forme algébrique $z_1 = \dfrac{7 + 4 i}{-4 + 8 i} $
\item Mettre le complexe suivante sous forme exponentielle $z_2 = 4 \sqrt{2} + 4 \sqrt{2} i$
\item Mettre le complexe suivante sous forme exponentielle $z_3 = - 4 \sqrt{2} - 4 \sqrt{2} i$
\item Calculer le produit $z_4=z_2\times z_3$ donner le résultat sous forme exponentielle puis algébrique.
\item Calculer le quotient $z_5=\frac{z_2}{z_3}$ donner le résultat sous forme exponentielle puis algébrique.
\end{enumerate}
\end{exercise}
\begin{solution}
\begin{enumerate}
\item $z_1 = \frac{1}{20} - \frac{9 i}{10}$
\item $z_2 = 8 e^{\frac{i \pi}{4}}$
\item $z_3 = 8 e^{- \frac{3 i \pi}{4}}$
\item $z_4 = 64 e^{- \frac{i \pi}{2}} = - 64 i = - 64.0 i$
\item $z_5 = 1 e^{i \pi} = -1 = -1.0$
\end{enumerate}
\end{solution}
\begin{exercise}[subtitle={Sortie du congélateur}]
Marie a invité quelques amis pour le thé. Elle souhaite leur proposer ses macarons maison.
Elle les sort de son congélateur à $-18$~\degres C et les place dans une pièce à $21$~\degres C.
Au bout de 15 minutes, la température des macarons est de $4$~\degres C.
\bigskip
\textbf{Premier modèle}
\medskip
On suppose que la vitesse de décongélation est constante : chaque minute la hausse de
température des macarons est la même.
Estimer dans ce cadre la température au bout de $30$~minutes, puis au bout de $45$~minutes.
Cette modélisation est-elle pertinente?
\bigskip
\textbf{Deuxième modèle}
\medskip
On suppose maintenant que la vitesse de décongélation est proportionnelle à la différence
de température entre les macarons et l'air ambiant (il s'agit de la loi de Newton).
On désigne par $\theta$ la température des macarons à l'instant $t$, et par $\theta'$ la vitesse de décongélation.
L'unité de temps est la minute et l'unité de température le degré Celsius.
\smallskip
On négligera la diminution de température de la pièce et on admettra donc qu'il existe un
nombre réel $a$ tel que, pour $t$ positif :
\[\theta'(t) = a [\theta(t) - 21]\quad (E)\]
\medskip
\begin{enumerate}
\item Vérifier que l'équation $(E)$ a pour solutions $\theta(t) = K e^{at} + 21$$K$ est un nombre réel.
Donner alors, en fonction de $a$, l'ensemble des solutions de $(E)$.
\end{enumerate}
On rappelle que la température des macarons à l'instant $t = 0$ est égale à $-18$~\degres C et que, au bout de $15$~min, elle est de $4$~\degres C.
\begin{enumerate}
\setcounter{enumi}{1}
\item En utilisant la condition à $t=0$ démontrer que $K = -39$.
\item En utilisant la condition à $t=15$ démontrer que $a \approx -0.06$.
\item En déduire l'expression de la solution de l'équation différentielle puis étudier ses variations.
\item La température idéale de dégustation des macarons étant de $18$~\degres C, Marie estime que
celle-ci sera atteinte au bout de $30$~min. A-t-elle raison ? Justifier la réponse.
Sinon, combien de temps faudra-t-il attendre ?
\end{enumerate}
\end{exercise}
\end{document}
%%% Local Variables:
%%% mode: latex
%%% TeX-master: "master"
%%% End:

Binary file not shown.

View File

@ -0,0 +1,165 @@
\documentclass[a4paper,10pt]{article}
\usepackage{myXsim}
% Title Page
\title{DS8 \hfill \Var{Nom}}
\tribe{TST sti2d}
\date{\hfillÀ render pour le vendredi 9 avril à 10h au plus tard}
\xsimsetup{
solution/print = false
}
\begin{document}
\maketitle
\begin{exercise}[subtitle={Étude de fonction}]
%- set a = Integer.random(-5, 5)
%- set A = (a/2).decimal
%- set x1 = Integer.random()
%- set x2 = Integer.random(rejected=[0, 1, x1])
%- set b = -1* a*(x1+x2)
%- set c = a*x1*x2
%- set N = Polynom.from_coefficients([c._mo, b._mo, a._mo])
On considère la fonction $f$ définie sur $\intOF{0}{+\infty}$ par $ f(x) = \Var{A}x^2 + \Var{b}x + \Var{c}\ln(x)$
\begin{enumerate}
\item Démontrer que la dérivée de $f$ est $f'(x) = \frac{\Var{a}x^2 + \Var{b}x + \Var{c}}{x}$.
\item Étude du numérateur de $f'(x)$: $N(x) = \Var{N}$
\begin{enumerate}
\item Démontrer que $x=\Var{x1}$ et $x=\Var{x2}$ sont deux racines de $N(x)$..
\item Proposer une forme factorisée de $N(x)$.
\item Proposer une forme factorisée de $f'(x)$.
\end{enumerate}
\item Étudier le signe de $f'$ et en déduire les variations de $f$.
\end{enumerate}
\end{exercise}
\begin{solution}
\begin{enumerate}
\item pas de correction disponible
\item
\begin{enumerate}
\item \[N(\Var{x1}) = \Var{N(x1)}\]
\[N(\Var{x2}) = \Var{N(x2)}\]
\item \[
N(x) = \Var{a}(x - \Var{x1})(x - \Var{x2})
\]
\item
\[
f'(x) = \frac{\Var{a}(x - \Var{x1})(x - \Var{x2})}{x}
\]
\end{enumerate}
\item Pas de correction disponible
\end{enumerate}
\end{solution}
%- set I = sympy.I
%- set latex = sympy.latex
%- set sqrt = sympy.sqrt
%- set exp = sympy.functions.exp
%- set integrate = sympy.integrate
\begin{exercise}[subtitle={Complexes}]
\begin{enumerate}
%- set z_num = randint(2, 10) + I*randint(2, 10)
%- set z_denom = -randint(2, 10) + I*randint(2, 10)
%- set z1 = z_num / z_denom
\item Mettre le nombre complexe suivant sous forme algébrique $z_1 = \dfrac{\Var{latex(z_num)}}{\Var{latex(z_denom)}} $
%- set base = choice([(1, sqrt(3)), (sqrt(2), sqrt(2)), (sqrt(3), 1)])
%- set z2 = randint(1, 10)*(choice([1, -1])*base[0] + choice([1, -1])*base[1]*I)
\item Mettre le complexe suivante sous forme exponentielle $z_2 = \Var{latex(z2)}$
%- set base = choice([(1, sqrt(3)), (sqrt(2), sqrt(2)), (sqrt(3), 1)])
%- set z3 = randint(1, 10)*(choice([1, -1])*base[0] + choice([1, -1])*base[1]*I)
\item Mettre le complexe suivante sous forme exponentielle $z_3 = \Var{latex(z3)}$
%- set z4 = z2*z3
\item Calculer le produit $z_4=z_2\times z_3$ donner le résultat sous forme exponentielle puis algébrique.
%- set z5 = z2/z3
\item Calculer le quotient $z_5=\frac{z_2}{z_3}$ donner le résultat sous forme exponentielle puis algébrique.
\end{enumerate}
\end{exercise}
\begin{solution}
\begin{enumerate}
\item $z_1 = \Var{latex(sympy.re(z1) + sympy.im(z1)*I)}$
\item $z_2 = \Var{latex(sympy.Abs(z2))} e^{\Var{latex(I*sympy.arg(z2))}}$
\item $z_3 = \Var{latex(sympy.Abs(z3))} e^{\Var{latex(I*sympy.arg(z3))}}$
\item $z_4 = \Var{latex(sympy.Abs(z4))} e^{\Var{latex(I*(sympy.arg(z2) + sympy.arg(z3)))}} = \Var{latex(sympy.re(z4) + sympy.im(z4)*I)} = \Var{latex(sympy.N(sympy.re(z4), 3)+ sympy.N(sympy.im(z4), 3)*I)}$
\item $z_5 = \Var{latex(sympy.Abs(z5))} e^{\Var{latex(I*(sympy.arg(z2) - sympy.arg(z3)))}} = \Var{latex(sympy.re(z5) + sympy.im(z5)*I)} = \Var{latex(sympy.N(sympy.re(z5), 3)+ sympy.N(sympy.im(z5), 3)*I)}$
\end{enumerate}
\end{solution}
\begin{exercise}[subtitle={Sortie du congélateur}]
%- set Tp = randint(15, 25)
%- set T0 = randint(-20, -15)
%- set T15 = randint(-4, 4)
Marie a invité quelques amis pour le thé. Elle souhaite leur proposer ses macarons maison.
Elle les sort de son congélateur à $\Var{T0}$~\degres C et les place dans une pièce à $\Var{Tp}$~\degres C.
Au bout de 15 minutes, la température des macarons est de $\Var{T15}$~\degres C.
\bigskip
\textbf{Premier modèle}
\medskip
On suppose que la vitesse de décongélation est constante : chaque minute la hausse de
température des macarons est la même.
Estimer dans ce cadre la température au bout de $30$~minutes, puis au bout de $45$~minutes.
Cette modélisation est-elle pertinente?
\bigskip
\textbf{Deuxième modèle}
\medskip
On suppose maintenant que la vitesse de décongélation est proportionnelle à la différence
de température entre les macarons et l'air ambiant (il s'agit de la loi de Newton).
On désigne par $\theta$ la température des macarons à l'instant $t$, et par $\theta'$ la vitesse de décongélation.
L'unité de temps est la minute et l'unité de température le degré Celsius.
\smallskip
On négligera la diminution de température de la pièce et on admettra donc qu'il existe un
nombre réel $a$ tel que, pour $t$ positif :
\[\theta'(t) = a [\theta(t) - \Var{Tp}]\quad (E)\]
\medskip
\begin{enumerate}
\item Vérifier que l'équation $(E)$ a pour solutions $\theta(t) = K e^{at} + \Var{Tp}$$K$ est un nombre réel.
Donner alors, en fonction de $a$, l'ensemble des solutions de $(E)$.
\end{enumerate}
On rappelle que la température des macarons à l'instant $t = 0$ est égale à $\Var{T0}$~\degres C et que, au bout de $15$~min, elle est de $\Var{T15}$~\degres C.
\begin{enumerate}
\setcounter{enumi}{1}
%- set k = T0 - Tp
\item En utilisant la condition à $t=0$ démontrer que $K = \Var{k}$.
%- set a = round(log((T15 - Tp)/k)/15, 2)
\item En utilisant la condition à $t=15$ démontrer que $a \approx \Var{a}$.
\item En déduire l'expression de la solution de l'équation différentielle puis étudier ses variations.
\item La température idéale de dégustation des macarons étant de $\Var{Tp-3}$~\degres C, Marie estime que
celle-ci sera atteinte au bout de $30$~min. A-t-elle raison ? Justifier la réponse.
Sinon, combien de temps faudra-t-il attendre ?
\end{enumerate}
\end{exercise}
\end{document}
%%% Local Variables:
%%% mode: latex
%%% TeX-master: "master"
%%% End: